Está en la página 1de 121

P.

Grebenienkait
E.Tumnait

Petr Grebenienkait
Erika Tumnait

Matematikos
korepetitorius
namuose

LCtOYKkA

Ar jau pasirengs 2
matematikos
egzaminui?
UDK 51 (075.3)
Gr 189 Pratarm

Pastati[j dviej met valstybini egzamin lidnoki


Leidinio autors: V V
rezultatai liudija, kad ne visiems abiturientams pavyksta rodyti, ko
Petr Grebenienkait - iauli S. alkauskio jie imoko mokykloje. Ojuk daugelis dirbo su korepetitoriais, kurie
reng egzaminui. Atsakymo klausim, kodl taip danai egzamino
vidurins mokyklos mokytoja ekspert; metu suklumpa abiturientai, neturi ir mokytojai. Taiaujie sutaria
Erika Tumnait - Panevio M. Karkos vidurins dl vieno - vienuolikt klas susirenka moksleiviai, kurie tikrai
mokyklos mokytoja ekspert. netobulai moka pagrindins mokyklos kurs. Tad natralu, kad
mokiniams prie valstybinius egzaminus nelengva pakartoti ar i
naujo mokytis prie kelet met ieit mediag.
Stai kodl Jums reikalinga i knyga - matematikos nam
korepetitorius. Jame glaustai ir aikiai pateikta teorin mediaga,
apimanti vidurins mokyklos matematikos kurs. Greta rasite ir
udavini su sprendimais.
Matematikos korepetitorius abiturientui pads savarankikai
pasikartoti ir mokytis sprsti udavinius namuose.
Mokomoji mediaga leidinyje idstyta suprantamai,
glaustai ir prieinamai kiekvienam moksleiviui.
Leidjaisitikin, kad korepetitorius -Js skmsgarantas
valstybini matematikos egzamin metu.
Skms/
Autors
ISBN 9986-705-90-8

P. Grebenienkait, 2002
E. Tumnait, 2002
Siaurs Lietuva, 2002
35. Logaritmins nelygybs 94
Turinys 36. Trigonometrins funkcijos 96
37. Pagrindini kamp trigonometrins funkcijos 98
Pratarm 3 38. Sudties formuls 102
1. Natralieji skaiiai 7 39. Sinuso ir kosinuso laipsniai 103
2. Dalumo poymiai 8 40. Trigonometrini funkcij sumos keitimas sandauga 103
3. Pirminiai ir sudtiniai skaiiai 8 41. Trigonometrini funkcij sandaugos keitimas suma 103
4. Didiausias bendrasis daliklis 9 42. Trigonometrini funkcij grafikai 105
5. Maiausias bendrasis kartotinis 9 43. Atvirktins trigonometrins funkcijos 105
6. Paprastosios trupmenos 10 44. Trigonometrins lygtys 107
7. Proporcijos 12 45. Trigonometrini lygi sprendimo metodai 107
8. Tiesioginis ir atvirktinis dviej dydi proporcingumas 13 46. Trigonometrini nelygybi sprendimas naudojantis
9. Deimtains trupmenos 14 grafikais 111
10. Procentai ir promils 17 47. Ivestin 114
11. Skaiiaibs 22
48. Kreivs liestins lygtis 114
12. Modulis 22
49. Diferencijavimas 114
13. Vienanariai ir daugianariai 24
50. Sudtins funkcijos ivestin 115
14. Lygtis 30
51. Diferencijavimo taisykls 116
15. Lygi sistemos su dviem kintamaisiais 31
52. Ivestins mechanin prasm 117
16. Laipsnis su natraliuoju rodikliu 35
53. Funkcijos tyrimas ivestins pagalba. Funkcijos
17. Kvadratins aknys 37
monotonikumo intervalai 117
18. Iracionalybs panaikinimas vardiklyje 39
54. Ekstremumai 118
19. Natralaus laipsnio aknys 41
55. Didiausia ir maiausia funkcijos reikm intervale ... 121
20. Laipsnis 44
56. Ivestini taikymas sprendiant vairius udavinius ... 123
21. Funkcijos 45
57. Pirmykt funkcija ir neapibrtinis inegralas 128
22. Elementariosios funkcijos, j grafikai 46
58. Pirmyki funkcij lentel 130
23. Kvadratins lygtys 50
59. Apibrtinis integralas 130
24. Iracionaliosios lygtys 54
60. Kreivalinijins trapecijos plotas 132
25. Nelygybs 56
61. Sukini tri skaiiavimas 135
26. Kvadratins nelygybs 61
62. Kampai 136
27. Lygtys su moduliu 65
63. Kampo pusiaukampin 138
28. Interval metodas 68
64. Talio teorema 139
29. Skaii seka 70
65. Lygiaonio trikampio savybs 142
30. Tekstini udavini sprendimas 76
66. Pusiaukratins savybs 143
31. Rodiklin ir logaritmin funkcija 81
67. Trikampi panaumo poymiai 144
32. Rodiklini lygi sprendimo metodai 86
68. Trikampio vidurin linija 146
33. Logaritmini lygi tipai 89
69. Lygiakratis trikampis 147
34. Rodiklins nelygybs 92
70. Statusis trikampis 149
71. Daugiakampis 1. Natralieji skaiiai
152
72. Lygiagretainis
154 Prie vardydami skaii, j suskirstome klases i deins
73. Rombas
58 puss kair po tris skaitmenis. Trys pirmieji deins puss
74. Staiakampis 1
158 skaitmenys sudaro vienet klas, trys kiti - tkstani, toliau -
75. Kvadratas
159 milijon, milijard klases ir t.t.
76. Trapecija
77. Apskritimas 162 1 milijonas = 1000000
164 1 milijardas = 1000000000
78. Apskritimas ir kampai
166
79. Apskritimas ir ties
Pvz.: a) skaiius 21078000108 skaitomas taip:
80. [brtinis ir apibrtims apskritimas 167
21 milijardas 78 milijonai 108;
81. Sinus, kosinus teoremos 168
b) 100006 skaitomas taip:
82. Taisyklingojo n-kampio plotas 170
imtas tkstani ei;
83. Stereometrija. Pagrindins aksiomos ir apibrimai 172
c) skaiius 10000006000 skaitomas taip:
84. Stereometrijos teoremos 174
176 10 milijard 6 tkstaniai.
85. Lygiagretumo poymiai
86. Tiess statmenumo ploktumai poymis 178
n enkl natralusis skaiius uraomas taip:
87. Trij statmen teorema I79 n
a na n-1
. ... a.a .10n"1 + ... + a J1O + a,(
1 n0 = a n10 + a n-1
88. Ploktum statmenumo poymis 180
89. Kampas tarp tiess ir ploktumos 181
Udavinys. Paeiliui suraomi natralieji skaiiai
90. Dvisienis kampas
18 123456789101112131415 ...
91. Briaunainiai S
92. Kubas 188 Koks skaitmuo yra 1999-oje vietoje?
189
93. Piramid
19 Sprendimas. Yra devyni vienaenkliai ir 90 dvienkli
94. Nupjautin piramid O
194 skaii, kuriems urayti reikia 9 + 90 2 = 189 skaitmen.
95. Ritinys
194 Iekomasis 1999-asis skaitmuo yra 1810-ojoje vietoje, jeigu
96. Kgis
195 skaitysime nuo pirmojo trienklio skaiiaus (100) pirmojo
97. Nupjautinis kgis
195 skaitmens.
98. Rutulys
201 Vadinasi, iekomasis skaitmuo yra 604-ojo trienklio skai-
99. Koordinai metodas
204 iaus pirmasis skaitmuo, t.y. 604 + 99 = 703. Pirmasis skaitmuo
100. Vektoriai ir koordinats
214 yra 7.
101. Aibs
215
102. Kombinatorika
103. Tikimybi teorija 219 Ats.: 7.
229
104. Statistika
28
105. Dydi sryi lentel 8
2. Dalumo poymiai 4. Didiausias bendrasis daliklis

Skaitmenys O, 2, 4, 6, 8 vadinami lyginiais, o skaitmenys 1, Norint surasti keli natralij skaii bendrj didiausi
3, 5, 7, 9 - nelyginiais. dalikl (DBD), reikia:
1. Jei skaiius baigiasi nuliu, tai jis dalijasi i 10. 1. Iskaidyti juos pirminiais dauginamaisiais.
2. Jei paskutinysis skaitmuo yra O arba 5, tai skaiius dalijasi 2. Surayti tuos dauginamuosius, kurie eina kiekvieno
i 5. skaiiaus skaidin.
3. Jei paskutinysis skaitmuo yra lyginis, tai skaiius dalijasi 3. Rasti vis dauginamj sandaug.
i 2.
4. Jei skaitmen suma dalijasi i 3, tai skaiius dalijasi i 3. Pvz. Raskite DBD (6600; 6300).
5. Jei skaitmen suma dalijasi i 9, tai skaiius dalijasi i 9. 6600 = 2 - 2 2 - 3 - 5 - 5 - 1 1
6. Jei i dviej paskutinij skaitmen sudarytas skaiius 6300 = 2 2 _ 3 3 5 5 7
dalijasi i 4, tai ir visas skaiius dalijasi i 4. DBD (6600; 6300) = 2 2 3 5 5 = 300
7. Jei i trij paskutinij skaitmen sudarytas skaiius
dalijasi i 8, tai ir visas skaiius dalijasi i 8.
8. Jei skaiius dalijasi i 2 ir 3, tai jis dalijasi i 6. 5. Maiausias bendrasis kartotinis
9. Skaiius anan~ ... a1a0 dalijasi i 11 tada ir tik tada, kai jo
algebrin suma aQ - a^ + a2 - a3 + ... + (-1) n a n1 dalijasi i 11. Norint surasti keli natralij skaii maiausi bendrj
10. Skaiius anan~ ... a,a 0 dalijasi i 7 tada ir tik tada, kai kartotin (MBK) reikia:
(a0 + Sa1 + 2a2) - (a3 + 3a4 + 2a s ) + ... dalijasi i 7. 1. Iskaidyti juos pirminiais dauginamaisiais.
2. Surayti vieno skaiiaus skaidinio dauginamuosius.
Pvz.: 3. Prirayti kit skaii skaidini dauginamuosius, kuri
a) 546 dalijasi i 3, nes 5 + 4 + 6 = 15, o 1 5 dalijasi i 3; nra pirmajame skaidinyje.
b) 86736 dalijasi i 4, nes 36 dalijasi i 4; 4. Rasti vis dauginamj sandaug.
c) 2862 dalijasi i 6, nes 2862 dalijasi ir i 2, ir i 3.
Pvz.: Raskite MBK (360; 825).
1. 360 = 2 - 2 . 2 - 3 - 3 - 5
825 = 3 - 5 - 5 - 1 1
3. Pirminiai ir sudtiniai skaiiai 2. Suraome dauginamuosius:
2-2-2-3-3-5
Natralusis skaiius vadinamas pirminiu, jei jis turi tik du 3. Priraome 5 ir 11
daliklius: vienet ir pat save.
4. MBK (360; 825) = 2 - 2 - 2 - 3 - 3 - 5 - 5 - 1 1 = 19800.
Pvz. Skaiius 19 turi du daliklius 1 ir 19.

Udavinys.
Natralusis skaiius vadinamas sudtiniu, jeigu jis turi
daugiau negu du daliklius. Dviej skaii suma sudta su j didiausiu bendruoju
Pvz. 6 turi 4 daliklius 1, 2, 3 ir 6. dalikliu ir maiausiu bendruoju kartotiniu. rodykite, kad skaiius
a + b + (a; b) + [a; b] negali bti lygus 999999.
rodymas. JL . =JLlii
Jei suma a + b nelygin, tai vienas i skaii a ar b yra b 'd b-c
lyginis, kitas nelyginis. Tada (a; b) nelyginis, o [a; b] lyginis. Vadinasi,
suma a + b + (a; b) + [a; b] lygin ir negali bti lygi 999999.
Jei suma a + b lygin, tai natralieji skaiiai a ir b yra arba
lyginiai, arba nelyginiai.
Pirmuoju atveju ir (a; b), ir [a; b] yra lyginiai, o antruoju atveju
- nelyginiai.
Pvz.:
Vadinasi, abiem atvejais suma a + b + (a; b) + [a; b] yra
lygin ir negali bti lygi 999999.
1 ) 1 9 | + 2 4 ; | = ( 1 9 + 24) + ( 1 + ) =
Pastaba. DBD (a; b) sutrumpintai galima ymti (a; b), MBK = 443 + 1/ J l +1 \ = 43 + _4 + 1 7 = HH
J
(a; b) sutrumpintai galima ymti [a; b]. 24 24 ' 24 24 24
o u 5 0 2 _ 29 12 _ 29 12 _292 _ 58
2 4
Udavinys. ) 6 '2S 6 ' T 5~ " T "
Raskite natralij skaii poras, kuri DBD - 5, o MBK - 3 1. 3 = 1. = = 1
105. 3 J
3 1 ' 3
1
Skaiiai y ir 3 vadinami tarpusavyje atvirktiniais.
Ats.: 5 ir 105; 15 ir 35; 35 ir 15; 5 ir 105.
Du skaiiai, kuri sandauga lygi 1, vadinami tarpusavyje
atvi rktiniais.
6. Paprastosios trupmenos
A\ A o l = 1 . - 1 1 = I . A = 9 ' 4 - 92 _ 18. _ z
' 2 4 2 '4 2*11 2-11 1-11 11 11
Paprastoji trupmena - tai reikinys , kur a ir b - sveikieji
skaiiai, b t 0. Sudedant ar atimant trupmenas su skirtingais vardikliais
reikia i pradijas subendravardiklinti (imamas bendras vardiklis,
Pagrindin s a v y b lygus trupmen vardikli maiausiam bendrajam kartotiniui).

a _ a-k _ a Pvz.
b b'k b: (k*0, biO)
_7_ . H _ 7-5 11-4 _ _35_ + 44_ _ 35 + 44 _ 79
24 30 24-5 30-4 120 120 120 120
Veiksmai
(d (b
a + c _ a+ c a + c _ ad + bc
b - b b b-d bd

. JL- = a ' c
b' d b-d
7. Proporcijos 8. Tiesioginis ir atvirktinis dviej
dydi proporcingumas
Lygyb -- = vadinama proporcija, a ir d - kratutiniai
proporcijos nariai, b ir c - viduriniai nariai. Be to, a, b, c ir d nelygs Du dydiai yra tiesiogiai proporcingi, jei vienam dydiui
nuliui. didjant (majant) kelet kart ir kitas dydis didja (maja) tiek
pat kart.
Proporcijos savybs
Pvz.
Jeigu ^ = , tai: 6 cm3 trio plieninio rutuliuko mas - 46,8 g. Raskite 2,5 cm3
trio plieninio rutuliuko mas.
1. ad = bc
Sprendimas.
9 A = V1 = 6 cm3, ITi1 = 46,8 g
b a
V2 = 2,5 cm3, m 2 = x(g)
o J _ b^
-5, c d A -L
V2 "m 2
, a-b _ c - d
b d <=> = 19,5 g
a+b _ c+ d
a-b c-d
Ats.: 19,5 g.
. . . g a, 3a, an . a. + a + ... + an a
6. Jei - == -i?= ... = Kil
, tai =
; - ;> b, + 2 + ... + - ; Du dydiai atvirkiai proporcingi, jei vienam dydiui
Pvz. didjant (majant) kelet kart kitas dydis maja (didja) tiek
Tarp miest atstumas vietovje yra 200 km, o emlapyje - pat kart.
5 cm. Kokio ilgio atkarpa emlapyje galima paymti 700 km
atstum? Pvz.
Keturi kombainai gali nuimti derli per 10 dien. Per kiek
Sprendimas. dien nuimt derli penki tokie kombainai?
Iekomos atkarpos ilg ymime x, tada:
Sprendimas.
5 _ 200 5-700 _ 17-rm n,= 4 kombainai, ^ = 10 d.
T - 700 X
- ~~2 - 1f b c m
'
n2 = 5 kombainai, t2 = d.
Ats.: 17,5 cm.
J k -t^ ^ 4
- x
~ y- 4-10
- ft
P
Ats.: 5 kombainai nuims derli per 8 dienas.
Udavinys. Dalijant deimtain trupmen i deimtains reikia dalinio ir
Trij trupmen skaitikliai proporcingi skaiiams 1; 2 ir 5, o daliklio kablel perkelti dein pus per tiek skaitmen, kiek j yra
vardikliai atitinkamai proporcingi skaiiams 1; 3 ir 7. i trupmen po kablelio daliklyje.
aritmetinis vidurkis lygus . Raskite trupmenas.
4) 0,32 : 0,8 = 3,2 : 8
Ats- A - A - 2 0 3,2 L8__
^ l b " 7 1 21 ' 49 -

~_0 0,4
32
9. Deimtains trupmenos
0
Baigtine deimtaine trupmena vadinama trupmena, kurios
vardiklis yra 10, 100, 1000 irt.t. 5) 9,541 : 4,7 = 95,41 : 47
95,41 47
Uraoma taip: 94 2,03
141
7
^=7'3= = 008; W = 0
017 141
0
Veiksm su deimtainmis trupmenomis pavyzdiai:
Paprastj trupmen veriant deimtaine reikia skaitikl
1 ) + 0,027 2) 5,00 padalinti pagal deimtains trupmenos dalybos i sveikojo skaiiaus
25,241 "0,06 taisykl.
25,268 4,94
Pvz.
0,027 + 25,241 = 25,268 5 - 0,06 = 4,94
f = 5 : 8 = 0,625
Dauginant dvi deimtaines trupmenas reikia sudauginti 8
skaiius nekreipiant dmesio kablelius, po to i deins puss 5,0 |_8
atskirti kableliu tiek gauto rezultato skaitmen, kiek j i viso yra 48 0,625
po kablelio abiejuose dauginamuosiuose. 20
"16
3) 7,12 40
x
86 ~40
4272 0
5696
612,32
Kartais gaunama begalin deimtain trupmena, kurios
skaitmenys, pradedant nuo kurio nors skyriaus, kartojasi.
7,12-86 = 612,32
Pvz. 2,0 |_3 10. Procentai ir promils
0_ 0,666...
20 imtoji vieneto dalis vadinama procentu.
18^
20 1%= 4 = 0,01
" 18
20 10% = =0,1

Vadinasi, = 0,(6) - i trupmena vadinama begaline 25% = = 0,25


deimtaine periodine trupmena.
50% = ^ = 0,5
Periodin trupmen veriant paprastja, reikia i skaiiaus
10% 1
iki antrojo periodo atimti skaii iki pirmojo periodo ir gaut skirtum = W =
urayti skaitiklyje, o vardiklyje parayti tiek kart skaitmen 9, kiek Tkstantoji vieneto dalis vadinama promile.
yra skaitmen periode, ir prirayti tiek nuli, kiek yra skaitmen
tarp kablelio ir pirmojo periodo.
1 %
= 0
' 0 0 1
45-0 JL
Pvz.: 0,(45) = 11
99 10%
= TSSS = 0.01,
3173 31 3142 1571
3 1(73) = - -
990
= So = ' 0 2 5
990 495 25%

Udaviniai.
Pvz.:
1. Ireikkite paprastja trupmena:
a) 0,(13) b) 3,(1) c) 2,0(15) 1) Mokykloje mokosi 800 moksleivi. 46% j yra mergaits.
Kiek mokykloje mokosi mergaii?

Sprendimas.
2. Atlikite veiksmus: 46% = 0,46
a)0,(5)-0,(2); b) 4,(2)-2,(4)
800 0,46 = 368 mergaits.

Ats.: a ) | ; b ) 1 - | .
Ats.: 368 mergaits.
131 1 Taisykl. Norint rasti p% skaiiaus a, reikia p% ireikti
3. Isprskite lygt: 3 (5) + ^ 2 0(15) = Q ^
deimtaine trupmena, po to skaii a padauginti i gautosios
trupmenos.

-16- -17 -
2) respublikin jaunj matematik olimpiad atvyko 140 Sudtini p r o c e n t formul
moksleivi, o tai sudar 3,5% norinij joje dalyvauti. Kiek
moksleivi norjo dalyvauti respublikinje jaunj matematik +
S = Sod W ) '
olimpiadoje?
ia S 0 - pradinis dydis, S - dydis po t laiko vienet (kart),
Sprendimas. p - pradinio dydio didjimo procentas.
I pradi surandame 1% vis norinij: Jei pradinis dydis S0 kiekvien laiko vienet (kart) maja
140 : 3,5 = 40 moksleivi. tuo paiu procentu, tai formul atrodo taip:
Randame vis norini dalyvauti moksleivi skaii:
40 100 = 4000 moksleivi. S = S0(1-4)'

Ats.: 4000 moksleivi. Pvz.

Taisykl. Norintsurasti vis skaii, kai inoma jo dalis b, ir Miesto gyventoj skaiius per dvejus metus padidjo nuo
j atitinkantys p%, reikia p procent ireikti deimtaine trupmena, 20 000 iki 22 050. Apskaiiuokite, kiek procent vidutinikai per
po to skaii b padalinti i gautosios trupmenos. metus padidjo miesto gyventoj skaiius.

3) 30 mokini, kurie sudar 15% vis dalyvi, buvo Sprendimas.


apdovanoti. Kiek mokini dalyvavo olimpiadoje?
raome formul S0 = 20 000, S = 22 050, t = 2:
Sprendimas.
30 : 0,15 = 200 (mokini). 22 050 = 20 000(1 + ^ ) 2
,

Ats.: 200 mokini. ( 1 + ^ ) 2 = 1,1025,

Taisykl. Norint rasti skaii a ir b procentin santyk, reikia 1 + T&r = 1.05.


skaii santyk padauginti i 100%.
P =- 0,05,
100
4) Batai atpigo nuo 200 Lt iki 150 Lt. Keliais procentais atpigo
p = 5%.
batai?
Pavyzdiai.
Sprendimas.
1. I pradi produkcijos vieneto kaina buvo 75 Lt. Per
Batai atpigo 200 - 150 = 50 (Lt).
pirmuosius metus kaina iaugo tam tikru procentu, o per antruosius
Tada -50.. 100% = 25%. metus ji sumajo tuo paiu procentu, dl to jos vert tapo 72 Lt.
200
Raskite t procent.
Ats.: 25%.
Sprendimas. P = 25 -1,5,
p = 37,5.
Sakykime, kad produkcijos kiekio kaina iaugo x%. Po
Ats.: 37,5%.
75.X

padidjimo jo kaina tapo (75 + ). 4. Kiek pinig reikia padti bank u 5% metini sudtini
palkan, kad po 4 met susidaryt 6000 Lt suma?
Po sumajimo ji tapo (75 + - ) - (75 + ) ^
Sprendimas.
Tada ( 7 5 + ^ - ) - ( 7 5 + )-^=72
7c; + 75x 75x 75x2 _ 72 10 000 6000 = x(1 + y
100 " 100 ~ 10000

75x 2 = 30 000 X= ^ = 4936,24 Lt

X2 = 400 Ats.: 4936,24 Lt.

= 20
Udaviniai.
Afs.: 20%.
1. Dviej skaii suma yra 1240. Vieno skaiiaus 4,5% yra
2. Lenktynininkas apskaiiavo, kad, padidins greit 10%, lygus kito skaiiaus 5,5%. Raskite tuos skaiius.
jis turi apvaiuoti rat per 15 minui. Keliais procentais jis turi
padidinti greit, kad apvaiuot rat per 12 minui? Ats.: 682 ir 558.

Sprendimas. 2. Kovo mnes preks atpigo 20%, o balandio mnes dar


10%. Keliais procentais atpigo preks per du mnesius?
Sakykime, kad greitis yra v, o j padidinti reikia p%. Tada:
Ats.: 28%.
12(v + ^ - ) = 15(v + 0,1v),
3. Du kartus sumainus preks kain tuo paiu procentu, ji
12(v + -100
) = 15 1,1 v : 3v atpigo keturis kartus. Keliais procentais buvo mainama preks kaina
kiekvien kart?
4 + g = 5,5,
Ats.: 50%.
JL = -1 5
25
11. Skaii aibs ( + ) + ( + ) = (+)
(-) + (-) = (")
N - natralieji skaiiai: N = {1, 2, 3 ...} a + (- a) = 0
Z - sveikieji skaiiai: Z = {... - 4; - 3; - 2; - 1 ; 0; 1; 2; 3; 4 ...} a - b = a + (- b)
Q - racionalieji skaiiai: Q = { ^ ; m e Z, n e N} m - (- n) = m + n
R - realieji skaiiai: ( + ) ( + ) = (+)
R = K +
V Z} (-) ( + ) = (-)
( + ) (-) = (-)
av a 2 , a 3 , ...e {0; 1; 2; ... ; 9} (-) (-) = (+)

C - kompleksiniai skaiiai Pvz.:


C = {a + bi: a, b e R, i = V- 1 } 1) . 6 + ( - 4 ) = - 10;
2 ) 3 + (-10) = - 7 ;
3 ) - 3 - ( - 4 ) = - 3 + 4 = 1;
12. Modulis 4 ) - 9 , 6 - ( - 5,1) = - 9 , 6 + 5,1 = - 4 , 5 ;

5) v( - 1 ) . v( - 1 ) = 1 . 1 = J L ;
' 2' 4 ' 2 4 8
6)M ( - 5 V A = - ( 1 . 1 ) = .5 = . -|1
6 3 6 2 ' 4" 4
Pagrindins modulio s a v y b s

|a| >- 0 |- a| = a |a - b| = | b - a| Udaviniai.

|a| -1 b|,<| a + |a | + |b |. 1. Neatlikdami skaiiavim padkite enklus >, < arba =


tarp reikini:
a ) - 4 3 3 6 : (- 54) ir 4336-(- 54);
Veiksmai su teigiamaisias ir neigiamaisiais skaiiais b) 1738-(- 69) ir 1738 : (- 69);
c ) - 8 2 5 4 - 1 9 ir 8254 : ( - 19).
Norint sudti du skaiius su skirtingais enklais, reikia:
1) nustatyti t skaii moduli didesnj modul; Ats.: a) >; b) <; c) <.
2) i didesniojo modulio atimti maesnj;
2. Per tris dienas oro temperatra pakito 4C. Per pirmsias
3) prie gaut skaii parayti enkl to dmens, kurio
dvi dienas ji sumajo 7C, o per antrj ir treij dienas ji sumajo
modulis didesnis.
1C. Kaip kito oro temperatra kiekvien dien?

Ats.: Pirmjdien pakilo 3C, antrj dien sumajo 100C,


o treij dien pakilo 110C.
13. Vienanariai ir daugianariai Skirtumo kubas
koeficientas laipsnio rodiklis
(a - b)3 = a3 - 3a2b + 3ab2 - b 3
T t
a + a + a = 3a a a a = a3 Kub suma
l J I I
dmuo laipsnio pagrindas a3 + b 3 = (a + b)(a2 - ab + b 2 )

3 kartus
Kub skirtumas
a3 - b3 = (a - b)(a2 + ab + b 2 )
1a = a

Vienanari pvz.: 2a; 4a3b; ^ab 3 . (a + b + c)2 = a 2 + b 2 + c2 + 2ab + 2ac + 2bc

5 a + b
Daugianari pvz.: 2a + be; 3a(b + c); ( > .
3 Daugianario skaidymas dauginamaisiais
3 2 3+2 5
a a = (a a a) (a a) = a =a.

c o a-a-a-a-a e ->
Bendrojo daugiklio iklimas prie skliaustus:
a 5 : a2 = a
a* r = a a a = a 5 - 2 = a3 . 1) a m + b m = m(a b);
2) 4ab + 14a2 - 2a = 2a(2b + 7a - 1).

Veiksmai su daugianariais Grupavimo metodas:


1) a 2 + 6ab + 9b2 = (a + 3b)2;
Daugianari sudtis: (a3 + ab - b) + (4a3 - 3ab + b) = 5a3 - 2ab. 2) a4 + 4 = a4 + 4a2 + 4 - 4a 2 = (a2 + 2)2 - (2a)2 =
= (a2 - 2a + 2)(a2 + 2a + 2).
Daugianari atimtis: (3a - b) - (4a + b) = 3a - b - 4a - b = - a - 2b.
Papildomos
n
formuls:
n 1 n 2 2
Daugianari daugyba: (a - 3b)(a + b) = a 2 - 3ab + ab - 3b2 = a - 1 = (a - 1)(a " + a "1 + ... a + a + 2 1). 3 3i
= a2 - 2ab - 3b2. a - b = (Va - VtT)(Va + VB ) = (a b ' + a b1 + b ).

Sutrumpintos daugybos formuls Pvz.:

Sumos kvadratas 1) 4 - xy = x(x3 - y);


(a + b)2 = a 2 + 2ab + b2 2) 4 + 3x3y + 3x2y2 + xy3 = x(x3 + 3x2y + 3xy2 + y3) =
= x(x + y)3.
Skirtumo kvadratas 3) 9a2 - 2 + 10xy -25y 2 = 9a2 - (x2 - 10xy + 25y2) =
(a - b)2 = a 2 - 2ab + b 2 = 9a2 - ( - 5y)2 = (3a)2 - (x - 5y)2 =
= (3a - + 5y)(3a + - 5y);
Sumos kubas
(a + b)3 = a3 + 3a2b + 3ab2 + b 3
Dalijimas "kampu":
4) Suprastinkite reikin ^ + ^ ^
3x3 - 2 - 3x - 2 2 + - 1
Sprendimas. 3x3 +3x2 - 3x 3x - 4
a + 3b a - 3b _
_-4x 2 -2
+
2
(a - b)2 a2 - b 2 (a - b): - 4x - 4x + 4
4x - 6
a2 +ab + 3ab + 3b2 +a 2 - ab - 3ab + 3b 2
(a - b)2(a + b) P(x) = 3x3 - 2 - 3x - 2, M(x) = X2 + + 1; Q(x) = 3x - 4;
2a 2 + 6b2 R(x) = 4x - 6.
(a - b) 2 (a + b)

Bezu teorema
Daugianari dalijimas
Liekana, gauta daugianar dalijant i dvinario - a, lygi a
3x2 - 7x - 26 + 2 daugianario reikmei, kai = a, t.y. R = f(a).
3x2 +6x 3X-13 f(x) = ( - a)Q(x) + f(x)
- 13x - 26
"- 13x - 26 Pvz.
O Neatlikdami dalybos veiksmo raskite liekan
(2x4 + 3x2 + - 5) : (x + 2).
3x2 - 7x - 26 = (x + 2)(3x - 13).
Sprendimas.
raome reikm = - 2 dalin ir gauname:
Dalyba su liekana 2- (- 2)4 + 3(- 2)2 + (- 2) - 5 = 37.

P(x) = M(x) -Q(x) + R(x), kur P(x) - dalinys, M(x) - daliklis, Ats.: 37.
Q(x) - dalmuo, R(x) - liekana.
Hornerio schema
Pvz.
Pvz.
3x3 - X 2 - 3x - 2 = ( 2 + - 1)(3x - 4) + (4x - 6). Padalykite daugianar f(x) = x3 + 5x2 - 3 i (x - 5).
P(x) M(x) Q(x) R(x) ia a = 5,
b2 = a3,
b, = b2- 5 + a2,
b
o = 5 + av
R = b0- 5 + a0,
R = f(5) = 247.
Daugianar f(x) = anx" + an ^ x n " 1 + ... + a ^ + a 0 dalijame todl padalijus i (x + 1)(x - y ), gauname lygt:
i - a. Sudarome kintamojo koeficient lentel. 4x 3 + 2x2 + 2x - 2 = O, kurios aknis yra skaiius y .
Gautosios lygties abi puses padalin i - y , gauname
an a., ... an kvadratin lygt 4x2 + 4x + 4 = O, kuri reali akn neturi.
a b .=a b
n-2 n-1= a + b
n-3= a
n-2 + ... R = a0 + Vadinasi, duotosios lygties aknys yra skaiiai - 1 ir y .
+ atV1 + abn -2 + ab0
Ats.: - 1 ir y .
3
? a
i a
O
1 5 0 -3 2) Raskite, su kuria m reikme lygties
5 1 10 50 247
z3 - (m2 - m + 7)z - (3m2 - 3m - 6) = O viena aknis lygi - 1 .

b
o R
2 1
Raskite kitas dvi lygties aknis su ia m reikme.
Daugianario aknimi vadinamas toks skaiius x0, su kuriuo
Sprendimas.
daugianario reikm lygi nuliui (f(x0) = 0).
z = -1,
Sveikosios daugianario su sveikaisiais koeficientais aknys - 1 + m2 - m + 7 - 3m 2 + 3m + 6 = O
yra jo laisvojo nario dalikliai. m2 - m - 6 = O
m1 = - 2, m 2 = 3.
Pvz.
Su iomis reikmmis duotoji lygtis bus:
X3+ 5x2 + 2x - 8 = O
z3 - 13z - 12 = O, z 3 - z - 1 2 z - 1 2 = 0,
Sveikj akn galima iekoti tarp skaii:
z(z2 - 1) - 12(z + 1 ) = 0
-1; 1; 2; - 2; 4 ; - 4 ; 8; - 8.

(z + 1)(z2 - z - 12) = 0
Ats.: x = 1 ; x = - 2 ; x = - 4 aknys.
z = - 1 arba z = - 3; 4.
Jei nesuprastinama trupmena yra daugianario su sveikais
koeficientais aknis, tai jo laisvasis narys dalijasi i p, o vyriausias Ats.: z = - 3; 4.
koeficientas i q.

Udaviniai.
Pvz.:

1. Atlikite veiksmus: (3a3 -2a 2 b + ab2)(2a2 - ab - 5b2).


1) Isprskite lygt 4x 5 + 4x4 + x 3 - 2x2 - 2x + 1 = 0 .

Ats.: 6a5 - 7a4b - 11a3b2 + 9a2b3 - 5ab4.


Sprendimas.
Sakykime, f(x) = 4x 5 + 4x 4 + x 3 - 2x2 - 2x + 1. Lygties
2. Iskaidykite dauginamaisiais: 1 + a4 + a 8 .
racionaliosios aknies skaitiklis gali bti tik + 1, o vardiklis - skaiiai
1, 2 ir 4.
Ats.: (1 + a4 + a 2 )(1 + a 4 - a2).
Vadinasi, lygties racionaliosios aknys gali bti kai kurie i
skaii + 1; + j , ^ Tikrinant paaikja, kad f ( - 1 ) = f( 1 ) = O,
3. Atlikite veiksmus: ^ g L . 3) - 1 = 0,4
= 0,4 : ( - 1 ),
Ats.-H . X =-3,6.

14. Lygtis Udaviniai.

Lygtis - lygyb su kintamuoju. 1) 3x = 0,4. 3 ) - - : x = 2.


Sprendinys - kintamojo, su kuriuo lygyb teisinga, reikm.
Ats.: = = . Afs.:x=.
15
Dvi lygtys, kuri sprendiniai sutampa arba kurios j neturi,
vadinamos ekvivaleniomis. 2) f = - 0,7.

Savybs Ats.: = - 8 y

1) = <=> + a = + a
2) = <=> a = a (a 0) 15. Lygi sistemos su dviem kintamaisiais
3 ) + 0 = ^ = - 0
Sistema - tai kelios lygtys, kurioms reikia surasti bendr
Tiesin lygtis sprendin.

ax = b Sistemos sprendinys - tai skaii dvejetas, kuris tenkina


kiekvien lygt.
1)7 a 0, = - vienintel aknis
a
Sistemos ekvivalentikos, jeigu turi tuos paius
2) a = 0, b = 0; - bet kuris realusis skaiius (0 = 0)
sprendinius arba neturi j.
3) a = 0; b 0 sprendini nra
Sprendimo bdai

Pvz. 1. Grafinis
2. Keitimo
3. Sudties
Isprskite lygtis:

1 ) 6 - = 1,3, 2) 0,6 : = - 5,
= 6 - 1,3, X = 0,6 : (- 5),
= 4,7. X = - 0,12.
Tiesini lygi sistema Sprendimas.
Sprendiame sudties bdu. Sistemos pirmosios lygties abi
Ta 1 X + b,y = C11 puses padauginame i 3, o antrosios - i 2. Sudedame gautsias
I a 2 X + b 2 y = C2 lygtis panariui ir gaut rezultat paraome vietoje sistemos antrosios
lygties.
1. Jei 4r- "^ l , tai vienas sprendinys.
2 2
J 2 x + 5y = 4 I - 3 J 6 x + 15y = 12
2. Jei = ^ l = I 1 , tai sprendini be galo daug. L - 3x - 2y = 5 I 2 +
l - 6 x - 4 y = 10
2 2 2

0 + 11y = 22
3. Jei - = ^ * , tai sprendini nra.
2 + 5 = 4, 2x + 5 2 = 4 r x = -3,
I 11y = 22; Iy = 2 ^ I y = 2.
Pvz.:
Ats.: (- 3; 2).
1) Isprskite lygi sistem
| | + ! y = 5 3) Isprskite lygi sistem:
f + = 3,
l 3x - 2y = 1. I 2x + 2y = 4.

Sprendimas. Sprendimas.
Sprendiame keitimo bdu. I pirmosios lygties ireikiame
1 1 3
per y: Kadangi y = * j , tai duotoji sistema neturi sprendini.
| x = 5 - f - y o x = f(5-|y).
a n t r j lygt vietoje raome g a u t reikin, g a u n a m e lygt 4) Grafikai isprskite lygi sistem
su v i e n u neinomuoju: 3x + 2y = 5,
I 2x - y = 8.
3 | ( 5 - | y ) - 2 y = 1 < = > | " 5 - . - | - y - 2 y = 1 <=>y = 4.
Sprendimas.
skaii raome iraik = - | ( 5 y) ir gauname = 3. Briame lygties 3x + 2y = 5 grafik per du takus, pvz.,
( - 1 ; 4) ir (1; 1).
Ats.: (3; 4). Briame lygties 2x - y = 8 grafik per du takus
( 0 ; - 8 ) ir ( 2 ; - 4 ) .
2) Isprskite lygi sistem:
f 2x + 5y = 4,
I - 3x - 2y =5.
5. Raskite visas a reikmes, su kuriomis lygi sistema
(a + 3)x + 4y = 5 - 3a,
2x + (a + 5)y = 8

a) neturi sprendini;
b) turi be galo daug sprendini.

Ats.: a) a = - 7; b) a = -1.

16. Laipsnis su natraliuoju rodikliu

m, n e N
Ats.: (3; - 2).
am = a -a ... a, kai m > 2

Udaviniai. m
a1 = a

Isprskite:
a m an = a m + n

1. 5 + 4 = 2, gm g n g m -

I x - + 8 = 0 sudties bdu. am. bm = (g .b)m

Ats.: (-2\2). am : bm = ( f )m, kai b P 0

2. f 3 x + y = 6, (am)n = a m ' n

L2x + 3y - 4 = 0 grafikai. Jei a > b > 0, tai an > bn

Ats.: (2; 0). Pvz.:

3. f 12x - y = 3,5, 1) 0,2 4 = 0,2 - 0 , 2 - 0 , 2 - 0,2 = 0,0016


.. .L
, _3x
H . J+ Ly\ = 2 keitimo bdu.
AlS.. { 3Q , 10 J.
2 ) ( 1 ) 2 . ( 1 1 )3 = ( 1
)5 = (3)5

= 1
4- "f
3) (0,4)6 : (0,4)4 = 0,42 = 0,16

Afs.; (8; 9).


4) (25)2 = 210 = 1024 Udaviniai

5 W J . \4 = .4 = Apskaiiuokite:
\ 3 ' 3 81
1 1

6) Kas daugiau: 2300 ar 3200 ? 1) 8" + 16"

Sprendimas. 2) 3 5 - ( f ) - 4
2300 = (2 3 ) 100 = 8 100
32 = (32^100 = 3)-10-0,5"3
Kadangi 9 > 8, tai 3200 > 2300.
4) (2 7 ) 3 -2- 18

Laipsnis su sveikuoju rodikliu Ats.: 1)-^; 2) 19; 3) - 80; 4) 8 .

a" n = ^r 1 kur a 0 ir n > 0 Uraykite standartine iraika:

a = 1 1) 6,4 -10 3 -1,6 -10"2


2) 1,1 10 4 -9,3 -103
( f ) " = (|)n> k u r a ^ O , b ^ 0 3) 5 - 1 0 " 2 : (0,25-10 4 ).

Ats.: 1) 1,024-10" 4 ; 2) 1,023-10 8 ; 3) 2 - 1 0 " 5 .


Pvz.:

1)(-4)"3 =
" (- 4)3 -64 64 ' 17. Kvadratins aknys
3 3 3
2)(1 ^)" = (f)- = (|) = -f.
Kvadratins aknies apibrimas:
Standartin skaiiaus iraika

= a 10 , n e Z, a| < 10. a

Pvz.: V25 = 5, nes 5 2 = 25 7 = 0,4


/25 * 7, nes 72 jt 25 V6400 = 80
1) 654000 = 6,54-10 5 V25 ? - 5, nes - 5 < 0 VO1OOI6' = 0,04
2) 0,00034 = 3,4 -- 4 V- 4 neturi prasms 2 < VT7 < 3
3) 2,8 103 5 10- 6 = 14 10"3 = 1,4-10' 2 0,8 < /08 < 0,9
4) 3,5-10" 6 + 7,4-10" 7 = 3,5-10" 6 + 0,74 -10" 6 = 4,24-10"
Tapatybs Udaviniai.

1) Apskaiiuokite:
(Va ) 2 = a, a O
/2*= |a |, a e R
a) 2,5 - V 3 , 2 4 - V225"
Pagrindins savybs

/7Vb = V a - b '
v4fs.:aJ-3;bj4L.

2. Suprastinkite:
(Va ) p = VaP1
a) 4 - 2 V0,49x6', kai ^ 0
Vab = VlaT-Vlbi
b) - 5 Y 3 ZiF kai <
i[E = i j L h? O
Ib VibT '

Va?= (V|a7) p
18. Iracionalybs panaikinimas vardiklyje
7
Jei a > b 5-0, tai Va > Vb . Va + Vb ^ V a + b ' .
Norint panaikinti iracionalyb trupmenos vardiklyje, reikia
Jei a > 1, tai a > Va ir Va > 1
surasti paprasiausi i reikini, kurio sandauga su vardikliu bt
racionalusis reikinys, ir i jo padauginti duotosios trupmenos
Jei 0 < a < 1, tai a < Va7 ir 0 < Va < 1. skaitikl ir vardikl.

Pvz..
Iklimas i aknies klimas akn 2 _ 2 V* 2 V5
1) VF V5-V5
V i = |a|-Vb\b^0 = / - V g b J e i a <0,
/ 1 a j/a - V? Va
Va 2 B , jei a > 0, b > 0. 2) W - 3
Vap
a >0
V72" = V T T = 6V2 , . _
r 7 2 Jei trupmenos vardiklyje yra dvinaris a + b, tai reikia ir
VTli = |a|VTT 3V2 = V 2 ^ 3 = V l
skaitikl, ir vardikl padauginti i reikinio, jungtinio vardikliui.
Veb 1 " + VtT= bV6~+ Vb > - 2V6 = - V4 -6 = - V24 .
V5c 2 + V - c = - cVT+V- c , c ^ < 0
Pvz.:

1\ 2 _ 2(V3+ 1) _ 2(V3 + 1) _ 2(V3* + 1) _


''VT-I (VT-1)(V3+1) ~ (V3')2-12 ~ 3-1 / +
2) Suprastinkite reikin: 3. Apskaiiuokite:

t 2 + 3 , 15 \ / 1 . JL\-1 a \ 3 + 3
V3*-2 s-V^ +
51 ' 1 - 2V5 1 + 2V5* '

Sprendimas. Ite--W-

2
I 3 . 15 v /JL + J_ v 1 = b) T r l l f +
(^5 H- 2V2 )2.
* V3*-1 V3^2 3-V3" ' 5 )

- 2(^+1) _3(V3 + 2) . 15(3 + ") , 5 + 3~ y 1 _ Ats.: 16,5.


~ ' (V3*- 1)(/3' + 1) (V3-- 2)(V3' + 2) (3-/3 )(3 + V T ) " V - T T f - >
_ , 2(?+ 1) , 3(/+ 2) . 15(3 +VT) 5 _
+ +
~ V 3-1 3-4 9-3 /"+ ~
19. Natralaus laipsnio aknys
= (V3*+ 1 - 3V3"- 6 + 7,5 + 2 , 5 V 3 l - - 5 =
Va = b, n e N, a > 0 <=> b > 0, b n = a
=
= (0,5V3~+ 2,5) - ^ = 4 ^
Pvz.:
Ats.: 2,5^.
VTe = 2
Dviej neneigiam skaii aritmetinis vidurkis ne maesnis V243 = 3
u j geometrin vidurk. Vo,125^ = 0,5
^Vab , a ^O; b ^ O . 2 < V9 <3

Nelyginio laipsnio aknies traukimas i neigiamo skaiiaus


Udaviniai. 3, 1 3/ 7
V- 125 = - V125 = - 5
Panaikinkite iracionalyb vardiklyje:
V- 0,0000001'= - Vo,0000001' = - 0,1
a 10x
)(V3 - 2 ) 3 ' = -]/(2- V 3 > = - (2 - V3 ) = V i T - 2.
Ats.: 2^5 .
Lyginio laipsnio aknis i neigiamo skaiiaus neturi prasms.
o 58
Tapatybs
2
-^T

Ats.:-4^5-14. (Vi 1 )" = a


2
Va5"1= |a |, a e R

2n
"Va2n"1 '= a, a e R
Pagrindins savybs
b) - 3V27= - = - VsFY = - 4VW;
Vab = Va -Vb 1
c) (1 - V3 )-V2 = - V(1 - V 3 ) 4 - 2 ;

Ib W d) (1 - V5 ) - W = V(1 - V5 ) 3 -2 .
r
IC = n VF

"Va m k ' = Va1


Veiksm su skirting rodikli aknimis pavyzdiai

t
a) V 3 - - = W - W - ^ - ,;33 32- 3' = V3 = 3
akn palyginimas

Jei a > b >-0, tai:

Va 1 > VtT
Udaviniai.

Va + Vb 1 > Va + b '
1) Duotuosius skaiius suraykite didjimo tvarka:

J e i a > 1, tai Va* > 1 ir Va*< a


VlO ; 2 V 0 j ; 2,5.

J e i 0 < a < 1, tai 0 < VeT < 1 ir Va*> a


>Afs.: 2 V 0 5 ; VlO ; 3 ) ; 2,5.

2) Suprastinkite reikin 3 ^ + 2VT- V i T .


Pvz.:
>4fs.;4V5*.
Iklimas i poaknio:

a) V48 = V e 7 F = 2V<T;

b) V(2 - V ^ ) 4 ' = ( V i T - 2)V6 ;

c) V(1 - V3 ) 3 -6 = (1 - V3 )V6\

klimas poakn

a) 5>W = VS 3 7 Y = V250 ;
Udavinys.
20. Laipsnis
Kasdaugiau 3 ^ ar
Neneigiamo skaiiaus laipsnis su racionaliuoju rodikliu
J2. n,- . Sprendimas.
a n = "vam, m e Z, n e Z
Abu lyginamuosius skaiius pakeliame laipsniu V5 + 2.
Jei m < 0, tai a > 0.
Jei m > 0, tai a' ^ 0. Tada ( ^ - 2 ) ^ +2
= <^-2><^+2> = 3 < - - = ( f ) 4 < ( - f - ) ^ +

5^* = 42] Ais.: A .

0 " ^ = 0;
21. Funkcijos
0 , 0 9 ^ = /0)9 = 0,3.
y = f(x)
Laipsnis su realiuoju rodikliu
- argumentas, y - funkcijos reikm.
D(f) - apibrimo sritis, E(f) - reikmi sritis.
ar, r e R, f r < 0 ir r > 0
Funkcijos grafikas - vis tak (x; y), kur y = f(x), aib.
L a > 0 I a > 0

Funkcijos savybi schema


Laipsni savybs
1. D(f).
a p ar = a p+ r
(ap)r = apr
2. Funkcijos grafiko susikirtimo su aimis takai.
3. Pastovaus enklo intervalai (kur f(x) > 0 ir f(x) < 0).
a p : ar = a p ' r (|")- =
()
4. Lyginumas (f(- x) = f(x)).
Nelyginumas (f(- x) = - f(x)).
a r b r = (ab)r a r : b r = (|-) r
5. Periodikumas (f(x + T) = f(x)).
6. Didjimo intervalai (kuo didesnis x, tuo didesnis y).
Laipsni savybs ir nelygybs
Majimo intervalai (kuo didesnis x, tuo maesnis y).
7. Ekstremumai (max, min).
a > b >. 0 fa > b > 0
a > b a r < b r 8. Elgesys arti ypatingj tak arba +
I r > 0 ^ Ir < 0 ^
9. E(f).
10. Grafikas.
f P > r ^ ap>ar / P > r =^ a p < a '
3 3
La > 1 ^ \ 0 < a < 1
Pvz. 3. Tiesin
y = kx + b arba ax + by = c
x2+ 3 2
Nubrkite funkcijos y = x +V Q rafik2 L-

Sprendimas. y y
\
s k < 0
D(y) = (- oo; - 1) U (- 1; +) Kai * - 1, tai ^ k >o ^ X
0 k = 0
f * 0 \ z
2 + 3x + 2 _ ( -h 1)(x + 2) _
X+1 X+1
+
AZ..
o
A x
b

Vadinasi, funkcijos y = ^ * 3 * / 2 grafikas sutampa su


funkcijos y = + 2 grafiku, kai - 1. 4. Kvadratin
y y = ax2 + bx + c
/
i/ > 0

\ jJ
D > 0

y = X2 0 X Oi X
A B
D = o

5. Kubin
22. Elementariosios funkcijos, y = ax3 + bx2 + cx + d.
j formuls ir grafikai

x

/6 y = X3

I
6. = Vx 7 (n e N). Funkcija y = kx + b, k O, b O nra nei lygin, nei
nelygin.
2 - 2*+1/
y = V-t = VX
Funkcija y = kx - tiesioginis proporcingumas (k > 0)
Nelygin funkcija.
y=M
k, = tga,
k2 = tgp
7. = Xn ( n e Z)
y=M
y\
Kvadratin funkcija y = ax2 + bx + c (a 0)

D(y) = R

, -<2* + I) Kai a > O maja intervale (- x0) ir didja intervale (xQ; +o).
V = X i =

X0 = - - minimumo takas.
Tiesin funkcija y = kx + b
y0 = f(x0) minimumas
D(y) = R
Funkcijos grafikas kerta Ox a take (- ; 0) ir Oy a take
() = [y0; +-)
(0; b), jei k p 0.
Funkcijos grafikas - parabol.
Jei k > O, funkcija yra didjanti, jei k < O, funkcija yra
Parabols virns koordinats: X0 = - ; y0 = y(x0).
majanti visoje skaii tiesje.
Simetrijos ais = x0.
Koeficientas k nusako kamp, kur sudaro ties su
teigiamja Ox aimi, todl k vadinamas krypties koeficientu.
Kai a < O didja intervale (- x0) ir maja intervale (x0; +).
Jei k > O, tas kampas smailusis, jei k < O - bukasis, jei
4 maksimumo takas,
k = O, ties lygiagreti Ox aiai. O 2a
y = kx + b (k < 0)y = kx + b (k > 0) Y0 = y(x0) - maksimumas.
y = b (k = 0)
() = (- ; y j .

y = ax2 ^
D = b 2 - 4ac > D = b 2 - 4ac = D = b 2 - 4ac < O Lygi sprendimas
Dvi aknys X1 ir X2 Viena aknis X0 Nra reali akn
c = 0 b =0
Grafikas kerta Ox a G r a f i k a s lieia Ox a Grafikas yra v i e n o j e
dviejuose takuose aies Ox pusje
x(ax + b) = O ax2 + c = 0
X1 = O arba ax + b = O c
X2 = - a
a > O
1
1,2 = +V
-C a

c 1 O ir b P O jei b dalus i 2

ax2 + bx + c = O X = (y)2-ac
D = b 2 - 4ac
diskriminantas X
1,2 a
- b + Vg
X
1,2 2a

ax2 = O viena aknis = 0.

akn egzistavimas

1. D > O - dvi realios aknys.


2. D = O - dvi realios lygios aknys.
3. D < O - reali akn nra.

23. Kvadratins lygtys


Vijeto teorema
2
ax + bx + c = O (a 0) nesuprastinta,
a -1 koeficientas; b - Il koeficientas, c - laisvasis narys. 2 + px + q = O

Tipai: pilnoji ax2 + bx + c = O arba x2 + px + q = O X1 ir x2 aknys X1 + x2 = - p arba - |-


nesuprastinta suprastinta X11- ,2 = q^ arba
a

nepilnosios ax2 + bx = O Kvadratinis trinario skaidymas dauginamaisiais


ax2 + c = 0
ax2 + bx + c = a(x - x j ( x - x2)
X1 ir x2 - aknys.
Pvz.: Sprendimas.
Sakykime, kad kvadrat kratins yra 5t ir 4t. Sumaint
1) Nesprsdami lygties X2 + 13x + 45 = O, raskite jos kvadrat kratins 5t - 2 ir 4t - 2.
Pagal udavinio slyg:
akn kvadrat sum.
(5t - 2)2 - (4t - 2)2 = 28
Sprendimas. 9t2 - 4t - 28 = 0
4 32
^ = g- < 0 (netinka pagal udavinio prasm);
Pagal Vijeto teorem
t = 132 = 2
X1 + X2 = - 13, 2 18
X1X2 = 45.
X12 + X2 = X12 + 2X 1 X 2 + X22- 2X,X 2 = (X1 + X2)2 - 2 x / 2 =
Vadinasi, kvadrat kratins lygios 10 cm ir 8 cm.
= (- 1 3 ) 2 - 2 4 5 = 169 - 90 = 79.
Ats.: 10 cm ir 8 cm.
Ats.: 79.
Udaviniai.
2
2) Raskite visas a reikmes, su kuriomis lygtis ax + 13x +
+ 1 = 0 turi dvi skirtingas aknis. 1. Lygties x 2 + px + q = 0 koeficient p ir q skirtumas
lygus 5, o jos diskriminantas 16. Raskite X21 + x2.
Sprendimas.
Kvadratinis trinaris turi dvi skirtingas aknis, jei jo Ats.: 10.
diskriminantas teigiamas.
D = 169 - 4a > 0 2. Raskite visas b reikmes, su kuriomis lygtis
2 + b(3x + b) + 5b + 6,25 = 0 neturi realij akn.

Ats.: (-1; 5).


Kai a = 0, tai duotoji lygtis tampa 13x + 1 = O ir turi tik
vien sprendin. 3. Kokios turi buti p reikms, kad lygties
Vadinasi, a t 0. x+15 _ 10x-p 1 - ,
Tada a e (- 0) U (0; ) . p saknys X1 ir X2 tenkint nelygyb
+
Ats.: (-oc; 0 ) U ( 0 ; f ) . X1-X2 /> 1?
1

3) Dviej kvadrat kratins proporcingos 5 ir 4. Jeigu Ats.: [2; 8].


kiekvieno kvadrato kratin sumainsime 2 cm, tai gautj kvadrat
plot skirtumas bus lygus 28 cm 2 . Raskite duotj kvadrat 4. Su kuria m reikme viena lygties mx2 + (m - 23)x + 16
aknis lygi 2?
kratines.

Ats.: 5.
24. Iracionaliosios lygtys X 2 - 5x = 0,
1x5-2.
Lygtis Vf(x) = g(x) Lygtis Vf(x) = Vg(
ekvivalenti sistemai: ekvivalenti sistemai: J x = O arba = 5;
1x^2.
r f ( x ) =g 2 (x); r f(x) = g(x);
I g ( X ) 5 0. l f ( x ) > 0 (arba g(x) > 0)
I ia = 5.
Udaviniai.
4 f s . ; = 5.
1. Isprskite lygt Vx + 2' = x.
3. Isprskite lygt (16 - x2)V3 - x' = 0.
Sprendimas.
Sprendimas.
Duotoji lygtis ekvivalenti sistemai: Kairje lygties pusje yra dviej funkcij sandauga.
: 2,
X + 2 = Sandauga lygi nuliui tada ir tik tada, kai nors vienas i daugikli
lygus nuliui, o kiti su ia slyga turi prasm.
I x ^0;
Duotoji lygtis ekvivalenti trij slyg visumai:
= - 1; 1 6 - X 2 = O,
X2 - - 2 = 0 <=>
= 2. 3 - ^ 0,
3 - = 0.
aknis = - 1 netenkina slygos >.0.
Todl lygties sprendinys = 2.
Ats.: - 4; 3.
Ats.: 2.
4. Nesprendiant lygties parodyti, kad lygtis
Vx + 1'+ V3 - ' = 17 neturi akn.
2. Isprskite lygt V3x + 1' - Vx + 4 = 1.

Sprendimas.
Sprendimas.
Rasime lygties apibrimo srit:
f + U 0, f x 5-- 1,
V3x + 1' = 1 + Vx + 4 ' .
I 3 - x > 0; I x ^ 3;
Pakeliame abi lygties puses kvadratu ir gauname:
- x^< 3.
3x + 1 = 1 + 2Vx + 4 + + 4.
Tada 2Vx + 4' = 2x - 4
Nustatome, kokias reikmes gauna Vx + 1' ir V3 - :
Vx + 4 = - 2.
Gautoji lygtis ekvivalenti sistemai: 0 ^ Vx + 1 4 2 , o 0 V3 - ' -< 2.
2
2
f + 4 == (x - 2) ; r x ++ 4 = - 4x + 4;
I x ^ 2. Vadinasi, Vx + 1 + V3 - x ' < 17, t.y. lygtis neturi akn.
L X - 2 ^ 0.
Udaviniai. 5. a > b > 0 => -- <T
Isprskite lygtis: 6. a > b > 0 => an > b n

7. a > b a > b
+
c > d c > d
/Ais.: 1,8. a + c > b + d a -d > b- c

2. V ( + 2)(x - 3) = 2 ( - 4). 8. a > b a > b > 0


c > d > 0 c > d > 0
Ats.: 7. ac > bd A >JL

3. V3 + V x ^ T 7 = V17 - ' .
Tiesini nelygybi sprendimas
.: 11.
> a < a

h >X i >
Ats.:-1,2. a a
[a; + (- a)

a ^ b
25. Nelygybs
->x
a > b, jei a - b > 0; b
a < b, jei a - b < 0; [a; b] (a; b)
a = b, jei a - b = 0.
ax > b

Savybs > y , kai a > 0

1. a > b => b < a < , kai a < 0


2. a > b irb>c=>a>c
3. a > b ir c - bet kuris skaiius => a + c > b + - bet kuris realusis skaiius, kai a = 0 ir b < 0.
4. a > b ir c > 0 => ac > bc
c < 0 => ac < bc Sprendini nra, kai a = 0 ir b > 0
Pvz.: (2) sistemos sprendinys - intervalas (a; b)

Isprskite nelygybes:
1 1 >x
a b
1.-1 + 2 ^ O
e (a; b)
-1>-2
(3) sistemos sprendinys - intervalas (- a)
<6

Ats.: e (- 6].

2. 1,5(x - 8) - 2(0,75 + 5) < 0 x e (- oo; a)


1,5x- 12 - 1,5x - 10 < 0
0- - 22 < 0 (4) sistema sprendini neturi

0 < 22, nelygyb teisinga su bet kuriuo realiuoju


1 t >x
a b
Ats.: xe (- +

Nelygybi sistema
Pvz.:

{ ; : > {;: < 2 > { ; : > Isprskite nelygybi sistemas:

Sakykime, kad a < b 1. f 3x + 2 > - 1 J 2x > - 3 ^


L 0,4x - 16 < O L 0,4x < 16
(1) sistemos sprendinys - intervalas (b; + )
^ >-1,5
\ <40
1 1
a b
Koordinai tiesje pavaizduojame abiej nelygybi
x e (b; + oo) sprendinius. o

t 1 >X
-1,5 40

Ats.: e (-1,5; 40).


S 3,5x - 2 > 5,3x - 5 f - 11, ,88xx > - 3 Isprskite nelygybi sistemas:
2x - 2 < 3x - 4 1 <=> I- < - 2 ^ 2
1. 7x + 8 < 6 + 5x,
I 7x - 5 > 7 - 5x.
X <
i '
Ats.: 0.
3 - 7x , + 1 a 7 - 3x
f 10 2 5 '
A 22 i
3
^ 7(3x - 6) + 4(17 - ) > 11 - 5(x - 3).
4fs.; Sprendini nra.
i4fs.: (0; + oo).
3. | | x + 3^-1 + |x , 0 r - x ^ < -4
I 4(x - 0,75) < 5(x + 1) 1 - < 88
26. Kvadratins nelygybs
rx^16,
ax2 + bx + c > 0; ax2 + bx + c < 0, a >0.
I >-8;
-8 16
S p r e n d i a n t k v a d r a t i n e s n e l y g y b e s reikia surasti
Ats.: xe [16; + >).
diskriminant D = b 2 - 4ac ir surasti kvadratinio trinario aknis.

Udaviniai.
D < 0 D =O D > 0

\J V7
Isprskite nelygybes:

1. 2(x + 1) + 5 > 3 - (1 - 2 x ) .
x
O
\ / ..
XlWx2 )X
Ats.: ( - o; + oo).

9 x
~ ^ + "Ic 5x -3 Nelygyb ax2 + bx + c > 0
1
6 ^ 2 " 2

xeR X e (- X 0 ) u (X 0 ; + o) xe X1) u (X 2 ; +
Ats.: ; + oo).
oo; ( - oo; oo)

Nelygyb ax2 + bx + c < 0


3. .
Sprendini nra Sprendini nra X (X1JX2)

Ats.: (- oo; + oo).


Pvz.: 4. - 3x2 - + 4 < 0.

Isprskite nelygybes: Sprendimas.


Nelygybs - 3x2 - + 4 <: 0 abi puses padauginame i ( - 1 )
1. 2 > 0. ir gauname: 3x2 + - 4 > 0.
Randame parabols y = 3x2 + - 4 susikirtimo su Ox aimi
Sprendimas. takus: 3x2 + - 4 = 0
Parabol y = X2 yra aukiau D = 1 + 4 3 4 = 49.
aies Ox su bet kuriuo ? 0. X1 = I i X 2 = - I -
-^x
Pavaizduojame parabol schemikai:
Ats.: e (- 0) U (0; + <*>).

2. - X 2 > 3

Sprendimas.
Kadangi - x2 < 0 su bet kuria realija reikme, tai nelygyb
y -0, kai e ( - ] U [1; +
neturi sprendini.

Ats.: Sprendini nra. Ats.: e ( - - f ] U [1; + ).

3 *X2 << 25

5. Su kuria sveika k reikme nelygyb
'
2 - 2(4k - 1)x + 15k2 - 2k - 7 > 0
Sprendimas. teisinga su bet kuria realia reikme?

2 - Sprendimas.
nc N< 0
25
Kadangi koeficientas prie x2 teigiamas, tai, kad nelygyb
Briame schemin funkcijos y = x 2 - grafik. Randame bt teisinga su bet kuria realia reikme, turi bti D < 0, t.y.
(4k- 1)2 - (15k2 - 2k - 7) < 0
funkcijos y nulius (t.y., takus, kuriuose y = 0).
16k2 - 8k + 1 - 15k2 + 2k + 7 < 0
k2 - 6k + 8 < 0
X2-^T = 0 ; 2
~l5 ' x =
T 'r x
~ ~ If Surandame takus, kuriuose trinario k2 - 6k + 8 grafikas
kerta abscisi a: ^y
k2 - 6k + 8 = 0
k, = 2; k2 = 4
2 < k < 4
Vadinasi, k = 3.
1 1
Vadinasi, e [ - - g - ' . j J- Ats.: k = 3.
6. Raskite funkcijos y = V3x2 + 6x + 12 reikmi srit.
27. Lygtys su moduliu

Sprendimas. |X| = a I - b| = a
1) a < 0 1) a < 0
Poaknis visada teigiamas, nes kvadratinio trinario Sprendini nra Sprendini nra
D < O, o a > 0. 2) a = 0, = 0 2) a = 0, - b = 0, = b.
Maiausi reikm trinaris gaus, kai 3) a > 0 f = a 3) a > 0 - b = a,
Lx = - _ - b = - a;
x =
o - W = - 1 , = 3(-1) 2 + 6 ( - 1 ) + 1 2 = 9. I ia = b+ a
= b-a
y maiausia reikm bus V9* , t.y. y = 3.
Vadinasi, E(y) = [3; +
|f(x)|=g(x)| |f(x)| = g(x)
ekvivalenti lygi visumai ekvivalenti lygi sistemai
Ats.: E(y) = [3; + >). f(x) = g(x) / T f ( X ) = Q(X)
L f(x) = - g(x) i Lf(x) = - g ( x )
L
g(x) 5-.
Udaviniai.

Isprskite nelygybes: Nelygybs su moduliu

j - b| < a |x - b| > a
1. - 9x2 - 6x - 1 < 0.
1) a < 0 1) a < 0
Sprendini nra xeR
Ats.: (- oo; - 1 ) U ( 4 ; + oo). 2) a > 0 2) a > 0
- a < - b < a - b > a arba - b < - a
2. 2x2 - 4x + 9 ^ 0. I ia I ia
b-a<x<b+a > a + b arba < b - a
Ats.: Sprendini nra.

3. 2x2 + 3x < 0. I f M l < |g(x)l |f(x)l > lg(x)l


Ekvivalenti sistemai Ekvivalenti nelygybi visumai
Ats.: (-1,5; 0). S f(x) < g(x) f(x) > g(x)
I f(x) > - g(x) - f(x) < - g(x)

Nelygyb I f(x)| > |g(x)| ekvivalenti nelygybei P(x) > g2(x)


arba nelygybei (f(x) - g(x))(f(x) + g(x)) > 0.
Pvz.: 3. Isprskite nelygyb |3x - 5| > 7x - 6.

1. Isprskite lygt I + 2 I = 6 - 2x. Sprendimas.


Duotoji nelygyb ekvivalenti nelygybi visumai:
Sprendimas. 3x - 5 > 7x - 6, 4x > - 1,
Gauname: 3x - 5 < - 7x + 6 ^ 10x < 11: * *
f + 2 = 6 - 2x, + 2 = - 6 + 2x;
arba
\ 6 - 2 x > 0; L 6 - 2x ^ 0. x
^ T Y <x 1
X<U; ^ T -
f 3x = 4, / = 8;
1x^3; 1x^3; Ats.: e

r =j ; Gautoji sistema neturi 4. Isprskite nelygyb 2| 2 - 1| > + 1 .


L x ^ : 3. sprendini.
Sprendimas.
. . 4
Is cia = -3-. Duotoji nelygyb ekvivalenti visumai:
2x2 - 2 > + 1,
Ite-T- 2x2 - 2 < - ( + 1).

2. Isprskite lygt | - 2| + | 2x - 3| = 5 . 2x2 - - 3 > 0,


2x2 + - 1 < 0.
Sprendimas.
Skaii ayje paymime takus, kuriuose po moduliu Pirmosios nelygybs sprendinys (- - 1) U ( 1 ; + >).
2
esantys reikiniai virsta nuliu: Antrosios nelygybs sprendinys e ( - 1 ; ! ) .
1 1 >
3 2 Ats.: e (- - 1 ) U ( - 1 ; ) U ( | ; + >).
2
Udaviniai.
kai $ - f , tai - (x - 2) - (2x - 3) = 5,
I ia x = 0. 1. |2x - 3| = 11.

kai - -< ^ 2, tai - (x - 2) + 2x - 3 = 5 >4fs.;-4; 7.


= 6 (netenkina slygos).
2. | 2 - x | = 5 - 4x.
kai > 2, tai - 2 + 2x - 3 = 5,
3 x > 10, Ats.: 1.

Ats.: 0; f .
Sprendimas.
3. |2x + 5 I= 2.
Nagrinjame funkcij f(x) = x(x - 4)(x + 5)2.
D(f) = R. f(x) = 0 takuose = 0; = 4 ir = - 5.
Afs.: - 7; - 1 .

4. |x - 1| +|6 - 2x| = 3.

A f s . : - i ; 2. f(- 6) > 0; f(- 1) > 0; f(1) < 0; f(5) > 0.

5. |x - 1 1 - 2|x - 2| + 3 |x - 3| = 4. A f s . : e (- oo; - 5) U (- 5; 0) U (4; + oo).

Ats.: = 5. 2. ( - 2)(x + 1)(x + 6 ) 2 ^ 0.

6. I 4x + 11 < 3. Sprendimas.
Nagrinjame funkcij f(x) = (x - 2)(x + 1)(x + 6)2.
Ats.: (-V, ) . D(f) = R.
f(x) = 0 takuose = 2; = - 1; = - 6.
7. - 1 < 2 | x | .

Afs.: (-oo; 1).


-6 -1 2

f(- 7) > 0; f(- 2) > 0; f(1) < 0; f(3) > 0.


28. Interval metodas
Afs.: e [ - 1 ; 2] ir = - 6.
Sprendiant nelygybes f(x) > O arba f(x) < O, reikia skaii x x+ 3
3 < > < nu
tiesje atidti takus, kuriuose f(x) virsta nuliu arba neturi prasms. -4 ^ -
Tie takai skaii ties padalija baigtin skaii interval, kuri
kiekviename funkcija f yra pastovaus enklo. Norint nustatyti t Sprendimas.
x(
enkl, utenka apskaiiuoti funkcijos reikm nors viename Nagrinjame funkcij f(x) = *_+43) .
pasirinkto intervalo take.
D(f) = (- oo; 4) U (4; + oo).
Pvz.: f(x) = 0 takuose = 0; = - 3.

Isprskite nelygybes:
-3 0 4
2
1. x(x - 4)(x + 5) > 0. f(- 4) < 0; f(- 2) > 0; f(3) < 0; f(5) > 0.

Afs.: e (- - 3] U [0; 4).


Udaviniai. Progresijos

Isprskite nelygybes: Aritmetin Geometrin


:
K) :: (bn)
1 2x + 1 > 2
2 +
Apibrimas
Ats.: (-*>;-2).
a
3x2 - 2x - 5 . n +1 = a n + d
bn + 1 = b n . q ( q = 0, 1).
Z. 2 ^
n-tojo nario formul
Afs.: ( - 1 ; f ) U (2; +
a = a +
121 n i (n-1)d bn = b1 'q n 1

3. Raskite didiausi sprendin - ^ r j ^ 20 - x. Sumos formul

Afs.: 9. S n = ^ - n Sn = A ^ i l

2a +
arba S n = - ?-1)d.n arba S =
29. Skaii seka n q -

Skaii seka yra natraliojo argumento funkcija: a n = f(n). Savybs


Sekas apibriame nurodydami jos narius: av a2, a 3 ,..., a n ,...
d = a - an = n = ^ i i -IlK
arba pateikdami n-tojo nario iraik an. 1 q

a + a
- n m
Riba

Skaiius C yra skaii sekos (a n ) riba, kai n jei


kiekvienam teigiamam egzistuoja toks natralusis skaiius n0, kad
aritmetinis vidurkis geometrinis vidurkis
Ian - C| < , visiems n, n ^ n0.

Iim (a + b ) = Iim a + Iim b kai I q I < 1, tai geometrin


progresija be galo majanti
Iim (a b ) = Iim a Iim b
b
->~ n
" n
n->= n S= <
1 - q
ma
Iimooi afn = '' n
on . kai Iim b n 0
4. Periodin trupmen 0,58333 ... ireikkite paprastja.
Pvz.:
Sprendimas.
1. Duoti du skaiiai - 13 ir 37. Kok skaii tarp j reikia 0,58333 ... - "ioQ- + -^qqo + Ioooo+ +
rayti, kad trys skaiiai sudaryt aritmetin progresij?
Deinje lygybs pusje dmenys, pradedant skaiiumi
, sudaro begalin majani geometrin progresij, kurios
Sprendimas.
_ _ - 1 3 + 37 _ 1 9
pirmasis narys lygus , o vardiklis .
2 2 3
-1- + 3 + 3 . _ 1000 _ __
1000 10000 100000 1 ZT 300 '
Trys skaiiai - 1 3 ; 12; 37 sudaro aritmetin progresij. 1
" 10

0,58333 ... = + ^ = - .
Afs.; 12.
2. Raskite aritmetin progresij, kurios ketvirtasis narys Ats
M i b . . 1 2 .
keturiais didesnis u dvigub pirmj nar, o pirmj penki nari
Udaviniai su sprendimais.
suma lygi 10.
1. Geometrins progresijos pirmasis narys lygus ViT , o
Sprendimas.
septintasis - V128'. Raskite atuntj progresijos nar.
a4 = 2a1 + 4,
Sprendimas.
\ S5 = 10;
Jei (bn) - geometrin progresija, q - progresijos vardiklis,
b - n-tasis progresijos narys, tai b, = V2j^b 7 = ^ ^ 1 2 8 = V2 *q6,
r a, + 3d = 2a 1 + 4, 2
qe = CT q e = 8 j ( q 2 ) 3 = 2 3, q = 2, q =V2 ; q = - V2 .
5 = 10;
f a : = 3d - 4,
1) q = V2]^b 8 = VT2T-V2T, b 8 = V25T, b8 = 16.
I 2a, + 4d = 4;
2) q = - V2 , b8 = - V2561, b8 = - 16.

Ats.: 16; - 1 6 .
Afs.: a, = - ; d = y .
2. Staiojo trikampio kratini ilgiai sudaro aritmetin
3. Kok skaii reikia rayti tarp skaii 7 ir 175, kad trys progresij. Raskite trikampio maesniojo kampo sinus.
skaiiai sudaryt geometrin progresij?
Sprendimas.
Sakykime, trikampio kratins a, a + d, a + 2d. Pagal
Sprendimas.
Pitagoro teorem a 2 + (a + d)2 = (a + 2d) 2 ; a2 - 2ad - 3d2 = 0,
b 2 = V7 -175 = /7 7 25 = 7 5 = 35.
a = - d arba a = 3d.
Kadangi a > 0; d > 0, tai a = 3d.
Trys skaiiai 7; 35; 175 sudaro geometrin progresij. =
Maesniojo kampo sinusas lygus I f = T-
A f s . : -O
f .
Afs.; 35.
3. Pro ang vamzd pateko viena materiali dalel, o po 6,8 37- pirminis skaiius, n arba 1 + n dalosi i 37 ir dalmuo
minui pro t pai ang pateko dar viena dalel. Kiekviena dalel lygus 1, nes tik tuo atveju lik dauginamieji nevirija 10.
i karto pradeda judti kito vamzdio galo link. Pirmoji dalel juda Akivaizdu, kad n = 36.
tolygiai 5 ^ greiiu, o antroji pirmj minut nueina 3 m keli, o
36.37 _ 6
kiekvien kit minut 0,5 m daugiau negu prie tai buvusi. Po A
2-3-37
kiek minui antroji dalel pavys pirmj?
Ats.: n = 36.
Sprendimas.
Tegul t - laikas minutmis, per kur antroji dalel paveja 5. Punktai A, B, C ir D isidst ant vienos tiess nurodyta
pirmj. tvarka. Pstysis eina i A D 5-- greiiu. Pasieks D, jis pasuka
Kelias, kur nueina antroji dalel, yra lygus t nari aritmetins atgal ir ateina punktB, visam keliui sugais 5 valandas. inoma,
progresijos, kurios a1 = 3, d = 0,5, sumai. kad atstum AC jis nueina per 3 vai., o atstumai tarp A ir B, B ir C,
Vadinasi, C ir D (duotja tvarka) sudaro geometrin progresij. Raskite
o _ 2a, + d (t - 1) _ 6 + 0,5(t - 1) t atstum tarp B ir C.

j I - 2
Sprendimas.
Sakykime, kad AB = km, BC = y km, CD = z km.
Tok pat atstum nueis ir pirmoji dalel:
Pstysis atstum AB nueina 1 kart BC - 2 kartus, CD - taip
pat 2 kartus.
5 = 5(6,8 + t) = 34 + 5t.
Visas veiktas atstumas lygus 5 ^ - 5 vai. = 25 km.
6 + 0 Tada + 2y + 2z = 25.
f-1>. t = 34 + 5t.
Antrj lygt gauname, inodami, kad atstum AC nueina
per 3 vai., t.y. x + y = 3 - 5 = 15.
t = 17 (min.).
Treij lygt gauname pagal geometrins progresijos
apibrim y2 = xz.
Ats.: 17 minui.
+ 2y + 2z = 25 r 1 5 - y + 2y + 2z = 25
4. Tam tikro skaiiaus i eils einani natralij skaii
(pradedant 1) suma lygi trienkliam skaiiui, kurio skaitmenys
vienodi. Kiek paimta skaii?
{
+ y = 15 => - S X = 15 - y
Sprendimas. y 2 = 1xz ^ y 2 = (15 - y) -z
7 - -
Tegul buvo n natralij skaii.
2 y 2~ = 12 5 0 - 1 5 y - IOy + y2
Trienklis skaiius - xxx, t.y. 100x + 10x + = 111x.
y2 + 25y - 150 = 0
1, 2, 3, ... - aritmetin progresija.
D = 625 + 600 = 1225
V D = 35
I i n n =111x, (1 + n) = 222x, = - ^ .
1
1 = = 5; 2 = ^ = - 30 (netinka).
<10.
Ats.: 5 km.
Udaviniai. Tamprireiks = ^-(vai.).
Ats.: 10 min.
1. Ketvirtasis aritmetins progresijos narys lygus 16, o
septintojo ir deimtojo n a r i s u m a lygi 5. Raskite pirmj 2. Maylis gali suvalgyti tort per 10 min., stiklain uogiens -
atuoniolikos nari sum. per 13 min. ir igerti puod pieno per 14 min. Karlsonas vis tai
gali padaryti atitinkamai per 6 min., 6 min. ir 7 min. Per kok
trumpiausi laik jie abu gali visk suvalgyti?
Ats.:- 9.

2. Raskite sum vis trienkli skaii, kuriuos dalinant i Sprendimas.


Jei Maylis ir Karlsonas nori per trumpiausi laik visk
11, gauname liekan 9.
suvalgyti, tai jie turi pradti ir baigti kartu.
Ats.: 45387. Paymkime:
Maylis suvalgo dal torto, y dal uogiens ir z dal pieno.
3. Tarp skaii 7 ir 56 raykite du skaiius, kurie su TuometKarlsonassuvaIgys 1 - dal torto, 1 - y dal uogiens
duotaisiais sudaryt geometrin progresij. ir 1 - z dal pieno.
Laikas t, kur valgymui sugaio Maylis ir Karlsonas, yra
vienodas.
Ats.: 14; 28.
t(M) = 10x + 13y + 14z.
4. Turime aritmetines progresijas 2; 7; 12; ...ir 3; 10; 17;... t(K) = 6(1 - ) + 6(1 - y) + 7(1 - z).
Raskite pirmj 50 abiej progresij sutampani nari sum. 0 < 1, O ^ y ^ 1, O ^ z ^ 1.
10x + 13y + 14z = 6(1 - ) + 6 (1 - y) + 7(1 - z)
Ats.: 43725. 10x + 13y + 14z = 6 - 6x + 6 - 6y + 7 - 7z
z = 2 ^ 1 9 - 16x - 19y)
t = 10x + 13y + 14z = 10x + 13y + J-(19 - 16x -
30. Tekstini udavini sprendimas -19y) = = 10x - f + 13y - f y + f = - f + y + f .

1. Snieg valo 2 sniego valymo mainos. Pirmoji gali nuvalyti I ios iraikos aiku, kad t bus tuo maesnis, kuo didesnis
gatv per 1 vai., o antroji - per 75% to laiko. Gatv valyti pradjo yra , ir kuo maesnis y.
abi mainos ir kartu dirbo 20 min., o po to dirbo tik antroji maina. Paimsime pai didiausi galim reikm, t.y. = 1 ir
Kiek laiko dar dirbs antroji maina, kol baigs valyti vis gatv? pai maiausi y, t.y. y = 0.

Sprendimas. Tuomett = - J - I + y + =y = 12 (min.), o z = f


Atliekam darb paymkime 1. gauname i galimo intervalo.
Pirmos mainos darbo naumas per 1 vai. - 1, o antros
mainos 1 : -f- =-f- (per vai.). Ivada: t gyja maiausi reikm, kai Maylis suvalgo vis
Abi mainos dirbo | vai. Jos atliko 1 + y f = ^ viso tort ir igeria pieno, o Karlsonas suvalgo vis uogien ir igeria
darbo. puodo pieno.
Vadinasi, antroji maina turi atlikti viso darbo.
Sprendimas.
3. auliui audykloje buvo sudarytos tokios slygos: u
Tarkime, kad visas kelias km.
kiekvien taikl v dovanojami 5 etonai, u kiekvien nepa-
I kelio dalis - 0,5x + 192,5 km
taikym - atimami 3 etonai. aulio bta nelabai taiklaus. Po
II kelio dalis - 0,5x - 1 9 2 , 5 km
paskutiniojo (n - tojo) vio jis neturjo n vieno etono. Keli viai
Sudarome lygt:
sudar serij ir keli i j buvo taikls, jei 10 < n < 20?
0,5x + 192,5 + 0,5x-192,5 _ __
Sprendimas. 220 330 250
Tegul buvo pataikyta m vi, o n - m praauti. 1,5x + 577,5 + - 385 _
Tuomet 5m - 3(n - m) = 0, 66 25
8m = 3n, 2,5x + 192,5
3n
66 25
m = T -
m, n - natralieji skaiiai, tai n turi buti 8 daliklis. 62,5x + 4812,5 = 66x
Intervale 10 < n < 20 tra tik 16 dalus i 8. 3,5x = 4812,5
Tai auta 16 kart, o pataikyta 6 viai. = 1375.

A f s . ; 16 ir 6.
Ats.: 1375 km.

4. Pirmj ketvirtadal kelio traukinys vaiavo 80 ^ j r greiiu,


6. Parduotuv gavo I ir Il ri preki u 4,5 milijono Lt.
o likusi dal - 60-^-greiiu. Kokiu vidutiniu greiiu vaiavo trauki- Visas prekes pardavus pagal Il ries preki kain bt 0,5 milijono
nys? Lt nuostolis, o visas prekes pardavus pagal I ries preki kain
bt 0,3 milijono Lt pelnas. U kokias sumas parduotuv gavo I ir
Sprendimas. Il ri prekes?
Tarkime, kad visas kelias s km.
Pirmoji kelio dalis buvo nuvaiuota per vai., o likusioji
Sprendimas.
kelio dalis - per -- = vai.
S a k y k i m e , p a r d u o t u v gavo v i e n e t I ries po a
tkstani Lt u vienet ir y vienet Il ries po b tkstani Lt u
Taigi vis keli traukinys veiks per =^-vai. vienet.
5s 320 _ . km Tada u I ries prekes parduotuv sumokjo ax tkstani
Vidutinis greitis bus: s : = -5- = 64
Lt, o u Il ries prekes - by tkstani Lt.
km Vadinasi, ax + by = 4500.
Ats.: 64~ .
Jei visas prekes parduot pagal I ries preki kain tai
pagal slyg a(x + y) = 4800 tkstani lit.
5. Lktuvas i pradi skrido 2 2 0 ^ - g r e i i u . Kai jam liko
Jei visas prekes parduotuv parduot pagal Il ries preki
skristi 385 km maiau, nei jis buvo nuskrids, jo greitis tapo 330 ^ r . kain, tai pagal slyg b(x + y) = 4000 tkstani Lt.
Vidutinis greitis visoje kelio atkarpoje buvo 250 ^ l . Kok atstum
Gauname sistem:
veik lktuvas?
ax + by = 4500, 31. Rodiklin ir logaritmin funkcija
{ bx + by = 4000,
ax + ay = 4800. Rodiklin funkcija Logaritmin funkcija

I pirmosios lygties atimame antrj, o i treiosios atimame y = a x (a > 0; a 1) y = Iog a X (a > 0; a / 1 )


pirmj ir gauname: f (a - b)x = 500,
D(y) = R D(y) = (0; + oo)
I (a - b)y = 300.
E(y) = (0; + ) E(y) = R

Gautosios sistemos pirmj lygt padalijame i antrosios ir Vienas monotonikumo Vienas monotonikumo
gauname: intervalas. intervalas.
7 = 1 =3 V=I x
-
Ekstremum nra Ekstremum nra
I lygties ax + ay = 4800 turime:
ax + - | a x = 4800 => J-ax = 4800 => ax = 3000 Kai a > 1, funkcija didja Kai a > 1, funkcija didja
by = 4500 - 3000 = 1500. visoje reali skaii aibje visoje apibrimo srityje D(y)

Ats.: 3000000 Lt ir 1500000 Lt.

->
Udaviniai.

1. Priekinis veimo ratas tam tikrame kelyje apsisuko 1000


kart daugiau negu upakalinis ratas. Jei priekinio rato ratlankis
bt 1,5 karto ilgesnis, tai tame paiame kelyje jis apsisukt 200 Kai 0 < a < 1, f u n k c i j a Kai 0 < a < 1, f u n k c i j a
maja visoje reali skaii aibje maja visoje apibrimo srityje
apskritim daugiau negu upakalinis ratas. inoma, kad upakalinio
rato ratlankis yra 1,5 m ilgesnis u priekinio rato ratlank. Raskite D(y)
ratlanki ilgius.

Ats.: Priekinio ratlankio ilgis 2,1 m, o upakalinio - 3,6 m.

2. Pirmame tirpale yra 6% mediagos A, 16% mediagos B


ir 4% mediagos C (pagal svor), antrame tirpale atitinkamai - 1 5 % ,
9%, 10%, o treiame - 3%, 5%, 2%. Kiek reikia paimti kiekvieno i
t tirpal, kad gautume 1 kg tirpalo, kuriame bt 12% mediagos
A, 10% mediagos B ir 8% mediagos C ?

Ats.: Pirmojojo tirpalo - 0,2 kg; antrojo tirpalo 0,7 kg, treiojo
Logaritmin ir rodiklin funkcijos su tuo paiu pagrindu yra tarpusavyje
tirpalo - 0,1 kg.
atvirktins, o j grafikai simetriki tiess y = atvilgiu.
Logaritmai Papildomi sryiai

l0
L o g a r i t m a s t e i g i a m u , nelygiu v i e n e t u i pagrindu b t e i g i a m o Qab = i s r
skaiiaus a, yra laipsnio rodiklis c, kuriuo reikia pakelti pagrind b, kad
gautume skaii a. logamb = -j- log a b

log b a = c (a > 0; b > 0; b tada ir tik tada, kai b c = a. l9 a mb" = log a b


l0 b l0
Sm _ 9 _ | 0 .
Pagrindin logaritmin tapatyb: b'%a = a IogmC log^ 9CD

Pvz.: log n b IogmC = log m b IognC


310936 = 6

25'952 = (52)10%2 = (5'0%2)2 = 2 2 = 4 Pavyzdiai.


Iog 2 16 = 4, nes 2 4 = 16 1
2Io932 _ 210^3 = 3
log 4 8 = 1,5, nes 415 = 8
log _16 = - 2, nes 0,25 2 = 16
Iog9V3 = 0,25, nes 9025 = V" Vlog 5 64-log 8 25 = Vlog 8 64 log s 25' = = 2

Logaritmas pagrindu 10 vadinamas deimtainiu ir ymimas Udavinys.


Ig : log 10 a = Iga.
Raskite log 30 8, jei Ig5 = a ir Ig3 = c.
Pvz.:
IgIOOO = 3 Sprendimas.
Ig0,001 = - 3
3 < Ig3154 < 4 Iog 30 8 = log 30 2 3 = 3log 30 2 = | ;
- 1 < IgO, 12 < 0
Ig2 = l g f - = l g 1 0 - l g 5 = 1 - a
Logaritmas pagrindu e vadinamas natriniu ir ymimas
In: log e a = Ina Ig30 = lg(2 15) = Ig2 + Ig15 = Ig2 + ig(3- 5)
e = 2,7 = Ig2 + Ig3 + Ig5 = 1 - a + c + a = 1 + c.

Logaritm savybs Vadinasi, log 30 8 = ^ .

Iog a I = 0 Ats 3(1 " a)


MiS).. 1 + c
I09ai = - 1
log a a = 1

Sandaugos, dalmens ir laipsnio logaritmai

log c (ab) = log c a + log c b


l o g c | = log c a - log c b
log c a k = klog c a
Logaritm palyginimas = log 2 4 7 + (2log 14 7 + log 14 2) log 14 2 = Iog24 7 +

+ 2log 1 4 7 log 14 2 + Iog24 2= ( l o g u 7 + log 14 2) 2 =


Jei O < a < 1 ir O < , < x 2 , tai Iog a X 1 > Iog a X 2
(Nelygybs enklas keiiasi)
= log 2 14 (2.7) = Iog 2 4 M =1.

Jei a > 1 ir O < X1 < x 2 , tai Iog a X 1 < Iog a X 2


Ats.: 1.
(Nelygybs enklas nesikeiia)

2. rodykite, kad Iog 17 19 > log 19 20.


Jei 1 < a< b ir > 1, tai IogaX > log b x.
rodymas.
Jei O < a < b < 1 ir > 1, tai IogaX < log b x
Reikia rodyti, kad - - < 1.
Jei 1 < a < b ir O < < 1, tai IogaX < IogbX

I ia log 19 20 Iog 19 17 < 1.


Jei O < a < b < 1, ir O < < 1, tai IogaX < IogbX

Kadangi (Va - VH ) 2 > O, tai a + b - 2VaF > 0.


Jei b > 1 ir a > 1, tai log a b > O
Jei b > 1; O < a < 1, tai log b a < O
I ia ^ > V i F 1 t.y.
Jei O< b < 1; a > 1, tai log b a < O
Jei O< b < 1 ir O < a < 1, tai log b a > O
Vadinasi, log 19 20-Iog 19 17 < ( ' o g " 2 \ log 17
" f

Pavyzdiai.
logl920.|og1917<(^)2<^=1

Og 0 s 0 2 < I09O,5.1 3. Kuris i dviej skaii - I i o g 1 ( ^ i ) + 2 a r t g - 2 ^ d i d e s n i s ?


l0
Qos 6 > l0 9o,9 6
log s 2 < log s 4 Sprendimas.
log 3 6 < log 2 6 < log 2 7
log 5 6 < log 8 6 enklas V yra palyginimo enklas (>; <).

Udaviniai
Tada-1Iog 7 +2 V tg(13^-n)

1. Raskite reikinio Iog24 7 + reikm. l o g 7 ( 2 f - j " + 2 V tg(13n + -)

Sprendimas. log7A V tgff


+ 2

Iog24 7 + = Iog2 7 + log1498- log 14 2 = Iogf4 7 log 7 6 - log 7 49 + 2 V tg -

+ log 14 (14 -7) log 14 2 = Iog24 7 + log 1 4 (7 2 -2)- log 14 2 log 7 6 V tg


log 7 6 < log 7 7 = 1 = tg = tg g - < tg-ff-
= log2147 + (log 14 7 2 + log 1 4 2)-log l 4 2 =
A b i e j lygties p u s i I o g a r i t m a v i m a s
Vadinasi, tg - > \9,- * 2.
6 2 = 12
Afs.: t g ^ ^lofiL^P- +2.
Logaritmuojama pagrindu 2:

32. Rodiklini lygi sprendimo metodai "7 log 2 6 + = Iog212

P e r j i m a s prie b e n d r o p a g r i n d o log 2 (2 3) + = log 2 (4 3)

2* 0,5 = 8 2 ; Y (log 2 2 + log 2 3) + = log 2 4 + log 2 3


3x 1
2* 2" 1 = 2^" 2? (1 + log 2 3) + = 2 + log 2 3
_ 3x + 1
2 x-1 = 2-2

X - I = ^ i f l = = - 3 1 + log 2 3 + 2 = (2 + log 2 3)x


2 - (2 + log 2 3)x + (1 + log 2 3) = O
af(x) = 1, a > O, a t 1 <=> f(x) = o = 1; = 1 + log 2 3.

g x 2 - 4x + 3 = 1 H o m o g e n i n s rodiklins lygtys
g x 2 - 4x + 3 = 50

X2- 4x + 3 = O 3 . 1 6 - 12 = 49
= 1; = 3 Dalijame i 9 > 0:

x
I k l i m a s u s k l i a u s t -'-f-) = 4

7* + 7 + 2 = 350 3-(f)2*-(f)x-4 = 0
7 + 7x .72 = 350

7X(1 + 7 2 ) = 350 ( ) " = > => 2 - - 4 = O => = ; = -1<0=>

=*( = = 1.

Pvz.:

Kintamojo pakeitimas 1. Isprskite lygt ({- ) + 2 3 " = 9.

4x + 2 + 1 - 24 = 0
Sprendimas.
4X + 2 2 - 2 4 = 0
2"2 + 2 3 2 - 9 = 0
2 = > 0 Paymime 2" = t (t > 0), tada t2 + 8t - 9 = O, t, = - 9 (netenkina
2 + 2 - 24 = 0 slygos t > 0), X2 = 1. Vadinasi, 2 x = 1, 2 x = 2, - = 0; = 0.
= 4; = - 6 < 0
2 = 4 => = 2.
Ats.: x = 0.
2. Apskaiiuokite, kokia radioaktyvios mediagos dalis liks po 1000 2 Ox2 - 6x + 0,5 = 1
met, jeigu jos skilimo pusperiodis lygus 1500 met.
16V2-

Ats.: 1; 5.
Sprendimas. 1
Radioaktyvusis skilimas apraomas lygtimi m = m 0 ( ) ,
3. 3 (0,(3)) -3 = ( ^ ) .
kur m - mediagos mas laiko momentu, t, m 0 - mediagos mas
pradiniu laiko momentu t = 0, T - skilimo pusperiodis lygus 1500 m e t
4is.;-1.
1000
1500
Tada m = m 0 ( -y)
4. 2 " 2 = 3 X " 2 .
2
-1 = (1\
m \ 21
2> Ats.: 2.
3 r

-=W 2x + 4
o
' M ' 5 6 = 33x 2 + 8
3
m V?
" 4 ~ 2 Ats.: 4.
3
^

Ats.:
e. 3X + 1 + 18 -3 - x = 29.

3. Radioaktyvioji mediaga netenka puss savo mass kas 20


Ais.; I l - 1 ; 2 .
dien. Po kiek dien i 32 gram radioaktyvios mediagos liks 1 gramas?
7. 3 4V? - 4
+ 3 = 0.
Sprendimas.
1 \20
1 = 32 . ( - i )
iAis.: 0; 0,25.
1
32 =(t)20. 8. 22x + 1 - 5 6X + 32x + 1 = 0.
5 2n
(y)
2 ' = (T
2 )'
v
Ais.;-1;0.
t
20 = 5,
33. Logaritmini lygi tipai
t = 100.

1) IogaX = b, a > 0, a 4 1 = ab.


Ais.: 100 dien.

2) log a (f(x)) = b, a > 0, a 4 1 => f(x) =ab.


Udaviniai.

3) log a (f(x)) = g(x), a > 0, a 1 => f(x) = a ^ .


Isprskite lygtis:

4) log a (f(x)) = log a (g(x)), a > 0, a 1 =>


1. 52 " f x = ^25 7 .

A i s . ; - f ; 1.
r f(x) = g(x) Lygtis, kurios kintamasis yra logaritmo pagrindas
< f(x) > O
t g(x) > O 3 = 8,
log x 8 = 3 < x*1, <=> x= 3 V8
l x > 0 ;
Logaritmini lygi s p r e n d i m o metodai
Ats.: 2.
4 - t o j o t i p o lygtis

log3(3 - ) + log3(4 - x) = 1 + 2log32


log3((3 - x)(4 - )) = log33 + log322
{ x 2

> 0,
P 1,

<=> I
r X >

X ^ 1,
0,

<=>
l o g 3 ( 3 - x ) ( 4 - x ) = log312
(2)05 = x. L m =* X- 1
(3 - x)(4 - x) = 12,
< 3 - > O, > o,
v. 4 - > 0 ; .

{
X2- 7x = 0, Ats.: s (0; 1)U(1;o).
< 3,
< 4; Pv.:
'9 = 100x

{ x = 0 arba = 7,
< 3;
Lygties apibrimo sritis (0; + oo).
Igxl9x = lg(100x),
Igxlgx = Ig100 + Igx,
Vadinasi, = 0. lg2x = 2 + Igx,
Ig2X - Igx - 2 = 0,
Kintamojo pakeitimas Igx = a,
a2 - a - 2 = 0,
I g 2 ( ^ L ) + Igx = 7, a = - 1 ir a = 2,
Igx = - 1, Igx = 2,
(Ig 10 - Igx)2 + Igx = 7, = 0,1; = 100.
Igx = y,
Ats.: 0,1; 100.
(1 - y)2 + y = 7 => 1 - 2y + y2 + y = 7 => y2 - y - 6
y = 3, Igx = 3,
y = -2. Igx = - 2,
Udaviniai.
Ats.: = 1000; = 0,01.
Isprskite lygtis:

1. Ig(x - 2)2 = 0.

Ats.: 1;3.
2. 0,5lg(2x - 1) + IgVx - 9' = 1. Pvz.:

Afs.: 13. 1. Isprskite nelygyb 2 4 - 8 x > 1.

3. Ig2 + lg(4x - 2 + 9) = 1 + lg(2x 2


+ 1). Sprendimas.
24-8x > 2
A f s . ; 2; 4. Kadangi 2 > 1, tai duotoji nelygyb ekvivalenti nelygybei
4 - 8x > O
4 . l o g 5 V i T - 1,25 = Iog X 2 VIT . I ia < .

A f s . ; Vi?; 5. Ats.; (;-!).

5. Ig 2 X 3 - 10 Igx + 1 = 0. 2. Isprskite nelygyb

A f s . ; V W ; 10. Sprendimas.
2+ 1
6. x'^2x * 2 = 256.

Kadangi J - < 1 , tai < = f > -. 2L > 0

Vf I v t "!
34. Rodiklins nelygybs
W7T) >
0 X e (- oo; - 1) U (1; oo).
x x
a < m a > m
>4is.; (- oo; - 1 ) (J (1; oo).
3. Isprskite nelygyb 52x + 1 > 5X + 4.
m-<: 0; a > 0; a t- 1 nra sprendini xeR

Sprendimas.
m > 0; a > 1 < log a m > log a m
Paymkime 5X = t > 0.,
5t2 - 1 - 4 > 0.
m > 0; 0 < a < 1 > log a m < logam
I ia t < - 0,8 arba t > 1.
> g9(x) Kadangi t > 1, tai 5X > 5. I ia > 0.
af(x) kai a > 1 ekvivalenti nelygybei f(x) > g^x)
kai 0 < a < 1 ekvivalenti nelygybei f(x) < g(x)
Ats.: (0; oo).

af(x) < m afw > m

m 0; a > 0; a t- 1 nra sprendini e D(f)

m > 0; a > 1 f(x) < log a m f(x) > logam

m > 0; 0 < a < 1 f(x) > log a m f(x) < log a m


35. Logaritmins nelygybs 2. log2(2x - 1 )log2(2x + 1 - 2) < 2.

Sprendimas.
m e R IogaX < m IogaX > m Iog a f M < m logaf(x) > m
log 2 (2 x - 1 ) - l o g 2 ( 2 - 2 x - 2 ) < 2 ,
a >1 J x o m > am Jf(X) am
< f(x) > am log 2 (2 x -1) (log 2 (2 (2X - 1))) < 2,
\ >0 \f(x) > 0
l o g 2 ( 2 x - 1 ) (log 2 2 + log2(2x - 1)) < 2,
0<a <1 > am r < am f(x)> a m Jf(x) < am log 2 (2 x -1) (1 + log2(2x - 1)) < 2,
\x >0 lf(x) > O Paymime log2(2x - 1) = y.
y(1 + y) < 2, y 2 + y - 2 < 0.
Iog 8 f M < igag(x) logh(x)f(x) < ioghWg(x) I ia - 2 < y < 1.
kai a > 1 kai 0 < a < 1 nelygyb ekvivalenti Tada - 2 < log2(2x - 1) < 1 => log22"2 < log2(2x - 1) < log22 =
nelygyb ekvivalenti nelygyb ekvivalenti sistemoms: ^ j < 2X - 1 < 2 = ^ < 2X < 3 => IogJ < log22x < log23
sistemai sistemai f h(x) > 1 r h(x) > O
/fM<gM J f (X) > g(x) f(x) < g(x) irJ h(x) < 1
=> Iog 2 -J < log22x < log23 => Iog2 f < < log23.
lf(x) > 0 Ig(X) >0 lf(x)>0 1 f(x)>g(x)
L
g(x)> O
Ats.: (Iog2 j ; log 2 3).

Pvz..

Isprskite nelygybes: Sprendimas.


Sprendiame lygt - g = 0.
2
1. I o g 1 ( S x - I ) > 0.
5 Jos apibrimo sritis: > 0; ? 1.
Sprendimas. Skaitiklis lygus nuliui, kai = y .
Iog^ (3x - 1) > Iog^ 1
Paymime takus = 1 ir = skaii tiesje.
Kadangi < 1, tai duotoji nelygyb ekvivalenti sistemai

3x - 1 < 1,
\ 3x - 1 > 0. 3

I ia 2
Vadinasi, O < < 1; > .
<.
\ x > i . Ats.xe (0; l)U(J;oo).
3
1 < x < 2
Vadinasi, "3 3"

Ats.: ( - f).
Udaviniai. Bet kurio k a m p o

Isprskite nelygybes: sina =

1. log 8 (x 2 - 4x + 3) < 1. cosa

Afs.; (-1; 1)U(3;5). tga = ^ ,

2. 3 " * ^ > < . ctga = - j - .

A f s . ; (1; 2). Trigonometrinis skritulys R= 1

3 . ( - 1 y o 9 j (>? - 2x - 3) > 1
Pagrindins tapatybs
A f s . : (1 - V"; - 1) U (3; + 1).
sin 2 a + cos 2 = 1
4. ( ) 1 0 > 1. 1 1 1
1 + tg 2 a = 1 + ctg 2 a = 2 ctga =
cos2a Sin Ct tga
A f s . : (-oo;-1,5) U (4; oo). sina cosa
tga = ctga tga ctga = 1
cosa sina

Funkcija Apibrimo Reikmi Lyginumas


36. Trigonometrins funkcijos f sritis D(f) sritis

Smailiojo kampo sinx oo + o o )


nelygin
[ - 1 ; 1]

sina = , COSX OO J + OO^


[-1:1] lygin
c
b tgx P + , k e Z OO J + oo)
cosa = nelygin

ctgx P , k e Z oo -t- o o ) nelygin


tga= -f-,

_
ctga = ,

seca = - -
37. Pagrindini kamp Trigonometrini funkcij pagrindiniai periodai
trigonometrins funkcijos Sinuso ir kosinuso T = 2; tangento ir kontangento T = .
Vadinasi, sin(x + 2) = sinx.
O0 30 45 60 90 180 270 360 cos(x + 2) = cosx.
tg(x + ) = tgx
3
0 6 4 3 2 2 2 ctg(x + ) = ctgx.

sina 0
1 NIT VT 1 0 -1 0 Pastaba: Periodas funkcijos, ireiktos keli periodini tolydi
2 2 2
funkcij suma, lygus funkcijos dmen period bendram maiausiam
cosa 1 / <7 1
0 -1 0 1 kartotiniui.
2 2 2

/
tga 0 3
1 V31 - 0 - 0 Pvz.:

ctga - V3" 1 3
0 - 0 - 1. Rasti funkcijos y = cos2J3x - sin26x + y trumpiausi period.

Sprendimas.
T r i g o n o m e t r i n i f u n k c i j enklai
y = cos26x - sin 2 6x + y = cos12x + y .
sirux cosa tga, ctga
cos12x = cos(12x + 2n) = cos(12(x + i )).

Ats.:4-.
6

2. Raskite funkcijos y = tg-|- + sin ~ + cos6x trumpiausi

period.

sina enklas sutampa su ordinats Oy enklu, o cosa reikms Sprendimas.


enklas - su absics Ox enklu.
tga ir ctga enklai yra teigiami, kai sina ir cosa enklai vienodi ir t g f = t g ( y + ) = t g ( ( x + 2)), T1 = 2
neigiami, kai sina ir cosa enklai prieingi.
s i n y = sin(y + 2) = sin(~(x + 4)), T 2 = 4

Periodikumas cos3x = cos(3x + 2) = cos(3(x + ) ) , T


3 = ^
o 3
(2; ] ) = 4.
Skaiius T A O vadinamas funkcijos f(x) periodu, jei f(x + T) = f(x),
kur e D(f). Maiausioji teigiama T reikm, jei ji egzistuoja, vadinama Ats.: 4.
pagrindiniu periodu.
Redukavimo formuls _ sina , _ JL IZ.- JL
tga
" cosa ~ M' 17 " 17 '15 " 15 '
Funkcija f- -2-+a - a + a -2- a" - f +a 2 - a 1
15 _ . 1
^a =
tga I - - 1
T -
sinx cosa cosa sina - sina - cosa - cosa - sina
8
4. Su kuriomis m reikmmis teisinga lygyb sinc) = ,
COSX sina - sina - cosa - cosa - sina sina cosa e [0; ]
tgx ctga - ctga -tga tga ctga - ctga -tga Sprendimas.
sinO = 0, sin =1
ctgx tga -tga - ctga ctga tga -tga - ctga
2m2 5 8
m; +7 ^1I- (m2 + 4) > 0
Pvz..
0 2m 2 - 5m + 84: m 2 + 4
1. Apskaiiuokite sin(- 405).
2m 2 - 5m + 8 0,

Sprendimas.
{ 2m 2 - 5m + 8^< m 2 + 4.
- visada teigiamas

Sprendiame sistemos antrj nelygyb:


sin(- 405) = - sin405 = - sin(360 + 45) = - sin45 = ~
m 2 - 5m + 4~< 0 + +

Ats,- f (m-4)(m-1)^0 ^ :
5 >m

m e [1; 4].
2. Apskaiiuokite cos1575.
{ ' + 00
Gavome f m e (- )
Sprendimas.
m e [1; 4]
cos1575 = cos(360. 4 + 135) = cos135 = cos(90 + 45)
Ats, m e [1; 4].
= - sin45 = - .
5. Nustatykite funkcijos y = 5sinx - 12cosx reikmi srit.
Ats,-f.
Sprendimas.
3. Duota: sina = - - , ~ < a < 2 . Pertvarkome taip, kad gautume formul
sin(a - ) = - sinpcosa
Raskite cosa, tga, ctga. 12 5
sina = jo - , cosa = .
Sprendimas.
y = 13 -^sinx - 13 cosx =
2
cosa = +_V 1 - sin a ', bet a i IV ketvirio, tai c -10
= 13( ^ s i n x - cosx) = 13(cosasinx - sinacosx) =
05=
cosa = ) T
cosa = Y-;
' f ;
5
= 13sin(x - a)
6. Parodyti, kad parabol y = 2 - + 5,35 nekerta funkcijos
39. Sinuso ir kosinuso laipsniai
y = 2sinx + 3 grafiko.

2cos 2 x = 1 + cos2x
Sprendimas.
1. Funkcijos y = 2 - + 5,35 grafikas yra parabol, kurios akos 2sin 2 x = 1 - cos2x
nukreiptos vir. i funkcija gyja maiausi reikm virnje, t.y. take (sinx + cosx) 2 = 1 + sin2x
X0 = 0,5, y 0 = ymin = 5,1. E(y) = [5,1; + - ) .
Puss kampo formuls
2. Rasime ir funkcijos y = 2sinx + 3 reikmi srit.
- 1 ^ sinx ^ 1, tai - 2 < 2sinx ^ 2, o 1 << 2sinx + 3 ^ 5 ,
I
jcos 2 : j f 1 + C0SX "
t.y. E(y) = [1; 5]. ymax = 5. I .
Sin )/ 1 - cosx~"
I reikmi srii aiku, kad funkcij grafikai neturi bendr tak. F 2
t X= r Sinx 1 - COSX _, I I -7
S 2 T Tcosx" = " i E T " ; X ^ ^k, k e Z
38. Sudties formuls
40. Trigonometrini funkcij sumos
COS(x + y) = COSX-cosy - sinx-siny keitimas sandauga
C0S(X - y) = COSX-cosy + sinx-siny
sin(x + y) = sinx - cosy + cosx -siny x y
cosx + cosy = 2cos * cos -
sin(x - y) = sinx.cosy - c o s x . s i n y
x + y
+ cosx - cosy = - 2sin sin ^ y -
tg(x ) = T ^ t i ^ , + , y , ' 2 2
y
sinx + siny = 2sin Cosiy .
+ ? , n e Z
x y
sinx - siny = 2sin ^ L cos *

sin(x + y )
tgx + tgy =
x-y +, n e Z cosxcosy
sin(x y)
tgx - tgy - -
cosxcosy
Dvigubo kampo formuls

cos2x = Cos2X - sin 2 x


41. Trigonometrini funkcij sandaugos
cos2x = 2cos 2 x - 1 keitimas suma
cos2x = 1 - 2sin 2 x
sin2x = 2sinxcosx cosx cosy = Y (cos(x - y) + cos(x + y))

tg2x
M = ^ t - , ^ + ,. sinx siny = (cos(x - y) - cos(x + y))
1 - tg 2 x 2

sinx cosy = (sin(x + y) + sin(x - y))


Pvz.: Udaviniai.

1. [rodykite tapatyb (sina - cosa) 2 = 1 - sin2a. rodykite tapatybes:


2sing - 2sin2g
Sprendimas.
2sing + sin2g ~ *9 T
Pertvarkome kairij lygybs pus:
(sina - cosa) 2 = sin 2 a - 2sinacosa + cos 2 a =
sin2(g + ) - sin2(g - p)
= sin 2 a + cos 2 a - 2sinacosa = 1 - sin2a. 2 =
sinacosp 4cosasinp.
Lygybs kairiosios ir deiniosios pusi reikiniai sutapo.

2. rodykite tapatyb - t g a ) ( ^ ^ + tga) = 2cos 2 a - cos2a. 3


^ 0 , 5 - 0,5^0^5 + 0,5cos2a = c o s f , k a i < 2 < 2 .
Sprendimas.

Pertvarkome kairij lygybs pus:


42. Trigonometrini funkcij grafikai
, 1 . W 1 . . _ 1 . 2 _ 1 sin2g _ 1 - Sin2H _
' cosa * t Q a Zt c o s a t a 2
Q / ~~ cos a" " a
cos a " cos2a
2
cos2a
_ cos2a _ ^
cos2a

Pertvarkome deinij lygybs pus:


2cos 2 a - cos2a = 2cos 2 a - (cos 2 a - sin 2 a) = 2cos 2 a - cos 2 a +
+ sin 2 a = cos 2 a + sin 2 a = 1.
Gavome, kad kairioji ir deinioji lygybs puss lygios 1.
Vadinasi, tapatyb rodyta.

1 + 2
3. rodykite tapatyb tg( f + ) = 02 W tg y = CtgX

Sprendimas.
rodysime, kad t g ( + ) - 1
= 0.
43. Atvirktins trigonometrins funkcijos

t /_+ 1 + sin2a _ sin(ij- +) 1 + sin2cx _ arcsinx = y (y e [ | ; ] , [-1 ; 1 ], kur siny = x).


y
M >' cos2a cos(-| + ) " cos2a
_ sinj cosa + cos sina _ 1 + sin2a _ arccosx = y (y e [0 ; ] , e [-1 ; 1], kur cosy = ),
cos i cosa7- sin -j sina cos2a - sin2a
i? V2- .
_ cosa + -g sina 1 + sin2a _ arctgx = y (y e (- ) , e (- + oo), kur tgy = ),
! cosa -M. sina (cosa - sina)(cosa + sina)
2 2
arcctgx = y (y e (0 ; ), e (- +>), kur ctgy = ).
y(cosa + sina) 1 + sin2a _
(cosa - sina) (cosa - sina)(cosa + sina) arcsin(- a) = - arcsina, arccos(- a) = - arccosa.
_ cos2a + 2cosasina + sin2a - 1 - sin2a
(cosa - sina)(cosa cos) arctg(- a) = - arctga, arcctg(- a) = - arcctga.
_ 1 + sin2a - 1 - 2sin2a _ 0 _ q
cos2a - sin2a cos2a
- 104-
Pvz.: 44. Trigonometrins lygtys
arcsin(- 1) = arccos(- 1) = sinx = a, I 1
= (-1)"arcsina + , n Z
arcsinO = O arccos O = y
sinx = 0
arcsinl = y arccosl = O = , n e Z
arctg(-1) = - f arcctg(-1) = sinx = 1

arctgO = O arcctgO = = I- + 2, e Z

arctgl = ^ r arcctgl = y cosx = a


= t arccosa + 2, n e Z

cosx = 0
Pvz.: = + , n e Z
1. Raskite x, jei inoma, kad - 1, + 2, sin(arcsinx) ia tvarka cosx = 1
sudaro geometrin progresij. = 2, e Z
Sprendimas. tgx = a
Kadangi sin(arcsinx) = , kur - 1 , tai gausime geometrin = arctga + , e Z
progresij - 1, + 2, . tgx = 0
Vadinasi, - 1 x = ( + 2)2, x2 + 5x + 4 = 0, x = 1 , x = - 4 ( x = - 4 = , e Z
netenkina slygos - 1-< ^ 1).

Ats.: -1.
45. Trigonometrini lygi
2. Raskite y = arccos(2 - ) apibrimo srit. sprendimo metodai
Sprendimas.
Lygtys, sprendiamos gaunant vieno argumento
trigonometrines funkcijas
- 3 < - f < - 1
1. sin2x + 4cosx = 2,75
1 - Cos2X + 4 c o s x = 2 , 7 5
U y ^ 2
c o s x = t; | t ! <; 1
t 2 - 4t + 1,75 = O
2 < -< 6.
t=-i:t= >1
cosx = ;
Ats.: e [2; 6].
X = j + 2, n e Z
2. tgx + 3ctgx = 4 " 2x = + nk, k e Z
t g * + tr = 4 . sin2x = -- > 1.
tgx = t
t 2 - 4t + 3 = O
Ats.: =-j + - y - , k eZ
t = 1, t = 3
x = j + , e Z
Pvz.:
= arctg3 + nk, k e Z

1. Isprskite lygt 3sin6x - V7 cos3x = 0.


Homogenins trigonometrins lygtys
Sprendimas.
1. 2sinxcos - Cos2X = O 3 2sin3xcos3x - V37 cos3x = O
cosx(2sinx - cosx) = O cos3x(6sin3x - V37 ) = O r,
cosx = O arba 2sinx - cosx = cos3x = O, 3x = y + , n e Z arba 6sin3x - V37 = O, sin3x = - y ,
= y + , n e Z Lygties cosx = O aknys
netenkina ios lygties, todl abi x =
+
"^f"' n e z
Gautoji lygtis neturi sprendini,
lygties puses dalijame i cosx P 0:
2tgx - 1 = 0 ,
tgx = |
= arctg + , n e Z Ats.: x=y+y-,neZ.

2. 5sin 2 x + sinxcosx - 2cos 2 x = 2 2. Isprskite lygt sin 2 x + 3x 2 cosx + 3x 2 = 0.


5sin 2 x + sinxcosx - 2cos 2 x = 2sin 2 x + 2cos 2 x
3sin 2 x + sinxcosx - 4cos 2 x = O Sprendimas.
(1 - cos 2 x) + 3x 2 (cosx + 1) = 0
Abi lygties puses daliname i cos 2 x: (1 - cosx)(1 + cosx) + 3x 2 (1 + cosx) = 0
3tg 2 x + tgx - 4 = O (1 + cosx)(1 - cosx + 3x 2 ) = O
tgx = 1; tgx = - -. 1 + cosx = O arba 1 - cosx +3x 2 = O
= , e Z cosx = - 1 2sin 2 y + 3x 2 = O
= - arctg j - + , k e Z = + 2^ k e Z Lygtis ekvivalenti sistemai
Tsin -- = O
I3x2 = O
Skaidymas dauginamaisiais
e
sin4x =3cos2x
{ y = , n
x = 0
Z

2sin2xcos2x = 3cos2x
e
2sin2xcos2x - 3cos2x = O
cos2x(2sin2x - 3) = O { = 2, n
x = 0
Z

~cos2x = O
2sin2x - 3 = 0 = 0.
Ats.: = + 2nk, k e Z, = 0.
8. Isprskite lygi sistem:
3. Isprskite lygt sinx + sin9x = 2.
a) Vx+ y = J r t b) fx + y = fit
\sinxsiny = j . \ tgx + tgy = 2\'3
Sprendimas.
Kadangi |sina|<: 1, tai duotoji lygtis ekvivalenti sistemai:
Ats.: a) = (2 + 3n) y , y = (2 - 3n) y , n e Z.
{
sinx = 1;
sin9x = 1.
b) = y + , y = , n e Z.

Sprendiame lygt sinx = 1;


= 2, n e Z. 46. Trigonometrini nelygybi sprendimas
Vadinasi, tiks tos reikms, su kuriomis teisinga lygyb naudojantis grafikais
sin(9(-| + 2Ttn)) = 1, n e Z
s i n ( ^ + 18) = sin(4n + y + 2 9) = sin \ = 1.
V* Sprendiant trigonometrines nelygybes patogu naudotis grafikais.

Kai = Y +
2, n e Z, gavome teising lygyb sin9x = 1. Pvz.:

+
t
Ats.: x = y 2, n e Z.
1. Isprskite nelygyb sin < .

Udaviniai.
Sprendimas:
Briame funkcij y = sinx ir y = i - grafikus.
Isprskite lygtis:
. ~
1. 2sin-2

COSy
5
= siny- . / \ n/I \ f \
_Z*V i 5 4\ / 13 '
6 6 6 ^ "6-
2. Raskite m reikmi aib, su kuriomis lygtis cos3x = 2m + 7
turi sprendinius.
1
I lygties sinx = y randame grafik susikirtimo tak abscises.

3. 3sin 2 (x ) = cos(x - y ) . = ( - 1 ) | - + , Z.
4. 4 Sin2X + 8sinxcosx + IOcos2X = 3. Duotos nelygybs sprendimai bus tos reikms, su kuriomis
funkcijos y = sinx grafikas yra emiau tiess y = y .
5. 1 + 2cosx + cosx = cos 2 45 + sin 2 135.
Ats.: ( - I l + 2\- + 2), n e Z.
6. 3sin 2 x - 2sin2x + 5cos 2 x = 4log 3 tg60.
2. rodykite, nelygyb cosx < 1 + 2 ^jtfx .
7. Raskite lygties 1 + sin2x = (sin3x + cos3x) 2 sprendini sum,
kai e [ - - ; ] . rodymas.
1
1 + , > 1 + \ > cosx.
Ats.: 0. 2 - Sin 2 X
3. Isprskite lygt sin5x - 2cos2x = 3.
V(sinx-)2 ^

Sprendimas. I sinx
- 111 < JL
1
1 2 2
Kadangi sin5x.< 1, - cos2x $ 1, tai kairioji pus lygi 3, jei
- 1 <: s i n x - 1 <1
2 2^2
{ sin5x = 1,
cos2x = - 1
Isprskime vien i lygi, po to irinksime i gautj reikmi
tas, kurios tenkina ir kit lygt:
cos2x = - 1.
2x = + 2,
= + nk.

Tada sin5x = sin(-|^ + 5nk) = s i n ( ^ +) = sin(2 + y + nk) =


= sin(y+nk). 2 < < JL+ 2, n e Z.

Vadinasi, kai k = 2n, tai s i n ( y + 2) = s i n y = 1.


Ats.: e [ 2 ; | - + 2], n e Z.

Ats.: j + 2, n e Z.
Udaviniai.
4. Isprskite lygi sistem: f 2x - sinx = 2y - siny,
\ x + 2y = 9.
Isprskite nelygybes:
Sprendimas.
Kairje pirmosios lygties pusje yra funkcija f(x) = 2x - sinx, o
1.sinfx^-f:.
deinje - f(y) = 2y - siny.
f (x) = 2 - cosx
Ats.: (8n - 3 ) | < ^ (8n - 1) | , n e Z.
f(x) > O, tai f(x) didjanti.
Todl lygtis turi vien sprendin
2. cosx < y .
f(x) = f(y), kai = y.
Ats.: (6n + 1 ) | < < (6n + 5) y n e Z.
Tada 3x = 9
= 3. . "I
3. j COSXI > 2

Ats.: (3; 3).


Ats.: y + < < y + , n e Z.

5. Isprskite nelygyb Vsin2X - sinx + ^ j .


4. Raskite funkcijos y = V4cos 2 x - 3' apibrimo srit.

Sprendimas.
Ats.: + ^< <: J - + , n e Z.
b b
2
Vsin X - Sinx +"4
47. Ivestin Ivestini lentel Pvz.:

C' = O, C e R (pastovus dydis) 5' = O


x' = 1, e R
(xk)' = kxk \ k e R, > O (>/)' = (*)' = 4 - 2Vx^
(cosx)' = - sinx, X e R
(sinx)' = cosx, x e R
(tgx)'=--,x?f +, k e Z

(ctgx)' = , 7- , e Z
Funkcijos y ivestin take x 0 vadinama funkcijos pokyio Ay =
= f(x + ) - f(x 0 ) santykio su argumento pokyiu riba, kai argumento (Inx)' = - , e (0; +
pokytis artja prie nulio.

f(X + Ax) - f(xn) (loSax)' = . ^ +

f(x 0 ) Iim Ax
*0
(e x )' = e x , e R
(a x )' = a x lna, e R

48. Kreivs liestins lygtis 50. Sudtins funkcijos ivestin


M 0 (x 0 ; y 0 ) - Iietimosi takas.
(Uk)' = k u k - 1 . u ' ((1 - 2x) 3 )' = 3(1 - 2x) 2 (1 - 2x)' =
= 3(1 - 2x) 2 .(- 2) = - 6 ( 1 - 2x) 2
f(x 0 ) = tga.
(cosu)' = - sinu -u' (coskx)' = - sinrtx -()' = - =
Kreivs y = f(x) liestins take M 0 (x 0 ; y 0 ) lygtis
+ x x = -
= y0 f(x 0 )( - o)
(sinu)' = cosu -u' ( s i n 2 y )' = 2sin j - ( s i n y )' =

= 2sin j - C O S y ' ( y ) ' = sin - y - ' y =


49. Diferencijavimas
- 1 2x
-Ts i n
X
Tam, kad funkcija bt diferencijuojama (turt ivestin) take
2
x0, btina (bet nepakanka), kad ji bt netrki tame take (t.y. Af(x) = f(x)- (tgu)'
v 3
'
-
COS2U
2 -u' (tg 3 2x)' = 3tg 2 2x J 2 2 x (2x)' =
-f(xQ)^0).
_ 6tg 2 2x
cos 2 2x

(ctgu)'
v 3
'
- - Sin
2U u'

v?
sin2V?x
52. Ivestins mechanin prasm
(Inu)' = r - u ' (lnV2x = (VZ-Vx)' =
Sakykime, s = s(t), tada Pvz.
_ 1 . 1 _ 1
2 v = v(t) =s'(t). Knas juda tiese taip, kad atstumas
- "^ - 2
Greitis - kelio ivestin pagal laik. S(t) nuo pradios tako kinta pagal
1
(IogaU)' = (Iog3Vx )' = J l n 3 ( ^ ) ' , ^ = a = a(t) = v'(t) = s"(t). dsn s = 12t - 3t2, kur t - judjimo
Pagreitis - greiio ivestin pagal l a i k a s s e k u n d m i s . Po kiek
_ 1
2xln3 laik (antroji kelio ivestin pagal sekundi nuo judjimo pradios
laik). knas sustos?
(eu)' = e u V ( 2 )' = e*2 (2)' = 2. 2 = 22
Sprendimas.
(au)' = a u lna ' u ' (32)' = 3 2 3.(2)' = 233 2 v(t) = s ' = ( 1 2 t - 3 t 2 ) ' = 1 2 - 6 t .
v(t) =0; 12 - 6t = 0; t = 2.
Pastaba: u = u(x)
Ats.: po 2 sekundi.

Antrosios ivestins mechanin prasm


51. Diferencijavimo taisykls
(sin3x + 2)' = 3cos3x + 2x Tiesiaeigio kno judjimo duotuoju laiko momentu pagreitis yra
(u + V)' = u' v'
antroji kelio ivestin pagal laik, t.y., jei s = f(t), tai a = s".
(u v)' = u' V + U ' 1) ( 2 e4x)' = (2)' e4x + 2 (e4x)' = Antrja funkcijos y = f(x) ivestine vadinama jos ivestins ivestin
= 2x -e4x + 2 -e 4x 4 = 2xe 4x + 4x 2 e 4x ir ymima y"; y"(x).
Ivada: (c u)' = cu'
2) (3cos2x)' = - 3sin2x - 2 = - 6sin2x

., ,4-x v (4-)'5-(4-)(5)' _
53. Funkcijos tyrimas ivestins pagalba.
/ u v u'v -i U-V' 1
(v ) - V ) ( 5x ) " 25x2 Funkcijos monotonikumo intervalai
-1 5x -(4 - x)*5
25x2
Jei f(x) > 0 intervale (a; b), tai f(x) Pvz:
_ - 5x - 20 + 5x _ 2 4 didja intervale (a; b). f(x) = X2 - 8x + 2
25x2 5x
f(x) = 2x - 8 > 0, kai > 4 => f(x)
O / X2 _ (*2)' _ 2x _ x Jei ta pati f u n k c i j a f(x) netrki didja intervale [4; + <*>)
\ > - 4 ~ 4 ~ 2 i n t e r v a l e [a; b], tai f(x) d i d j a
intervale [a; b]
Pvz.:
Jei f ( x ) < 0 intervale (a; b), tai f(x) f(x) = V -
maja intervale (a; b)
f(x) = - < 0, kai X < 0 =>
Jei ta pati f u n k c i j a f(x) netrki
intervale [a; b], tai f(x) maja f(x) maja intervale (- 0].
intervale [a; b]
Funkcijos kritiniai takai Vadinasi, norint rasti funkcijos ekstremumus intervale (a; b), reikia
rasti lygties f (x) = 0 aknis.
Funkcijos apibrimo srities vidiniai takai, kuriuose ivestin lygi Sakykime, kad aknys C1, c 2 , ..., ck.
Nustatome ivestins f(x) enklus kiekviename intervale (a; C1),
nuliui arba neegzistuoja, vadinami funkcijos kritiniais takais.
(C1; c 2 ) (Ck; b).
Jei pereinant vien t tak, sakykime, c, ivestin keiia enkl
Pvz.:
i (-) (+), tai tame take funkcija turi minimum, jei i (+) (-) - tai
maksimum.
Raskite funkcijos kritinius takus:

Antras bdas.
1) y = Vx .
Sakykime, C1, c 2 , ..., ck - lygties f(x) = 0 aknys. Randame antrj
Sprendimas. ivestin f'(x) ir nustatome jos enkl su kiekviena C1, c 2 , ..., c k reikme.
Jei, pavyzdiui, f'(a) < 0, tai tame take turi maksimum, jei
y = W
f'(Cj) > 0 - tai minimum.
Ivestin neegzistuoja take = O, bet jis nra apibrimo srities
Pastaba.
vidinis takas, todl funkcija neturi kritini tak.
Jei f ' ( c ) = 0, tai antruoju bdu rasti funkcijos ekstremumo take c.
negalima.
Ats.: Nra kritini tak.

Pvz,
2. y = 2 -

Sprendimas. 1. Raskite funkcijos f(x) = 2x 3 - 3x 2 - 1 e k s t r e m u m u s ir


y' = 2x - 1 = O => = monotonikumo intervalus.

Sprendimas.
Ats.: .
D(f(x)) = R.
f (x) = 6x 2 - 6x = 6x(x - 1).
54. Ekstremumai f(x) = 0, kai = 0; = 1.

T(X0)= O, kai < x0, tai f(x) > 0; kai f () = 0, kai < x0, tai f(x) < 0, kai ->x
> x0, tai f(x) < 0. > x0, tai f () > 0 ^ 0 V1 1 ^
max min
f(x) didja intervaluose (- 0) ir (1; +
f(x) maja intervale (0; 1).
0 - maksimumo takas, 1 - minimumo takas,
f max(0)
v
' = - 1;' f (1)' = 2 - 3 - 1 = - 2.

Ats.: f(x) didja intervaluose (- oo; 0) ir (1; + oo), maja intervale


rr.in ( ; i ) ; U i ) = -2;fmax(0) = - i .
2. Raskite funkcijos f(x) = + monotonikumo intervalus ir 55. Didiausia ir maiausia funkcijos
ekstremumus.
reikm intervale
Sprendimas.
D(f(x)) = (- 0) U (0;). Didiausia (maiausia) funkcijos reikme intervale t
f(x) = 1 -
v a d i n a m a s toks skaiius M(m), kuriam egzistuoja X0 e M o k s , kad

f(x) = 0, kai = 1 ir = - 1. f(x 0 ) = M, (f(x 0 ) = m), M > f ( x ) (m ^ f(x)) visiems i intervalo t.


+ - -

SI -1 0 Vsl 1 ^r
min
D i d i a u s i o s ir m a i a u s i o s f u n k c i j o s , n e t r k i o s i n t e r v a l e
max trkio
takas
[a; b], r e i k m i i e k o j i m a s i n t e r v a l e [a; b]

f(x) didja intervaluose (-<*>; - 1 ) ir (1; oo). 1. Reikia rasti f(a) irf(b) - funkcij reikmes intervalo galuose.
fix) maja intervaluose ( - 1 ; 0) ir (0; 1). 2. Reikia rasti funkcijos kritinius takus intervalo (a; b) viduje (t.y.
f max = - 2 ;' f.(1)' = 2. intervale (a; b)).
3. Reikia rasti funkcijos reikmes kritiniuose takuose.
Ats.: f(x) didja intervaluose (- - 1) ir (1; f(x) maja 4. I vis gaut reikmi reikia irinkti didiausi ir maiausi. Jos
intervaluose (-1; 0) ir (0; 1); - 1 - maksimumo takas, 1 - minimumo takas; ir bus didiausia ir maiausia funkcijos reikm intervale [a; bj.
f max*(_ 1)' = - 2;' f min
4 . (1)' = 2.
Pvz.:
3 5
3. Raskite funkcijos y = 5x - 3x didjimo, majimo intervalus ir
ekstremumus. 1. Raskite maiausi funkcijos f(x) = 3x2 + 18x + 7 reikm
intervale [- 5; - 1 ] .
Sprendimas. Sprendimas.
D(y) = ( - oo; o o ) . f(x) = 6x + 18.
y' = 15x2 - 15x4 = 15x2(1 - x)(1 + x) f(x) = 0; 6x + 18 = 0; = - 3.
y' = 0, kai = 0; = 1; = - 1. f(- 5) = - 8; f(- 1) = - 8; f(- 3) = - 20 - maiausia reikm.
Funkcijos y kritiniai takai skaii a padalija 4 intervalus:
Ats.: minf(x) = f(- 3) = - 20.
[-5:-11
2. R spindulio pusskritul brkite didiausio ploto staiakamp
taip, kad dvi virns bt ant apskritimo, o kitos dvi ant skersmens.
Randame ivestins enkl pirmame i kairs intervale:
y'(- 2) = 15(- 2)2(1 + 2)(1 - 2) < 0
Vadinasi, funkcija y maja intervaluose (- oo; - 1 ) ir (1; oo), o didja Paymime:
intervaluose ( - 1 ; 0) ir (0; 1). AB =
y mjn (-1) = 5 ( - 1 ) 3 - 3 ( - 1)= = - 5 + 3 = - 2 AD = 2y.
Y 1 1 J D = 5 - 1 - 3 - 1 = 2. 0 D

Ats.: y min (-1) = - 2 ; ymax(1) = 2; funkcija didja intervaluose ( - 1 ; 0)


ir (0; 1); funkcija maja intervaluose (- - 1 ) ir (1; Tada S = 2xy = 2x\ R2 - x 2 ' , nes y =VR2 - x 2 '.

- 121 -
S = 2\R 2 x 2 - 4 ', e (0; R). 56. Ivestini taikymas sprendiant
Randame funkcijos f(x) = R2X2 - x4 maksimum intervale (0; R):
f(x) = 2R2X - 4x 3 = 2x(R 2 - 2x 2 )
vairius udavinius
Pvz.:
f(x) = 0, kai = ( x e (0; R)).
+
> 1. Su kuriomis c reikmmis parabols y = x2 - 6x + c virn
yra nutolusi nuo koordinai pradios per 5 vienetus?

Sprendimas.
Kai = , tai staiakampio plotas didiausias.
Randame parabols virns tako A
koordinates
Ats.: ~ ; RV? .
V2 y' = 2x - 6,
2x - 6 = 0,
3. Raskite funkcijos y = sinx - / cosx ekstremumus intervale
[0; ]. Xvir.
4 = 3,'
>X = 32-6'3 + c = c-9.
Sprendimas.
D(y) = R.
Vadinasi, A(3; c - 9).
y' = cosx + V3 sinx,
inodami, kad atstumas OA lygus 5, sudarome lygt:
cosx + ViTsinx = 0 I : cosx,
3 2 + (c - 9)2 = 52
1 + VTtgx = 0,
(c - 9)2 = 16
c - 9 = 4 arba c - 9 = - 4
^ = - c = 13; c = 5.

= - -g- + , m e Z. Ats.: 5; 13.

Kai m = 1, = -y-, m e Z. 2. Ties y = 6x - 7 lieia parabol y = x2 + bx + c take A(2; 5).


Paraykite parabols lygt.
Tik vienintelis takas = max
- 6 - priklauso duotajam intervalui [0; ].
-> X Sprendimas.
5
6 1) y' = 2x + b
'() > 0
y' = 2- 2 + b
/(*.)<0
4 + b = 6
Vadinasi, duotoji funkcija take = -g- gyja maksimum. b = 2.

y max (f > = sin -VScosf =sinf+V3.cos-5- = 2) y = X2 + 2x + c

= =2. 3) Kadangi parabol eina per tak A(2; 5), tai:


5 = 2 2 + 2 - 2 + c.
Ats.: Ymax = 2, kai = .
I ia = - 3.
2x - 2
Vadinasi, y = 2 + 2x - 3. 5. Turime funkcij y = x + 1 . Raskite susikirtimo su koordinai
aimis takus t liestini, kuri krypties koeficientas 4.
Ats.: y = 2 + 2x - 3.
Sprendimas.
, 4
3. Raskite trumpiausi atstum nuo tako M(~-; 1) iki kreivs Y = (X + 1)2
= X2 +
Domina tos liestins, kuri k = 4.
2
Sprendimas. inome, kad k = y'(x0). Vadinasi, -.~jr
+ 22 = 4, (x0 + 1) = 1,
(x0 1)
2
Sakykime, kad tas kreivs takas yra N(x; x + y ) . X0+ 1 = - H 1 X 0 = p, X0 = - 2 .

f(x) = |MN M (X - g-)2 + (X2 - j ) 2 a) X0= 0, y(0) = - 2, y'(0) = 4.


Liestins lygtis: y = - 2 + 4(x - 0) = 4x - 2.
f(x) = 2(x - y ) + 2(x2 - ) 2x = 2x - j + 4x 3 - 2x = 4x3 - --. Ji kirs Ox a take (0,5; 0), o Oy a - (0; - 2).

f (x) = O, kai 4x 3 - = O, = . b) X0 2, y(- 2) = 6, y'(- 2) = 4.


= -

-> Liestins lygtis: y = 4x + 14.


Ji kirs Ox a take (-3,5; 0), o Oy a - (0; 14).

1
Xmin. = -TT
Z
Ats.: (0,5; 0); (0; - 2); (- 3,5; 0); (0; 14).

TAI I M N I = ] / ( F } ) 2
+ (14) 2 +
= = W 6. Raskite funkcij y = x2 ir y = x3 grafik bendr liestini lygtis.

Ats: Sprendimas.
~i2r
Sakykime, kad antrosios funkcijos y = x3 grafiko take (t; t3)
4. Raskite takus, kuriuose funkcijos y = grafiko liestin su liestins lygtis yra y = t3 + 3t2(x -1),
krypties koeficientu - 1 kerta abscisia. y = 3t2x - 2t3.
Ta ties turi liesti funkcijos y = x2 grafik.
Sprendimas. Todl 2 = 3t2x - 2t3, t.y. X2 - 3t2x + 2t3 = 0.
D =O
y
. _ - 3 - 2(x + 1) _ 2-4 4 - 8t3 = O
(x-3) (x-3)
t = 0;t= f .
4
Duota slygoje y' = - 1 , todl - = - 1; ( - 3)2 = 4;

X 1 = 1; X 2 = 5.
(X 3) Vadinasi, bendrj liestini lygtys yra:

Kai Iietimosi tako abscis = 1, tai y(1) = - 1; y'(1) = - 1 ir y = 0 ir y = 3 - f x - 2 . | f


liestins lygtis: y = - 1 - 1(x - 1 ) , t.y. y = - .
Kai Iietimosi tako abscic = 5, tai y(5) = 3, y'(5) = - 1 ir liestins = 64 1024
W
y
27 " 729 '
lygtis: y = 3 - (x - 5), t.y. y = - + 8.
.. n 64 1024
Liestins kerta abscisia tada, kai y = O, t.y. takuose (0; 0); (8; 0). Ats.: y = 0; y = - .

Ats.: (0; 0); (8; 0).


7. Valtis M yra 3 km atstumu nuo artimiausio kranto tako A. Valties Sprendimas.
keleivis nori pasiekti kranto tak B, esant 5 km atstumu nuo A. Valtis Duotosios funkcijos apibrimo sritis: (0; +
plaukia 4 km / vai. greiiu, o keleivis, ilips i valties, eina 5 km / vai. 2
greiiu. kok kranto tak turi iplaukti valtis, kad keleivis pasiekt B per
trumpiausi laik? Randame funkcijos ivestin: f () = - 2x.

A 0 6
Sprendimas. Randame kritinius takus:

- 2x = O | * x
2 - 2x 2 = O
2 = 1
M
X 1 = - 1; X 2 = 1.
Sakykime, kad valtis M turi iplaukti kranto tak O. Tada laikas t,
per kur valtis pasieks B, bus lygus: duotosios funkcijos apibrimo srit patenka tik takas = 1.
2
_ VAM + AO 2
^ OB max
1
~ 2 5 T
>
2
V9 + X ' 5 O i
Paymime AO = x, tada OB = 5 - ir t = 3+ y-
f ( r ) = T - 2 4 = 4 - 1 >0
t'/v\ = 2x = X 1 2
y 2 2
' 4 2V9 + X ' 5 4V9 + X 5 '

f(x) = O, kai "I = ty- 4V9 + 2 = 5x; f(2) = T - 2 2 = 1 - 4 < 0

16(9 + 2) = 25x 2 , ymax(1) = 2ln1 - 12 = 2 - 0 - 1 = - 1.


9x2 = 9 -16. Vadinasi, intervale (0; 1) funkcija didja, intervale (1; oo)
2 = 16 funkcija maja ir ymax(1) = - 1.
= 4
min
T Ats.: intervale (0; 1) funkcija didja, intervale (1; <) funkcija
1 ? X
maja, V m a x ( I ) = - 1.

t'(3) < O, t'(5) > 0.


Vadinasi, valtis M turi iplaukti kranto tak O, esant4 km atstumu
Udaviniai.
nuo tako A.
1. Su kuria parametro a reikme funkcijos f(x) = a - x2 liestin
Ats.: tak, esant 4 km atstumu nuo tako A.
atkerta pirmame ketvirtyje nuo koordinatinio kampo lygiaon trikamp, kurio
plotas -9-?
8. Raskite duotosios funkcijos f(x) = 2lnx - x2 didjimo, majimo t
^ 32

intervalus ir ekstremumus.
Mis..
Ats 32 .
Jei funkcijos f(x) pirmykt yra F(x), rodymas.
2. Duota funkcija y = ax2 + bx + .
o k - pastovus dydis, tai funkcijos F(x) = cos2x 2 + 1 =
1) Uraykite funkcijos lygt, jei jos grafikas eina per takus
kf(x) pirmykt yra kF(x). = 1 + cos2x = f(x)
A(- 2; 3), B(- 1; 1), D(2; 7).
2) Uraykite liestini, einani per parabols
Jei funkcijos f(x) pirmykt yra F(x), Gautoji lygyb teisinga su
a) virn ir b) per tak E(2; 7), lygtis.
3) Kur turi bti takas C (tarp B ir D), kad keturkampio ABCD o k ir b - pastovieji dydiai, tai visomis realiomis reikmmis.
plotas bt didiausias? funkcijos f(kx + b) pirmykt yra
Y F(kx + b). 2) R a s k i t e p i r m y k t f u n k c i j
2
Ats.: 1) y = + + 1 f u n k c i j o s f(x) = 3x - 5, kurios
grafikas eina per tak (4; 10).
2) a) y = - y ; b) y = 5x - 3
Sprendimas.
3) C ( |
F(x) = - y - 5x + C.

Grafikas iekomos pirmykts


funkcijos F(x)eina per tak (4; 10),
57. Pirmykt funkcija ir vadinasi, F(4) = 10;
neapibrtinis integralas
- . 4 2 . 5 . 4 + C = 10,

Funkcija F(x) vadinama pirmykte V i s pirmyki funkcij aib


C = 10 - 24 + 20 = 6.
funkcija funkcijos f(x) intervale t, jei vadinama neapibrtiniu integralu ir
visiems i intervalo F'(x) = f(x) ymima Jf(x)dx.
F(x) = | - x 2 - 5x + 6.

Jei F ( x ) - v i e n a i f u n k c i j o s J f(x)dx = F(x) + C, C e R.


Ats.: F(x) = f x 2 - 5x + 6.
pirmyki funkcij intervale t, tai
F(x) + C yra bet kuri pirmykt
funkcija funkcijos f(x).
Pvz.:
Pirmyki funkcij savybs

1) rodykite, kad funkcija


Jei funkcijos f,(x) pirmykt yra
F(x) = y sin2x +
F1(X)1 0 funkcijos f 2 (x) pirmykt yra
yra pirmykt funkcija funkcijos
F 2 (x), tai f u n k c i j o s f,(x) + f 2 (x)
f(x) = 1 + cos2x realij
pirmykt yra F1(X) + F 2 (x).
skaii aibje.
58. Pirmyki funkcij lentel Pagrindins savybs
f(x) F(X) Intervalas t jf(x)dx = }f(t)dt = Jf(z)dz
a a a
k + c R
/f(x)dx = 0
X"
X" *1
+ C e N, e R;
a +1
( * - 1) - e N, e (- 0) U Jf(x)dx = - }f(x)dx
U (0; 00) a b
b c b
a <2 Z; e (0; jf(x)dx = jf(x)dx + jf(x)dx, jei a, b, ir c - bet kurie
j_ takai i intervalo
X In I + C (- o=; 0) arba (0; b b

e x x i kf(x)dx = k if(x)dx
e +C R a a

ax ~Ina + C R
a
Jtf1(X) + f 2 (x))dx - aIf 1 (x)dx +aJf 2 (x)dx
COSX sinx + C R

sinx - cosx + C R if(px + q)dx = ff(t)dt


1
COS2X tgx + C (-2"+ ; 2 + ), k e Z
1 Pvz.:
Sin2X - ctgx + C (; + ), e Z
o I
1. j 4 J - dx = / V + - )dx = ( # + f ) =4(42-1) +

59. Apibrtinis integralas + 2(47-1)= 4 - 7 + 2-1 =6 y

Jei funkcija f(x) netrki skaii aies intervale L, kurio takai 3


1 + cos2x 1
= a ir = b, tai pirmykts funkcijos reikmi F(b) ir F(a) skirtumas 2. J cos 2 xdx = j dx =2 i (1 + cos2x)dx =
24L 4i
yra pirmykt funkcija F(x) funkcijos f(x) intervale t, vadinamas Jt
+
apibrtiniu integralu funkcijos f(x) nuo a iki b = j ( Idx + }cos2xdx) = I ysin2x|! ) =
b
1 . 2 Vf 1
i f(x)dx = F(b) - F(a) 2(3--4)+ J(Sin^--Siny) 24
a

l,jL f L
- 2(12+ 4 - 2 )
2 3 2 + 4 . . 10
J
f dx
2 _
_ JL r dt2 _ 11 = .l/J_.lx=_L
- J (3x + 4)
4) 3 J t 3 t ) 90
3(-1) + 4

Keitinys 3x + 4 = t.

60. Kreivalinijins trapecijos plotas Iekomasis integralas I y g u s ^ skritulio su spinduliu 2 daliai,


t-y-, 2

Figra, apribota tiesmis y = 0; dX = I S = | 22 = .


= a; = b ir netrkios neneigiamos
funkcijos f(x) intervale [a; b] grafiku, Ats.: .
vadinama kreivalinijin trapecija.
Kreivalinijins trapecijos plotas
2. Raskite trikampio, apriboto abscisi aimi ir liestinmis,
S = }f(x)dx. nubrtomis i tako (0; 3) funkcijos y = | x 2 - + 5 grafikui, plot.

Sprendimas.
Pvz.: Paraome Iiestiniuy 1 ir y2 lygtis.
2
Sakykime, kad Iietimosi tak abscises yra X1 ir x2.
1. Apskaiiuokite JV4 - X2 dx.
O Tada 1 = ^ X
2
- X 1 + 5 + (X 1 - 1)(x-X1)
Sprendimas.
2 = y X 2 2 -X 2 + 5 + ( X 2 - X 1 ) ( X - X 2 ) .
Nagrinjame funkcij y = V4 - x 2 . Lygyb y = V4 - x2'
ekvivalenti sistemai Gautosios tiess eina per tak (0; 3), todl
r y2 = 4 - 2, r 2 + y2 = 4;
\ y ^ 0 ; I y 5-0. 3 = - ^ 2 - X 1 + 5 +(X1-D(O-X1)1

Funkcijos y = \ 4 - x2 grafikas yra pusapskritimis su spinduliu X12 = 4; X1 = + 2.


2 ir centru koordinai pradioje, esantis virutinje pusploktumje.
Matome, kad X1 sutampa su x2, todl gauname dvi liestini
lygtis:
y = + 3, kai = 2 ir y = - 3x + 3, kai = - 2.
Iekomasis plotas:
61. Sukini tri skaiiavimas
+
^ ~ ^ AOB ^BOC
Kreivalinijin trapecija (apie a Ox)

* =3-3 + j-3' 1 = 6.
A(- 3; 0) O

kfM

Udaviniai. X

1. Apskaiiuokite:
v
Ox = M f 2 M d X
a) J2dx
-3

Kgis
Ats.: 14.

b) J(x5 - 4x 11 - x)dx. y =-H X


-3

A f s . : 0.
jt
2

c) Jsinxcosxdx.
o
Vk = K j ( f - ) 2 d x = ^ ^ _ 1 2
A f s . ; 0,5. ^R H

2. Isprskite lygt:
Rutulys
2
2
a) Jydy = .
O
b) 2 / ( - z)dz =X 2 .
-2

A f s . : a) = \ b) X = + 2.

3. Rasti plot figros, kuri riboja kreivs: y = 2x 2 - 2x + 1,


Vrut = J r ( V W ^ Ydx = 2^ R (R 2 - x 2 )dx =
= O, y = O ir parabols liestin, kuri eina per tak, kurio abscis
X 0 = 2.
= 2TT(R2X-!)| ' = !-R 3
A f s . ; 1,25.
Pvz,
62. Kampai
1. Vienas gretutini kamp lygus 58. Raskite kito kampo
K r a i S ^
didum.
Virn
Sprendimas.
Kito kampo didum paymime x.
Itiestinis kampas Tada + 58 = 180
a ZAOB
= 122.
ZaOb
ZO
Ats.: 122.

2. Dviej krymini kamp suma lygi 130. Raskite kiekvieno


keturi kamp didum.
Gretutiniai kampai
Sprendimas.
Savyb: Z 1 + Z 2 = 180
Kryminiai kampai AOB ir COD lygs,
todl ZCOD = ZAOB = 65.
Tada
Z B O D = ZAOC = 180 - 65 = 115.
Kryminiai kampai

Savyb: Z 1 = Z 2
Ats.: 65; 65; 115; 115.

A
Duota: A S l AP; A M i A N .
Statieji kampai rodyti: ZPAM = ZSAN.
s*/ atstumas
1
1 b rodymas.
n ZPAM = ZPAS - ZMAS
ZSAN = ZNAM - ZMAS
0
(Pagal slyg ZPAS = ZNAM = 90)

ZPAM = 90 - ZMAS
ZSAN = 90 - ZMAS

Todl ZPAM = ZSAN.


63. Kampo pusiaukampin Vidaus vienaali kamp 1 ir 2 suma lygi 180.
Z 1 + Z 2 = 180.
Pagal udavinio slyg:
Z 2 - Z 1 = 30
Tada 2 + Z 1 = 180
+1 Z 2 - Z 1 = 30
2 Z 2 = 210
Z 2 = 105
Z 1 = 180 - 105 = 75
OM - kampo AOB pusiaukamp
Ats.. 75; 105.

Tiesi lygiagretumo poymiai


64. Talio teorema

Trikampio kampai
Z 4 ir Z 6 I Vj c J aus vienaaliai
Z 3 ir Z 5 J
Trikampio vidaus kamp suma Trikampio priekampis
Z 4 ir Z 5 \
vidaus prieiniai
Z 3 ir Z 6 J

Jei Z 3 = Z 6 arba Z 4 = Z5, tai a ||b.


Jei Z 4 + Z 6 = 180 arba Z 3 + Z 5 = 180, tai a ||b.

Pvz.
Z 1 + Z 2 + Z 3 = 180 1) Z 4 = Z 1 + Z 2
Dvi lygiagreios tiess perkirstos treija. Dviej vidaus
2) Z 4 > Z 1
vienaali kamp skirtumas lygus 30. Raskite iuos kampus.
Z4 > Z2
Sprendimas
Trikampi rys Trikampio auktin, pusiaukampin, pusiaukratin

Trikampiai vairiakraiai Lygiaoniai Lygiakraiai

Smailieji

/\
L "
A auktin h pusiaukratin (m) pusiaukampin (I)

Bukieji Pvz,

K .
- 1. Atkarpos AB ir CD kertasi take O, kuris yra kiekvienos
atkarpos vidurys. Raskite BD1 jei AC = 6 m.

Statieji

^ 4 5 4

Sprendimas.
AAOC = ABOD, nes ZAOC = ZBOD;
Trikampi lygumo poymiai < AO =' BO
^ CO = OD.

I trikampi lygybs seka kratini lygyb: AC = BD.


Vadinasi, BD = 6 m.
-
Ats.: 6 m.

", A
Pagal dvi kratines Pagal kratin ir Pagal tris
ir kamp tarp j kampus prie jos kratines

Brinyje Z D A B = ZCBA; ZDBA = ZCAB 1 CA = 13 m.


Raskite DB.
Sprendimas. AB - bendra; Trikampio nelygyb
ZiACB = AADB, nes -s Z D A b = ZCBA
^ ZDBA = ZCAB.
I trikampi lygybs seka kratini lygyb: BD = AC, t.y.
BD = 13 m.

Ats. 13 m.

3. Trikampio ABC kampas A lygus 50, o kampui ACB a-b<c<a+b


g r e t u t i n i s k a m p a s lygus 100. rodykite, kad k a m p o
pusiaukampin lygiagreti kratinei AB.
66. Pusiaukratins savybs
rodymas.
Udavinio slyg pavaizduojame brinyje.
B

CD - kampo pusiaukampin.
Pagal udavinio slyg = 100, tada DCE = 50.
Gavome, kad Z A = ZDCE. AM : MA1 = BM : MB1 = CM : MC1 = 2
Kadangi ZBAC ir ZECD atitinkamieji, tai ABIIDC.

Pusiaukampins savyb
65. Lygiaonio trikampio savybs

CA1 _ AC
A1B - AB

AA 2 = A B - A C - A 1 B - A 1 C
- 143 -
Pvz.:
67. Trikampi panaumo poymiai
1. Lygiaoniame trikampyje ABC (AB = ) nubrta
pusiaukratin BD. Raskite jos ilg, jei AABC perimetras lygus 50
cm, o AABD perimetras lygus 30 cm.

Sprendimas.
Duota: AB = , BD - trikampio ABC
pusiaukratin.
p 5 0 Cm
AABC = ' pAABD = 30 Cm
"
Rasti: BD.

AABC
= AB + BC + CA = 2 (AB + AD) = 50.
Vadinasi, AB + AD = 25.
PAABD = AB + AD + BD.
I ia: 30 = 25 + BD, BD = 5 cm.

Ats.: 5 cm.

2. I vis trikampio virni nubrtos auktins, kuri


kiekviena padalija trikamp du trikampius. rodykite, kad bet kurie
du j, turintys bendr virn su duotuoju, yra panas.

Sprendimas.
Duota: A E l , BD1AC, CFlAB.
rodyti: AAEC ~ ABDC, ACFB - ,
ACFA - ABDA.

rodymas.
A A E C - A B D C , nes
Pagal tris kratines
{ Z C - bendras
ZAEC = ZBDC kaip statieji.

Panaiai rodome ir dviej kit trikampi por panaum.

- 145 -
3. Trikampio ABC pusiaukratin AM lygi pusei kratins S = rp, kur r - brto trikamp apskritimo spindulys.
BC. Kampas tarp AM ir auktins AH lygus 40. Raskite trikampio
ABC kampus. S = , kur R - apibrto apie trikamp apskritimo
spindulys.
Sprendimas.
Panaieji trikampiai
m h
-S= JL2 = _L = - a- P2
H M S, a b,2 C12 -T^ T T

Kadangi A M = BM = MC 1 tai takas M yra centras Panaieji daugiakampiai


apskritimo, apibrto apie AABC. Vadinasi, Z A = 90.
Sakykime, kad AB < AC. _S_ _ a2__bi__i_ _ e2 p2
S, ~ b2 C2 " e2
ZHMA = 50, ZAMC = 130.
Tada Z C = 25, nes lygiaonis. Z B = 65.

Ats.: Z A = 90, Z B = 65, Z C = 25. 69. Lygiakratis trikampis

AB = BC = AC = a
Z A = Z B = ZC = 60
68. Trikampio vidurin linija AA 1 = BB1 = CC1 = h

MNiiBC i2V3" a
h =

MN = -- BC
R = 2r = -f-h = ^

kur R - apibrto apie apskritimo spindulys, r - brto j


apskritimo spindulys.

Trikampio plotas
Pvz.:

Duota: AM = MB, BK = KC
S=Tah a = 4 bh b = i Chc
KTU AB, P a b c = 42.
Rasti: PlMKT"
S = absinC = \ acsinB = \ bcsinA

S = Vp(p - a)(p - b)(p - c)' (Herono formul),


OJ-K-I-O
kur p =
Sprendimas. 70. Statusis trikampis
MK = AC
Pitagoro teorema
KT = AB
c 2 = a2 + b2
.

statinis
MT =


Atvirktin Pitagoro teoremai:
Jei vienos kratins kvadratas lygus
MK + KT + MT = (AC + AB + ) b kit kratini kvadrat sumai, tai
statinis trikampis statusis.
P
1
MKT
=
2
P ABC

P = 2 11
1
MKT OA = OB = OC = R = y AB

.Afs.; 21.

ZL
C

Staiojo trikampio plotas


x
a h/ V
S = ab = I c - h

Sprendimas.
Trikampi ABC ir auktin, nuleista i virns B
X ac - statinio a projekcija ambinje c
pagrind AC, yra ta pati. bc - statinio b projekcija ambinje c
AC-h
\ c
a hA \
Tada -I ifla - = - ^ r = 2- // bc \X.
y AC'" a2 = c -ac
2 C b b2 = c - b c
b ^
saa h2 = aC bC
Vadinasi, -I - = 2.
Statusis su 30 kampu Z C = 90
Ats.: 2. c Z B = 30
b /
b = I
. /BO 0
c O - b2V3" _ a 2 ^ _ V3 2
59 C
6 ' 2 8
3. Duota: ZACB = 90
Pvz.:
C D i AB, AC = 8,
Duota: ZACB = 90 AD = 2.
CElAB Raskite: BD.
Z1 = 2Z2
AC + CE = 3
Raskite: . Sprendimas.
AC 2 = AB -AD,
AC 2 = (AD + DB) -AD,
Sprendimas.
64 = (2 + B D ) - 2
Z 1 + Z 2 = 90
2 + BD = 32
2Z2 + Z 2 = 90
BD = 30.
3Z2 = 90
Z 2 = 30
Ats.: 30.
: CE = - AC => AC = 2CE
4. Dvi trikampio kratins lygios 49 ir 99, o kampas tarp j
59. Koks tai trikampis, smailusis, bukasis ar statusis?
2CE + CE = 3
3CE = 3
Sprendimas.
CE = 1
AC = 49, AB = 99, ZBAC = 59.
I virns B nuleidiame statmen[
BD kratin AC.
Duota: ZACB = 90
BC = 7
Z A = 30.
Raskite: SABC.
Tada AD = AB cosZBAC = 99 -cos59.
Sprendimas.
Kadangi kosinusas intervale [0; ] maja, tai
CB = ^ A B AB = 14

cos59 > cos60 = .


: BC2 + AC 2 = AB 2
49 + AC 2 = 196
Tokiu bdu AD > 49,5 > AC, todl - bukasis (nes
AC 2 = V W
statmens BD pagrindas yra kratins AC tsinyje).
AC = VTiT n = 7 VT.

_ AC CB _ 7V3-7 _ 49"
ABC 2 2 2 Ats.: Bukasis.
A t s . - . ^ - (kv. vienet).
71. Daugiakampis B a C

n - kampio vidaus kamp suma lygi 180(n - 2), kur


n - kratini skaiius.

n(n 3)
Ikiliojo n - kampio striaini skaiius lygus 2 .

Pvz,
Raskite bet kokios penkiakamps vaigds vidaus kamp
sum.
S = ah a = bh.b

S = absint)

S = dusina

d; + d22 = 2a2 + 2b2


Sprendimas.
Pagal trikampio priekampio savyb: Pvz,
Z 6 = Z 1 + Z4;
Z 7 = Z 2 + Z5.
Kadangi Z 7 + Z 6 + Z 3 = 180, Duota: BE I A D
tai Z 1 + Z 2 + Z 3 + Z4 + Z 5 = 180 ZABC = 150
P a b c o = 24, BE = 2.
Raskite: .

Sprendimas.
Atskiri keturkampi atvejai ZABE = 60, Z A = 30.
Tada AB = 2BE = 4,
72. Lygiagretainis 2AB + 2BC = 24
AB + BC = 12,
Lygiagretainiu vadinamas BC = 12 - 4 = 8.
keturkampis, kurio:
AB = CD BO = OD Ats.: 8.
BC = AD AO = OC
Z A + Z B = 180
Pvz.
Duota: Z A = 30, AB : = 3 : 7
= 120. Duota: AC = 20
Raskite: SABCD - BD = 10
Raskite: S.

E D

Sprendimas.
AB = , = 7
2( + ) = P,
20 = 120 Sprendimas.
= 6 S = ^ - A C -BD

Tada AB = 18, = 42. S = ~ 2 0 - 1 0 = 100.

BE = AB = 9 Ats.: 100 kv. vienet.

S = A D - B E = 4 2 - 9 = 378. 2. c Duota: ZBAD = 30c


PABCD = 2 4
Pa,
Ats.: 378 (kv. vienetai). Raskite: S.

73. Rombas

Rombu vadinamas lygiagretainis, kurio visos kratins

Sprendimas.
d , l d2 (rombo striains AB = BC = CD = AD = 24 : 4 = 6.
tarpusavyje statmenos)
S = AB 2 sin30 = 36 . = 18.
ZDCA = ZACB (rombo striains
kampus dalija pusiau) Ats.: 18 kv. vienet.

S = ha = a 2 sinA = y d,d 2
1. Kai takas staiakampio viduje.
74. Staiakampis
Per tak P ivedame tieses,
Staiakampiu vadinamas lygiagretainis, kurio visi kampai lygiagreias staiakampio
stats. kratinms.
AB = BD = d Susidariusius atstumus iki
d2 = a 2 + b2 kratini paymime a, b, c, d.

S = ab = j d 2 sina
BP| 2 = a 2 + c2, IPD] 2 = b2 + d2
IBPl 2 + |PD| 2 = a 2 + b2 + c2 + d2.
Pvz.:
Analogikai:
1. Duota: Z1 = Z2 IAP| 2 = b 2 + c2, |CP| 2 = a2 + d2,
BM = 5 I AP| 2 +|CP| 2 = a 2 + b2 + c2 + d2.
MC = 4
Raskite: S. 2. Kai takas yra staiakampio kratinje.
c
Per tak P ivedame ties,
lygiagrei staiakampio
kratinei.
Sprendimas.
Z 2 = ZBMA, todl lygiaonis ir
AB = BM = 5
BC = BM + MC = 9 I PBl 2 = a2, |PD|2 = b 2 + d 2
S = A B - B C = 5 - 9 = 45 |PB| 2 +|PD| 2 = a2 + b2 + d 2
ir
Ats.: 45 kv. vienetai. |PA|2 = b2, I PC| 2 = d 2 + a2;
\PA\2 + I PCl 2 = b2 + a2 + d 2
2. Ploktumoje duoti staiakampis ir takas. Parodyti, kad
atstum kvadrat sumos nuo tako iki prieing staiakampio 3. Kai takas alia staiakampio.
virni lygios.

Sprendimas.
Inagrinsime 3 atvejus.
1ra| 2 = 2 + 2 ,
2. Kiekviena kvadrato virn sujungta su tarp kit dviej
2 2 2 virni esanios atkarpos viduriu. rodykite, kad centre gausime
|| = + d
kvadrat, kurio plotas penkis kartus maesnis u pradinio kvadrato
j PAl 2 + I | 2 = 2 + b2 + 2 + d 2
plot.
ir 1 R B | 2 = 2 + 2 , I P D l 2 = b 2 + d 2
I p b I 2 + | p d | 2 = a2 + b2 + c2 + d 2 1. Tegul AB = a, tai BC = - .

75. Kvadratas 2. Kadangi AK = CD ir jos


lygiagreios, tai
Kvadratu vadinamas staiakampis, kurio visos kratins
lygios. figra ACDK - lygiagretainis, kurio

d = aV J plotas AK- AB = - .
3. AKDM = AAMF (Z1 = Z2; Z 3 = Z4).
S = a2 = y d 2 Z 2 + Z 4 = Z 3 + Z 1 = 90.
Vadinasi, keturkampis centre - lygiagretainis, staiakampis
ir kvadratas.

4. S a c d k = PQ -AC
Duota: BE = EC
2
Raskite: Sabcd : Sabe . PQ = ^ = a . aV5* _ 2a _ a
_
2 2a\5 V5-

S kvadrato centre = P Q 2 =
5 '

76. Trapecija
Vbcd = AB 2
Trapecija vadinamas keturkampis, kurio dvi kratins
BE = -- BC
lygiagreios, o kitos dvi nelygiagreios.
1
3
ABE
- AB y AB = AB 2

SABCD : S abe = A B 2 : AB 2

Ats.: 4.
BC ir AD - virutinis ir apatinis Sprendimas.
pagrindai. BC HAD. ED = AD-BC _ 17-9
=_ 4
2 2
AB ir CD - onins kratins
AC ir BD - striains ACED - lygiaonis, todl h = ED = 4.
MN - vidurin linija
_ BC|AD.h= 1 7 ^ . 4 = 52
S
MN = ^ ( B C + AD), MN Il BC
Ats.: 52 kv. vienetai.
S = h = { h(BC + AD)
2. Ant trapecijos ABCD pagrind AD ir BC trapecijos iorje
Trapecija lygiaon, jei AB = CD. nubrti kvadratai ADEF ir BCGH. Trapecijos striains kertasi take
Z 1 = Z2, Z3 = Z4
O. Raskite AD 1 jei BC = 2, GO = 7, GF = 18.
AE = FD
Sprendimas.
Lygiaons trapecijos, kurios 1. ACOB - A A O D :
striains statmenos viena kitai, H G
plotas S lygus jos auktins h AO - AD
OC BC
kvadratui, t.y. S = h2.
2. ZOAF = ZOAD + 90 =
= ZOCB + 90 = ZOCG,
Trapecija staioji, jei BA1AD. todl AOGC ~ AOFA.

I ia seka, kad
ZAOF = ZCOG,
todl takas O yra tiess FG
takas ir

AO _ AD - AF _ u
OC BC CG

OF = 18 - 7 = 11 => k - -OG_ -
OF
Jl
7
AD = k BC;
Duota: AB = CD
AD = 17 => AD = 2 -- - - -
ZD = 45
BC = 9 Ats. 22
Raskite: S ABCD .

D
77. Apskritimas Kadangi C = m, tai m = 2kR arba R 2 ir skritulio plotas

R - spindulys S = KR2 = T r O 2 = ^ = 4
^
.f. D - skersmuo
D = 2R Ats.:^f.4
O skersmuo)

2. Lygiakraio trikampio ABC kratin - a . I dviej virni


briami apskritimai spinduliu --. Raskite ubrkniuotos dalies
plot.
Apskritimo ilgis C = 2n:R
C =
Skritulio plotas S = TTR2

u A C B = ( - lanko ACB
Iaipsninis matas)

lanko ACB ilgis I =- 180


Sprendimas.
_ Va2
ABC

Ipjov AKM ir CML plotai lygs = -g- .


g _ nR24"
ls
PJ- 360
V3a2
Iekomasis plotas S = 2- - g - = -^IT- f-).
_ 2
S nuopj. = 4
3
b h = Sispjov.
.
+ S a
Ats.: -f (VT-f).

3. Duotas staiakampis ABCD. Apskritimas su spinduliu 2


lieia kratines AB ir AD. Antrasis apskritimas su spinduliu 1,4 lieia
pirmj apskritim ir kratines , CD. AB = 5. Raskite staiakampio
antrj kratin.
Pvz,
Sprendimas.
1. Raskite plot skritulio, jei apskritimo ilgis lygus m. O 1 E = r, = 2
O 2 F = F2 = 1,4
Sprendimas. O 2 K = AB - F1 - F2 =
C = 2tcR. = 5 - 3,4 = 1,6
O 1 O 2 = T1 + 2 = 3,4

O 1 K = V(r, + r 2 ) 2 - O 2 K 2 ' = /11,56 - 2,56 = 3


AD = 2 + 3 + 1,4 = 6,4. ZABC = 90

Ats.: 6,4.

Pvz.:
78. Apskritimas ir kampai
Takai A, B ir C yra apskritimo su
centru O takai. ZABC = 60.
Kampas, kurio virn yra apskritimo
Raskite ZAOC.
] centras, vadinamas jo centriniu
kampu
Sprendimas.
zAOB = uAmB
ZABC brtinis.

ZABC = ZAOC

Z A O C = 2 ZAB C
Z A O C = 120.
Kampas, kurio virn priklauso
apskritimui, o jo kratins yra
apskritimo stygos, vadinamas
brtiniu.
Takai A, B ir C yra apskrtimo takai.
ZABC _ 1
uAmC ZABC = 30, apskritimo skersmuo
lygus 10 cm.
_ 1 Raskite AC.
ZABC ZAOC

Sprendimas.
ZABC = ^ZAOC.
ZBAC = Z C P B = ^-uBMC
Vadinasi, ZAOC = 60, AAOC - lygiakratis, todl

AC = R = j - 10 = 5 cm.

Ats.: 5 cm.
Sprendimas.
79. Apskritimas ir ties AE-BE = ED-CE
kirstin
5- 2 = 2,5 -ED => ED = ^ - = 4 (cm).
OMla

Ats.: 4 cm.
liestin
80. brtinis ir apibrtinis apskritimas

brtas trikamp
AM = AN
AO - ZMAN - pusiaukampin r = , kur

a + b +c
P =

S - trikampio plotas

brtas keturkamp
AM 2 = A B - A C = A E - A D =
= AO 2 - R 2 c. _ ,c

a + c = b + d

a* b = c - d
Apibrtas apie trikamp

_ abc
R 4S

Pvz. S - trikampio plotas


a, b, c - trikampio kratins
Stygos AB ir CD kertasi take E. Raskite ED1 jei AE = 5 cm,
BE = 2 cm, CE = 2,5 cm.
Apibrtas apie keturkamp Sprendimas.

OF = OE = r = 2; AO = OB = R
Trikampis OFB statusis, kurio
Z O B F = 30

OF = ^ O B (statusis prie 30 kamp)

OB = 4.

Pastaba: 2. Duota: trikampis ABC, AB = BC, AC = 24, R = 13.


Lygiaons trapecijos, kuri galima brti apskritim, Raskite S.
auktin lygi pagrind geometriniam vidurkiui.
a Sprendimas.

AC 24
sinp = 2R sinp = 26
12
b = 13
cosp = V1 - 32 ,
A D
cosp = 13
81. Sinus, kosinus teoremos
1 - cosp = 2 s i n 2 1 ; sin f - = T ^ f p T ;
B a _ b _ c - op
sina sin(p
sin
2~_)?r - W
kur R - apibrto apie trikamp
apskritimo spindulys Trikampis BDC:--= Sin L2 ^ B C = - I J2- = 1 = 6

c2 = a2 + b 2 - 2abcos(p S A B C = A B - B C - s i n p = --- - =216.

Pvz.: Ats.: 216.

1. Duota: trikampis ABC, BC = AC = AB, brto apskritimo Duota: SABE = 4SBEC


spindulys r = 2. Raskite R. BE = 1, DE = 16.
Raskite: AC.
Sprendimas. 4. Trikampi p a n a u m o koeficientas lygus 2. Vieno j
perimetras lygus 12 cm. Raskite panaaus j didesniojo trikampio
I to, kad S a b e = 4S BEC seka, kad AE = 4CE. perimetr ir j plot santyk.
Sakykime, CE = x, tada
AE = 4x, A E - E C = B E - E D , Ats.: 24 cm; 4.
4x 2 = 16
= 2. 5. Staiojo trikampio smailieji kampai sutinka kaip 1 : 2.
Raskite maesniojo statinio ir ambins santyk.
Vadinasi, AE = 8 ir AC = 8 + 2 = 10.
Ats.: .
Ats.: 10.
6. Kok kamp sudaro tarp savs laikrodio rodykls, kai jis
82. Taisyklingojo n - kampio plotas
rodo 8 valand?

A 1 A 2 = A 2 A 3 = ... = A A 1 = a
Ats, 120.
P = a - n - perimetras
R - apibrto apskritimo spindulys
7. Trikampio kratins, sudaranios bukj kamp, lygios
r - brto apskritimo spindulys
6 cm ir 8 cm. Kokias sveikas reikmes gali gyti treioji kratin?
1
o
S = 2 Ats, 3; 4; 5; 6; 7; 8; 9; 10; 11; 12; 13.
2
O nR . 2
S = -- Sin-pp
8. Kiek simetrijos ai turi lygiaon trapecija?
2 2
C _ 6R . 2 _ ^
S s i n
6 - T ' ^ - - T - Ats, 1.

Udaviniai.
9. Staiakampio striain lygi 10 cm, o jo plotis 6 cm. Raskite
staiakampio perimetr.
1. Apskritimo styga lygi spinduliui. Raskite kamp tarp j.

Ats, 28 cm.
Ats, 60.

10. Lygiagretainio kratini santykis 3 : 2. Kiek procent jo


2. Staiakampio plotas 60 cm 2 , o vienos kratins ilgis
perimetro sudaro trumpesnioji kratin?
cm. Raskite staiakampio perimetr.

Ats, 20%.
Ats.: 34 cm.

3. Kiek kvadratas turi simetrijos ai?

Ats.: Keturias.
83. Stereometrija
Dvi tiess, turinios vien bendr tak,
vadinamos susikertaniomis. IVr^^h
Pagrindins aksiomos ir apibrimai
an b = M

Per tris bet kuriuos takus, nesanius


vienoje tiesje, eina viena ir tik viena
ploktuma Dvi vienos ploktumos tiess, neturin-
ios bendr tak, vadinamos lygiag-
reiomis.

Jei du tiess takai yra ploktumoje, tai a 11 b => (a c a ; b c a )


ir visa ties yra toje ploktumoje

Dvi tiess, nesanios vienoje ploktu-


moje, vadinamos prasilenkianiomis.
A e a, B e a
A e a, B e }
=> c a
a ir b prasilenkianios

Jei dvi ploktumos turi bendr tak, tai


jos kertasi tiese, einania per t tak. Ties, kurios visi takai priklauso
ploktumai, vadinama ploktumos

/

/ tiese.

ivV

M e , M e
Ties kerta ploktum, jei jos turi tik
a n = vien bendr tak.
4 M e

an = M
Ties vadinama lygiagreia ploktumai, Per dvi susikertanias tieses eina viena
jei jos neturi bendr tak. ir tik viena ploktuma.

all , a|| a an b = M
{
b

Dvi ploktumos, neturinios bendr Per dvi lygiagreias tieses eina viena ir
tak, vadinamos lygiagreiomis. tik viena ploktuma.

oc 11 all b
{
b

Ties vadinama statmena ploktumai, Jei viena lygiagrei tiesi kerta


jei statmena bet kuriai ploktumos ploktum, tai ir kita ties kerta t
tiesei. ploktum.

T~~t
all
a l l b
a l a => ( a l e ; a 1 b) U b n a
an a J

84. Stereometrijos teoremos


Per ties ir alia jos esant tak eina
viena ir tik viena ploktuma.

M ^ j - a
LMea
P l o k t u m lygiagretumo poymis
85. Lygiagretumo poymiai
Jei vienos ploktumos dvi susiker-
Tiesi lygiagretumo poymiai tanios tiess atitinkamai lygiagreios
kitos ploktumos dviem susikertan-
Jei dvi tiess lygiagreios treiai, tai jos ioms tiesms, tai tos ploktumos
lygiagreios tarp savs. lygiagreios.

an b = M

a Il c
b Il c } a Il b
, b c a
alia,; bllb 1
a, c , ^ } all

Jei dvi ploktumos lygiagreios treiajai,


tai jos lygiagreios tarp savs.
Tiess lygiagretumo ploktumai
poymis

Jei ties, esanti alia ploktumos,


lygiagreti bet kokiai ploktumos tiesei,
tai ji lygiagreti iai ploktumai.
aHb
a
bcaj all
!I Y }

Jei viena susikertani ploktum eina Jei ploktuma kerta dvi lygiagreias
per ties, lygiagrei kitai ploktumai, ploktumas, tai susikirtimo tiess
tai ploktum susikirtimo linija lygiagreti lygiagreios.
tai tiesei.

an = b
=
all } => a Il b = b
Y n a = b

all
K => a II b
J
86. Tiess statmenumo ploktumai poymis
a) Jei viena lygiagrei tiesi statmena
Jei ties statmena dviem ploktumos
ploktumai, tai ir kita ties statmena iai
susikertaniomis tiesmis, tai ji statmena
ploktumai.
paiai ploktumai.
b) Jei dvi tiess statmenos vienai
ploktumai, tai tos tiess lygiagreios.

a) a K b
an b
}
}
bia
, b ala
cla
cla
clb b) a I aa
a! I b
bl a J

a) Jei ties statmena vienai lygiagrei


ploktum, tai ji statmena ir kitai. 87. Trij statmen teorema

b) Jei dvi ploktumos statmenos vienai Jei pasvirosios projekcija ploktumoje


ir tai paiai tiesei, tai tos ploktumos statmena ploktumos tiesei, tai ir pati
lygiagreios. pasviroji statmena tai tiesei.

Atvirktin.
Jei pasviroji statmena ploktumos tiesei,
tai ir jos projekcija ploktumoje stat-
mena tai tiesei.

a) cx|| ala

}
ala bna
bla . cj_m<i=>
b) a l laa \ . c = "pra b
Pla m c a

<s=> b l m
88. Ploktum statmenumo poymis 2. Lygiaonio trikampio ABC pagrindas BC lygus 12 cm,
onin kratin - 10 cm. I virns A ikeltas 6 cm ilgio statmuo
Jei ploktuma eina per statmen kit AD trikampio ABC ploktumai. Raskite tako D atstum iki kratins
ploktum, tai ji statmena tai ploktu- BC.
mai.
Sprendimas.
D AH - trikampio ABC pusiaukampin,
auktin ir pusiaukratin.
Kadangi B C l , tai B C l D H (trij
statmen teorema).


3 } => Vadinasi, DH - iekomasis atstumas.
AADH: DH2 = AD 2 + AH 2
Pvz.:
AACH: AH 2 = AC 2 - CH2 = 102 - 6 2 = 64.
Tada DH2 = 62 + 64 = 100, DH = 10 cm.
1. rodykite, kad erdvinio keturkampio kratinividuritakai
yra lygiagretainio virns.
Ats.: 10 cm.
rodymas.
Sakykime, kad ABCD - duotasis
erdvinis keturkampis (keturkampio 89. Kampas tarp tiess ir ploktumos
virns nra vienoje ploktumoje).
A1, B1, C 1 ir D 1 - jo kratini viduri Ties vadinama statmena ploktumai,
takai. A 1 B 1 - trikampio ABC vidurin jei statmena bet kuriai ploktumos tiesei.
linija, lygiagreti kratinei AC; C 1 D 1 -
trikampio ACD vidurin linija taip pat Kampu tarp tiess ir ploktumos vadi-
lygiagreti kratinei AC. namas kampas tarp tiess ir jos projek-
cijos toje ploktumoje.
Vadinasi, A1B11| C 1 D 1 ir yra vienos ploktumos tiess.
Tokiu pat bdu [rodome IiesiuA 1 D 1 ir B1C1 lygiagretum. Pastaba.
Vadinasi, keturkampis A 1 B 1 C 1 D 1 yra vienoje ploktumoje ir Kampu tarp susikertanij tiesi a ir b
vadinamas maiausias i susidariusij m
jo prieingos kratins lygiagreios. Aiku, keturkampis A 1 B 1 C 1 D 1 -
kamp (0 < Z(a, b) < 90). a
lygiagretainis.
Z(AB, ) = ZA 0 BA =
Pvz, Sprendimas.

1. Staiojo trikampio ABC statiniai CA ir CB lygs 6 cm ir 8 Briame CE1AB.


cm. Per staiojo kampo C virn nubrta ploktuma lygiagreti Kadangi D B 1 (ABC), tai DB1CE.
CElAB
AB. Maesnysis trikampio statinis sudaro su ia ploktuma 45
kamp. Raskite kampo, kur sudaro antrasis statinis su ia CElDB } C E l pi. AADB => ZDEC = 90.

ploktuma, sinus.
Vadinasi, pasvirosios DC projekcija ploktumoje ADB yra
DE.
BB1 = AA 1 Iekomasis kampas yra ZCDE.
Trikampis AA 1 C - statusis
CE
tgCDE = DE
AC = sin45
Sakykime, AB = a, tada CE = f

ADBA : DB = AB = a.
7
AA 1 = 3V2
BB1 = AA 1 <2 ADBE : DE = VDB2 + EB2 = Ik 2 + aV?

_ 1 V?
Trikampis BB1C - statusis. ADECitgDEC = - ^ - = ^ r
aVif VF 5
2
SinBCB 1 =-- Ats: V?

SinBCB1 =
3V? 90. Dvisienis kampas

3 V? Dvisieniu kampu vadinamas dviej


Ats,
nelygiagrei pusploktumi susikirti-
Duota: AABC, mas.
ZACB = 90
AC = BC Dvisienio kampo tiesiniu kampu vadina-
DBl(ABC) mas kampas tarp dviej s t a t m e n
ZDAB = 45 dvisienio kampo briaunai. a c ^
Raskite: Kampo tarp tiess b c ,' b I A B
DC ir ploktumos an b = O
ADB tangent. Z(a, b) - tiesinis kampas
Pvz.
Lygiaonio bukojo trikampio ADM (ZA = 120) ir rombo
ABCD su kratine a ir kampu ACD, lygiu 60, ploktumos statmenos.
Raskite atstum MB.
91. Briaunainiai
n - kampe prizme vadinamas briaunai- virutinis pagrindas
nis, kurio dvi sienos - lygs n - kampiai
su atitinkamai lygiagreiomis kratin-
mis, o likusios n sien - lygiagretainiai

ZBKA = Z M K A = 90 (pagal dvisienio kampo brim)


AM = AB
ZKAM = 180 - 120 = 60 ir apatinis pagrindas
ZKAB = 180 - 120 = 60,
cell
Vadinasi, ZKAM = ZKAB.
ABHA 1 B 1
BCIlB 1 C 1
= ABAK, nes f BA = MA,
CDIIC 1 D 1
I ZBAK = ZMAK;
DAIID 1 A 1
-J? AA1H BB 1 IICC 1 II DD1
I ia BK = MK1 o M K = asin60 = ^ f - .
AA1 - onin briauna
BD1 - striain
ABMK statusis, ZBMK = 90, (nes duotosios ploktumos
v3
statmenos), ir lygiaonis - BK = BM = -g.
Staioji prizm
: MB = VBK2 + KM2 = ^ - 2 + ( # ) 2
= ~
AA1I(ABC)

S.son. = P pagr. - H

S visas = S .son + 2 S pagr.

V = -I-S H
3 pagr.

Taisyklingja prizme vadinama tokia staioji prizm, kurios


pagrindas yra taisyklingasis daugiakampis (t.y. toks daugiakampis,
kurio visi kampai ir visos kratins lygios).
Gretasienis Pvz.:

Gretasieniu vadinama prizm, kurios 1. Staiosios prizms pagrindas - lygiaon trapecija, kurios
pagrindai - lygiagretainiai. vienas pagrindas du kartus didesnis u kit. Nelygiagreios prizms
onins sienos - kvadratai. Prizms auktin lygi 6 cm. Jos oninio
paviriaus plotas lygus 144 cm 2 . Raskite prizms tr.

Statusis gretasienis

Staiuoju gretasieniu vadinamas greta-


sienis, kurio onin briauna statmena
/D1
pagrindo ploktumai. --v...
B

h /
ABCD - lygiagretainis S.son. = P pagr. H = (6 + 6 + + 2x)' 6
v

AA1I(ABC) (12 + 3)6 = 144


AC 1 = A1C = d,; 12 + 3x = 24
BD1 = B1D = d2 = 4.
d, ^ d 2 AD 1 = 2.

AABD 11: BD11 = VAB2 - ADzIy = = V32~.


Staiakampiu gretasieniu vadinamas
statusis gretasienis, kurio pagrindas -
staiakampis. A1 \
-4d
n,
/ Spagr. = ^ r2^ - V 3 2 = 24<2 .

V = 24V2 6 = 144V27.
R
/
A b D
/ Ats.: 144V2cm 3 .

ABCD - staiakampis 2. Staiakampyje gretasienyje per apatinio pagrindo


AA11(ABC) briaun ir prie j esanios sienos striaini susikirtimo tak
nubrtas pjvis. Kokiu santykiu to pjvio ploktuma dalija
cP = a 2 + b 2 + c2
gretasienio tr?
S pavi...
rsiaus = 2(ab
V + ac + bc)'
V = abc
Sprendimas. Pwz.
K = AB 1 n A 1 B
. Kubo briauna lygi a. Raskite kubo striaininio pjvio plot.
MN eina per tak K ir

N
/ MN IICD
MNCD - iekomasis pjvis.
MK = KN (nes staiakampio
Sprendimas.
AADC:
B1 C.
> K A C = VAD 2 + DC 2 '

/
striains susikirtimo take
B
dalijasi pusiau). AC = Va 2 + a 2

r/
MNHA 1 B 1 AC = a V ?
MK - trikampio AA 1 B vidurin
B
D linija, todl MK = ^ A A 1 . Jc S ACCiAi = A C - A A 1 = a - a V 2 = a 2 V2\

Sakykime, V - staiakampio gretasienio ABCDA 1 B 1 C 1 D 1 turis, A D


V 1 - prizms AMDBNC tris.
Ats.: a 2 V 2 .
V = AD -AA 1 - CD = AD -2AM- CD = 4 - y - A D - A M - CD =
93. Piramid
= 4 S a d m - C D = 4V 1
Piramide vadinamas briaunainis, kurio
Vadinasi, gretasienio dali tri santykis lygus 1 : 4. viena siena yra bet koks daugiakam-
pis, o kitos sienos - trikampiai, turintys
Ats.: 1 : 4 . bendr virn.

son = S1 + S2 + .... Sn,


92. Kubas S 1 , S 2 , . . . , S n - piramids onini
sien plotai
Staiakampis gretasienis su lygiomis
briaunomis vadinamas kubu. S visos = S .son. + Spagr.
S - piramids virn
SA - onin briauna
V = a3 v 43 - s pagr.
-H
ASAB - onin siena
\d ABCD - piramids
S = 6a2 pagrindas
H - piramids auktin
d = aV^ Hl(ABC)
Piramid, kurios pagrin- Pvz,
Jei piramids visos onins briaunos
lygios, tai virns projekcija - apie das - taisyklingasis
daugiakampis, o pirami- 1. Taisyklingosios keturkamps piramids auktin lygi 6
pagrind apibrto apskritimo
ds virns projekcija - cm ir sudaro su onine siena 30 kamp. Raskite piramids tr.
centras.
to daugiakampio cent-
ras, vadinama taisyklin- Sprendimas.
gja s SE - onins sienos auktin,
Jei piramids visi dvisieniai kampai
prie pagrindo lygs, tai virns OP - trikampio SOE auktin.
projekcija - pagrind brto apskri- on. cosa '
Kadangi B C l S R t a i B C i O P
timo centras. (tiesi statmen teorema)
kur a - onins sienos
pasvirimo pagrind
kampas

S.son. = k - p ,
r

kur k - onins sienos


auktin, p - pagrindo Vadinasi, ZOSE - kampas tarp SO ir onins sienos BSC.
pusperimetris. ASOE: OE = S0tg30 = 2ViT,
BC = 2 0 E = 4V3".

V = ( 4 / 3 ) 2 - 6 = 96.
94. Nupjautin piramid
Ats, 96 cm3.
Nupjautine piramide vadinama pilnos
p i r a m i d s dalis tarp pagrindo ir j 2. Takas E yra alia ploktumos , o takai C, F ir D
k e r t a n i o s l y g i a g r e i o s pagrindui ploktumoje . Duota, kad EFl CF, CE = DE = 5, CF = 3, ZFCD =
ploktumos. = Z F D C = 30.
1) Parodykite, kad trikampio CFD plotas S .
ABCDEA 1 B 1 C 1 D 1 E 1 - nupjautin 2) rodykite, kad atkarpa EF statmena ploktumai .
piramid. 3) Apskaiiuokite piramids CDEF tr.
E.
V = ^ - H ( S + VS-S 1 + S1) Sprendimas.

S ir S 1 - pagrind plotai,
H - auktin
1) Trikampis CFD lygiaonis, nes Z F C D = Z F D C = 30. 3. Piramids pagrindas - rombas su 30 cm ir 40 cm
Tada CF = DF., striainmis. Piramids virn nutolusi nuo pagrindo kratini 13
Z C F D = 180 - (30 + 30) = 120. cm atstumu. Raskite piramids auktin.
O _ CF*DF*sin120 _ 3'3'Sin60 _ *
Ats.: 5 cm.
i5
CFD 2 2 4

4. Trikampje piramidje nubrta ploktuma per pagrindo


2) AEFC: EF = VCE2 - CF 2 = V25 - 9 = 4. trikampio vidurin linij ir piramids virn. Kokiu santykiu pjvio
Trikampis EFD statusis, nes DE = 5, EF = 4, ploktuma dalija piramids tr?
DF= 3, o 5 2 = 4 2 + 3 2 . Ats.: 1 : 3.
Vadinasi, E F I D F .
Gavome: E F l C F j ^ E F l a (pagal t i e s s 5. Taisyklingosios trikamps prizms oninio paviriaus
E F l D F J s t a t m e n u m o ploktumai iklotin yra staiakampis su 15 cm ir 12 cm kratinmis. Raskite
poym) prizms tr. (Du atvejai).

3) V = | S C D F .EF Ats.: 75/ cm 3 ; 60V3cm 3 .

V = 4 = 3^3 7 . 6. Taisyklingosios trikamps piramids auktin lygi 12 cm,


o apskritimo, brto jos pagrind, spindulys lygus 5 cm. Raskite:
Ats.: 3V3\ a) onins sienos auktin, nubrt piramids pagrindo
briaun;
Udaviniai. b) kampo tarp onins sienos ir pagrindo ploktumos
tangent;
1. Kubo ABCDA 1 B 1 C 1 D 1 briauna lygi a. K - briaunos AB c) piramids pilno paviriaus plot.
vidurio takas. E - briaunos DD1 vidurio takas. Raskite trikampio
A 1 KE perimetr ir nustatykite, kokiu santykiu ploktuma, einanti per Ats.: a) 13 cm; b) 1,2; c) 270V3W.
takus A 1 , K ir E, dalija kubo tr.

Ats.: a(V5 + ) ; ^ .

2. Staiakampio gretasienio striain sudaro su pagrindo


ploktuma 45 kamp, o onins sienos striain - 60 kamp.
Staiakampio gretasienio auktin lygi 8 cm. Raskite jo tr.
95. Sukiniai Kgio iklotin - skritulio ipjova ir B
skritulys
Ritinys
Iklotins lanko BCB 1 ilgis lygus 2kRH ^

M
S.son. = 2KRH = 2nRl / o, oninio paviriaus plotas J
S.son. = nRt
= 2rcRH + 2nR2 = >1 >I
A
= 2kH(R + H) Pilno paviriaus plotas
H
I Svisas = nRU + R)'

V = S pagr. - H = nR 2 H -JT
/ O2
A Kgio tris
I" V = ^kR2H
IV
97. Nupjautinis kgis
Skrituliai su centrais O1 ir
O2 - ritinio pagrindai.
AB - sudaromoji (AB = t) r ir R - pagrind spinduliai
H - ritinio auktin (H = t) AB = t - sudaromoji
Staiakampis M P S T - a - sudaromosios pasvirimo pagrindo
ainis pjvis. ploktum kampas
O 1 O 2 = H - auktin
R
Ritinio iklotin - staiakampis ir du Son = nt(r + R)
skrituliai.

2KR S . = n(lr + R) + kR2 + 2


4
VISOS '

R V = ^- TtH(R2 + rR + r2)

96. Kgis 98. Rutulys


Takas S - kgio virn
CD - sferos skersmuo
SA = t - sudaromoji
AM = MB =s> O M l A B
SO = H - kgio auktin
OA = R - pagrindo spindulys Rutulio paviriaus plotas
BC = 2R - pagrindo skersmuo S = 4nR2
ASBC - ainis pjvis
ZSBO = - sudaromosios pasvirimo Rutulio tris
pagrindo ploktum kampas. V = ^
Rutulio n u o p j o v a Pvz.:

H - nuopjovos auktin 1. rodykite, kad jeigu trij rutuli spinduliai sutinka kaip
r - nuopjovos pagrindo spindulys 1 : 2 : 3, tai didiausio rutulio tris 3 kartus didesnis u dviej
r = VH(2R - H)' maesnij rutuli tri sum.

Rutulio nuopjovos paviriaus plotas rodymas.


S = 2rcRH = (2 + H 2 ) Sakykime, R 1 - maiausiojo duotj rutuli spindulys. Tada
antrojo ir treiojo rutuli spinduliai 2R 1 ir 3 R r
Rutulio nuopjovos tris
V = nH2(R-|H) Tada trij rutuli triai:

Rutulio s l u o k s n i s V1 = A l t R 3 ; V2 = A 7 ^ R 1 ) 3 ; V3 = ITt(SR1)3

I ia 3(V, + V 2 ) = yJt(3R; + 24R 1 3 ) = - n 27F, =


H - sluoksnio auktin
R - rutulio spindulys
= I ( S R 1 ) 3 = V3.
r, ir r 2 - pagrind spinduliai
2. Kgio auktin lygi 20 cm, atstumas nuo pagrindo centro
S = 2kRH
iki sudaromosios lygus 12 cm. Raskite kgio tr.

V = | (3 + Sr22 + H 2 )
Sprendimas.
OKiSN
OK = 12 cm
ASKO : SO 2 = SK 2 + KO 2
Rutulio ipjova I ia SK = 16
ASKO - ASON, nes Z K S O bendras
H - nuopjovos auktin ir Z S K O = Z S O N = 90.
r - nuopjovos pagrindo spindulys
R - rutulio spindulys I panaumo seka:

SK _ _KO_ t l J6_ _12_ .1


S = nR(2H + r) SO ON > " 20 ON

W _ 2kR2H I ia O N = 15.

Tada V = ^ R 2
H = ^ j i O N 2 - S O = - y 2 2 5 20 = 1500.

Ats.: 1500 cm 3 .
3. Apvalios varins 2 mm storio vielos ritinio mas - 3,52 kg. 5. Drenao vamzdio, kurio skerspjvis pavaizduotas
Koks vielos ilgis metrais, jei vario vieno kubinio decimetro mas - paveiksle, vidinis skersmuo 12 cm. Kiek vandens nuteks iuo
8,8 kg. Atsakym suapvalinkite iki deimi metr. vamzdiu per 1 valand, jei srovs greitis 0,4 m/s ir jei vanduo
uima pus vamzdio skerspjvio.
Sprendimas. Laikykite = 3,14.
Atsakym pateikite
Surandame vielos ritinio tr: kubiniais metrais, suapvalin
3,52 : 8,8 = 0,4 (dm3) iki vienet.
0,4 dm3 = 0,0004 m 3

R = 1 mm = 0,001 m
V = Tir2H
0,0004 = 3,14-(0,001) 2 -H, Sprendimas.

_ 0,0004

Vandens skerspjvio spindulys r = 0,06 m.
3,14-0,000001

H = 127,389 = 130 m Vandens skerspjvio plotas lygus:

Ats.: 130 m. S =
2 =
' 3 14
' ' ( ' 0 6 ) 2 = . 0 5 6 5 2 ( m 2 )

4. Kiek 2 cm skersmens rutuliukgalima nulieti i metalinio 0,4L = = 1440-?-


kubo su 4 cm ilgio briauna? ' s 1 h n
3600
Vandens, nutekanio per 1 valand, tris
Sprendimas. V = 0,005652-1440 = 8,1388 - 8 (m3).

Kubo tris 43 = 64. Ats.: 8 m3.

Vieno rutuliuko tris y i i ' 1 3 = - | - . 6. taisyklingjtrikamp prizm brtas ritinys. Ritinio tris
lygus V. Raskite prizms tr.
4 _ 64 _ 48
V, ~ 4 -
3 Sprendimas.
48

rutuliuk.
Kadangi 15 < < 16, tai galima nulieti 15 toki

A

/
Ritinio ir prizms auktins
lygios H.
Prizms pagrindo kratin
Ats.: 15. I paymime x, tada ritinio

// 4
- - pagrindo spindulys ~ .
Randame prizms ir ritinio tri santyk: 5. Ritinio ainio pjvio plotas lygus 20 cm2. Raskitejo oninio
paviriaus plot.
v l r " (|)2 Ats.: 20 cm 2 .
V X2H
Vritinio X' TrH
r J m I 6. Kgio pagrindo spindulys /^, sudaromosios ilgis -
4
6 cm. Raskite:
V prizms =

V ritinio' a) kgio auktin;
b) kamp tarp sudaromj, per kurias eina ainis pjvis;
V prizms =
Tt
V c) kokiu atstumu nuo virns reikia nubrti ploktum,
lygiagrei pagrindui, kad kgio pjvio plotas bt lygus ainio
Atsv pjvio plotui.

Udaviniai. Ats.: a) 3<2 cm; b) 90; c) % cm.

1. Keturi rutuliai, kurispinduliai r, lieia vienas kit. lomel,


susidariusi tarp j, dtas dar vienas tokio pat spindulio rutulys. 99. Koordinai metodas
Raskite atstum nuo penktojo rutulio aukiausio tako iki
ploktumos. Tako koordinats tiesje

Atkarpos vidurio koordinats: Atkarpos AB takas M dalija atkarp


Ate.: n / ? (1 + V 2 ) .
AB santykiu k (skaitant nuo A), jei
AM = kMB (k f -1).
2. Rutulys lieia visas kubo briaunas. Raskite rutulio dalies, Tako M, dalijanio atkarp AB
esanios kubo viduje, tr, jei kubo briauna lygi a. Ilgis atkarpos kordinatmis santykiu k, koordinats:
(atstumas tarp dviej tak): X + kx 0
MN = |x,-X 2 1 x = 1 +k
A t e . ; - (15-8V2 1 ).
M(X 1 ) M(X 2 ) A
(xa) MjXjB(Xb)
3. Kgio pagrindo spindulys lygus 20 cm, atstumas nuo iI '
C(xv)
pagrindo centro iki sudaromosios lygus 12 cm. Raskite kgio
oninio paviriaus plot.
P l o k t u m o s tako koordinats
Ats.: 500 cm 2 .
Atkarpos vidurio koordinats:
4. Kgio auktin lygi 12 cm, o pagrindo spindulys lygus 3
AC = CB
cm. Kokiu atstumu nuo kgio virns reikia nubrti pjv,
lygiagret pagrindui, kad jo plotas bt lygus cm2 ? Atkarpos ilgis koordinatmis:

Ats.: 4 cm. AB = V(X-Xb)2 + ( v - v


Tiess lygtis Tako koordinats erdvje

Tiess lygtis: ax + by + c = O z,
Kubo tak koordinats
a, b, c f- O a= O b =O c = 0
y 0(0; 0; 0) A(0; a; 0)
i\\Ox ^ Il Oy Oei.
C(a; 0; 0)
B(a; a; 0)
0 , ( 0 ; 0; a) A 1 (0; a; a)
->x ->X B,(a; a; a) ; 0; a)

ax + by + c = O by+ c = 0 ax + c = O ax + by =

Tiess su krypties koeficientu lyg- Tiess lygtis (c t 0) Atkarpos vidurio koordinats:


n =1
tis (b = 0)
m -.A
y = kx + p AC = CB
Xa+ X b b_ Jya +. 7y.
b Z +
Zbh
2 2 ' 2
k = tga Xa'
-> -> X
O Atkarpos AB ilgis: O

Tiess, einanios per tak M(x 0 ; y) y = y0 + k(x - x0) 2 2
A B = V(xb - xa) + (yb - ya) + (zb - za) 2 aZ
su krypties koeficientu k, lygtis

Dviej tiesi lygiagretumo slyga k1 = k2


Tako M, dalijanio atkarp AB santykiu k, k p 1, koordinats
Tiesi statmenumo slyga K1-K2 = - I >
j
AM = k -MB ^ M( ^r^, Vrrb- ; " )
Kampas tarp dviej nestatmen ~ 1 + k,k2
tiesi

Ploktumos lygtis: T^a; b; c)


Apskritir no lygtis ax + by + cz + d = 0

Ploktum lygtis:
Su centru koordinai pradioje X2 + y2 = r2
x = 0 - ploktuma yOz
y = 0 - ploktuma xOz
Su centru take M(x0; y0) (X - X0)2 + (y - y0)2 = r2
z = 0 - ploktuma xOy

Bet kuri 2 + y2 + + by + = O tipo lygtis yra arba apskritimo, arba ax + by + cz + d = 0


tako lygtis, arba tuia aib. n(a, b, c ) l a
(normalinis vektorius)
100. Vektoriai ir koordinats Sprendimas.
Sakykime, kad vektoriai a ir b nekolinears, tada
-> -> :
a arba AB Pradia Pabaiga a ^ - 2b, t.y. + 2b 0 ir tada egzistuoja toks skaiius k, kad
>
~t - 3> = k ( t + 2.
I ia seka, kad 2k& + 3" = a- ka,"B(2k + 3) = a | l - k).
a - vektoriausjlgis (arba atkarpos ilgis)
Jei 2k + 3 7 0, tai b"= ^ ^
AB = l"a| = IABl
Jei 2k + 3 = 0, t.y. k = -1,5, tai 1 - k =s0, aiku a*= 2 I _+k3

Bet kuriuo atveju i lygybs seka, kad vektoriai "a i r i )


kolinears.
TT vienakrypiai; T l prieing krypi - kolinearus.
Gavome prietaravim.
P r i e t a r a v i m a s atsirado tarus, kad v e k t o r i a i a ir b
Pvz.
nekolinears. Vadinasi, vektoriai a ir b kolinears.
>

AD = . Nustatykite keturkampio ABCD r.


2. Atkarpa EF jungia tetraedro ABCD briaun AC ir BD
vidurio takus. rodykite, kad 2EF = BA + D . Ar vektoriai Ffe,
Sprendimas.
ir DC komplanars?
I lygybs = BC seka, kad keturkampio ABCD kratins
AD ir BC lygios ir lygiagreios. Vadinasi, ABCD - lygiagretainis.
Sprendimas.
A Kadangi takas E -j^tkarpgs A p
Vektori lygyb: "a = 1 o ^
vidurio takas, tai FE = FA + F C ,
2
Statmenieji vektoriai: " a l b <=> Z(si f S ) = 90
2RE = (FB_+ ( + DC) =
= ( + Dp) + (FB + FD).
Komplanars vektoriai - tai vektoriai, esantys vienoje
Kadangi FB + FDj= 0,
ploktumoje arba lygiagreiose ploktumose.
tai 2FE = BA + DCf
R n I to, kad vektoriFE galima ireikti
AA 1 , BB1 - komplanars
vektoriais ir Du, tai vektoriai FE,
DD Db, A - nekomplanars
BA ir DC komplanars.

Duota: AABC, AK = KB, AM = MC.


rodykite, kad KM IIBC1 KM = ^ B C .
1
Udaviniai. B' C

rodymas.
1. Vektoriai ~a + 2b~*ir "a - kolinears. rodykite, kad I slygos AK = KB seka AK^= y A.
slygos A M =_yiC
I Sc seka, AlOl = y A . _
vektoriai a ir b kolinears. >
.
t KM = AM - AK = I2AC - 2-
B e t AB = -1(A
2 V ~
- ! ) = 2\
Gautoji lygyb rodo, kad KM || BC ir KM = -1 .
Vektori sudtis Sprendimas.

BK = BM + MK.

Kadangi B~M = \ BA = - / = - f c , tai BK = - ~ c +"n.


>

BC = 2BK, vadinasi,
AB + BC = AC

BC = 2(--1 c + n)
M
BC = 2n - c

AK = AM + MK

AK = - ! " c + n*

Ats.: BC = 2n - c, AK = ~ c + n.

Sudties dsniai
Prieingi vektoriai
a> +
b> = b +a>

|~aj = I "b I \ _ ft / 7*
(a + b) + c = a + (b + c)

a + O= a

Vektoriaus dauginimas i skaiiaus

ka = m*; | m I = k I LAI ir jeigu k > O, tai a* ir m TT


k < 0, tai a* ir m T i .
Pvz.

Brinyje MK ^trikampio v i d u r i n j
linija. AB = ct MK =~n. Ireikkite BC
ir AK per duotuosius vektorius.
Skaliarin vektori sandauga Pvz. Duoti vektoriai ^(1; 0) ir b (1; 1). Raskite tok skaii k,
kad vektorius ~a + kb bt statmenas vektoriui a.
IaE* = |a*| |1 cos(la;l5)
Sprendimas.
Jei a i b , tai a b = 0. ~a(a + kt>) = O
et2 + kati= O
1 + k(1 -1 + 0 - 1 ) = O
1 + k = O
Pvz. Brinyje duota AB = BC, AD = DC, k = - 1
AC = 7, O - pusiaukratini susikirtimo
takas. Raskite AO Afc .
Afs.:-1.

A D ^
Atstumas tarp tak Afx1; y^ ir B(x2; y2):
Sprendimas. _^
AB = V(x2 - X1)2 + (y2 - y,)2
Sakykime, ZOAC = a, tada A O - A C = AO -AC- cosa.
A D B
AO DA: cosa = .
Atkarpos AB vidurio tako D(x; y) koordinats lygios:
I ia AO- AC = AO- AC- = AC- AD = AC-^p- = 24,5.
+
, + X,
i y, y,
Vektoriaus koordinats ploktumoje X= ^ r ; =^lTTj-

A B ( x 2 - X1; y 2 - y , ) , k u r A ( x , ; y , ) ; B ( x 2 ; y 2 ) .
Pvz. Duota: AABC, AM - pusiaukratin, A(2; - 6); B(5; 3),
Jei y 1 );J(x 2 ; y2), tai: 0(1; 1).
- = (X1 + x2; y , + y2)
Raskite: AM.
a ^ b = ( X 1 - x 2 ; y1 - y2)
ka = (kx^; ky t ) k e R Sprendimas.
aa + pb = (ax1 + ax2; Py1 + Py2), , e R BC vidurinio tako M koordinats yra (3; 2).
AM = V(3 - 2)2 + (2 + 6)2 = V1 + 64 = V65^.
> >

Vektori ir b skaliarin sandauga ab = X1X2 + y1y2.


Ais.: V657.
Jei " a l i a , tai X1X2 + y,y2 = O
Kampo tarp vektori a^x,; y,) irl?(x 2 ; y2) kosinusas:
Vektoriaus a*ilgis l~al = Vx12
+ y*', kur a*(x r y.,)
cosZ(a , b ) -
(a ir b kolinears) <=i> (- = )
Pvz. Lygiaoniame trikampyje ABC AB = BC = 8. Takas Kampo tarp vektori a(x,; y/, Z1) ir b(x2; y2; z2) kosinusas:
E dalija onin kratin AB santykiu^ : 1, t.y. BE : EA = 3 : 1.
Raskite kamp tarp vektori CE ir CA, jei C A = 12.
cosZ(a ; b) = ^ 2 + +

Sprendimas.
vedame koordinai sistem Oxy
Vektoriai I ^ x 1 ; y,; Z1) ir1b(x2; y2; z2) kolinears:
taip, kaip parodyta brinyje.
OA = OC
: OB = V b c 2 - o c 2 ' = 2V7". X2 y 2 2

AE = ^AB;
Jei ]_1 , tai: X1X2 + y,y 2 + Z1Z2 = O
> >

CE = CA + 4- AB
Vektoriaus AB vidurio tako D(x; y) koordinats, jei A(x^
7
21 . V7 v Z1), B(x2; y 2 ; z2):
Vadinasi, CA(-12; 0), AB(6; 2V7), C E ( - ^
x + y
z Z
> X
- V
2? -~
v 1
2 2
' 7
~
- '
2
Paymime Z(CE ; CA ) = a.

= (-12).(- f ) _ Pvz. Raskite vektoriaus AB ilg, jei A(3; 5; 1) ir B(5; 6; 3).


Tada cosa =
\2 m+-L
f Sprendimas.
.
Ats.: 3V7
IABU V(5 - 3)2 + (6 - 5)2 + (3 -1) 2 = VT+ 1 + 4 =
= V9 =3.
Vektoriaus koordinats erdvje
Ats.: AEsI = 3 .
Jei ^(X 1 ; y ^ z 1 ) iria (X2; y2; z2), tai:
- = (X1 + x 2 ; y 1 + y 2 ; Z1 + z 2 ) Pvz. Duoti keturi takai A(0; 1 ; - 1 ) , B1;-J;2); C(3; 1; 0),
a - b = (X1 - x 2 ; y, - y 2 ; Z1 - Z2)
D(2; - 3; 1). Raskite kampo tarp vektori AB ir CD kosinus.
ka = (kxn; ky,; kz^, k e R
> >
Sprendimas.
Vektori^ a ir b skaliarin sandauga: Randame vektoriaus AB koordinates ir ilg:
ab*= X1X2 + y ^ 2 + Z1Z2 1 . O = 1; - 1 - 1 = - 2; 2 - (- 1) = 3;

Vektoriaus a^(x ; y,; Z1) ilgis: IABI = Vl 2 + (- 2)2 + 3 2 = V l 4


I "a I = Vx21 + y + Z12'
>

Randame vektoriaus CD koordinates ir ilg:


2 - 3 = - 1 ; - 3 - 1 = - 4 ; 1 -O = 1;
Udaviniai.
I cdi = V ( - 1 ) 2 + (- 4) 2 + 1 2 =
1. Duotasgretasienis ABCDA 1 B 1 C 1 D 1 .
AB-CD
cos Raskite sum: Ef1C + AX 1 + CB +
^
+
_ 1* (- 1) (- 2)(- 4) + 3 1
Ais.; D^B.

5
cosy = 2. Raskite vektori "a(- 2; 3; 1) ir T)(5; 7; - 4) skaliarin
V63" '
sandaug.
Ats.:-5- .

Ats.: 7.

Pvz. Nagrinsime vektorius a*(\'x - 2 ; Vx ; 2) ir b(Vx - 2'; 3. Raskite kosinus kampo tarp vektori a ( - 1 ; 2; - 2) ir
V x - 3 ' ; - V x - +--). b(6; 3; - 6).
1. Apskaiiuokite skaliarin sandaug ab.
2. Su kuriomis reikmmis vektoriai a* ir b yra statmeni? Ais.: A .

Sprendimas. 4. Apskaiiuokite koordinai aies Oy tako, vienodai


nutolusio nuo tak A(2; - 1 ; 1) ir B(0; 1; 3), koordinates.
1. "A-- = V X T
Y W X - 2 + V Z - V x - 3' + 3(- V X - - + J ) =
= - 2 + Vx V x - I
T - 3 V x ' - + 2 = Vx^Vx - 3 - Vx = Vx ( V x - 3 ' - 3). Ats.: (0; 1;0).

2. Vektoriai apibrti, kai >. 3 ^ ^ 5. Trikampio virni koordinats yra A(3; - 2; 1), B(3; 1; 5),
Vektoriai a ir b statmeni, kai a- b = 0. C(4; 0; 3). Apskaiiuokite trikampio kampo kosinus.
Vadinasi, Vx (Vx- 3' - 3) = 0.
I ia Vx = 0 arba Vx - 3' - 3 = 0. Ats.: cos = ^ .
x= 0 Vx - 3 = 3
- 3= 9
= 12.

x = 0 (netinka, nes netenkina slygos > 3).

Ats.: 12.
101. Aibs 102. Kombinatorika
" -b a e A - a yra aibs A elementas, Pagrindins taisykls
b ^ A - b nra aibs A elementas.
Kombinatorins sudties taisykl.
/ a \
Jei aibs A ir B neturi bendr element, tai vien element
i j sjungos galima irinkti n(A) + n(B) bdais.
A c B - aib A yra aibs B poaibis,
t.y. kiekvienas aibs A elementas Pvz. Mama Andriui gimimo dienos proga paadjo nupirkti
a e A; b $ A
priklauso aibei B. magnetofon arba dvirat. Parduotuvje buvo 4 ri dvirai ir 6
ri magnetofon. Keliais bdais Andriui mama gali parinkti
dovan?
AcB

>
Sprendimas.
A U B - aibi A ir B sjunga, t.y. aib, Vien daikt galima pasirinkti 4 + 6 = 10 bd.

AUB
sudaryta i element, priklausani
nors vienai i aibi A arba B. Ats.: 10 bd.

Kombinatorins daugybos taisykl.


A D B - aibi A ir B sankirta, t.y. aib,
sudaryta tik i toki element, kurie I aibi A ir B galima sudaryti lygiai n(A) n(B) tokielement
yra bendri abiems aibms A ir B. por (a; b), a e A, b e B.
A4- J B

B Pvz. Pintinje yra 8 obuoliai ir 12 kriaui. Keliais bdais


galima pasirinkti ir obuol, ir kriau?
4 i
X - - - 4\ A\ B - aibi A i r B skirtumas, t.y aib,
B
sudaryta tik i toki aibs A element, Sprendimas.

kurie aibei B nepriklauso.
Av- ' - -pr
/
Ir obuol, ir kriau galima pasirinkti 8 12 = 96 bdais.
A\B
Ats.: 96 bdais.
0 arba {0} - tuia aib.
n(A) - aibs A element skaiius.

Aibi uraymo bdai:


A = {a , a2, ..., an} - aib, sudaryta i element av a2, ..., an.
Kliniai, gretiniai, deriniai Pvz. Klasje yra 15 berniuk ir 15 mergaii. Koncertui reikia
okj poros, daininink poros ir skaitov poros (kiekvien por
sudaro berniukas ir mergait). Kiek yra bd tai padaryti?
Faktorialas n! = 1 - 2 - 3 ... n, kai n 5-2

1! = 1 , 0 ! = 1.
Sprendimas.
I 15 berniuk parinkti 1 okj, 1 daininink ir 1 skaitov
pVZm 4! = 4 - 3 - 2 1 = 2 4 . galima A15 bd. I 15 mergaii parinkti 1 okj, 1 daininink ir
1 skaitov yra A 15 bd. Pagal sandaugos taisykl okj,
daininink ir skaitov poroms parinkti yra A^5- A15 bd.
Kliniai
Ats.: (15-14-13)2.
Bet kuris tam tikro skaiiaus objekt dstinys vadinamas
kliniu.
Deriniai
Pvz. I skaitmen 1 , 2 , 3 , 4 , 5 sudaromi penkiaenkliai skai-
Junginiai i n element po k, kurie skiriasi vienas nuo kito
iai, nesidalijantys i 5 ir neturintys vienod skaitmen. Kiek yra
nors vienu elementu, vadinami deriniais.
toki skaii?
_ n(n - 1 ) . . . (n - k + 1)
Sprendimas. 4 k!
I penki skirting skaitmen galima sudaryti P 5 penkia-
Ck = n!
enkli skaii. Skaii, kurie dalijasi i 5, t.y. j paskutinis skaitmuo n k!(n - )!

5, yra P 4 . Vadinasi, udavinio slygas tenkina P 5 - P 4 = 5! - 4! -


Cnk = Cnn "k
= 120 - 24 = 96 skaiiai.

Ats.: 96 skaiiai.
Cnk"1 + cin = cn+1
k
,

C = 1 ; C > n ; Cnn = 1
Gretiniai
Pvz. I 20 firmos darbuotoj 5 mons turi vykti tobulintis.
Vadybininkas ir du vyresnieji ininieriai negali ivykti vienu metu.
Bet kuris k (k ^ n) element, paimt i n element aibs,
Kiek gali bti skirting vykstanios grups sudi?
dstinys vadinamas gretiniu i n element po k.
Sprendimas.
I 20 moni irinkti penkis yra C250 bd. I io skaiiaus
reikia atimti tuos variantus, kai vadybininkas ir du vyresnieji ininieriai
Akn = n(n - 1 ) . . . (n - k + 1)
eina j vaiuojanij grup. Jei vadybininkas ir du vyresnieji
ininieriai eina penki moni grup, likusias dvi vietas parinkti
2 mones i septyniolikos yra C27 bd.
Vadinasi, vykstanios grups sudi gali buti: 103. Tikimybi teorija
5
P p 2 _ 20! 17! - - I t r o f i Q
2 0 17 ~ ^ ! 5 ~ 25 ~ l t K j b a
[vykis

Ats.: 15368. Bandymo arba stebjimo rezultatas vadinamas vykiu.

Du vykiai vadinami sutaikomais, jei jie gali vykti kartu to


Paskalio trikampis paties bandymo metu.

Paskalio trikampis yra binomo (a + b)n skleidinio koeficient Pvz. Bandymas. Metamas loimo kauliukas. vykis A -
lentel:
atsivert 4 akuts. vykis B - atsivert lyginis akui skaiius. vykiai
A ir B sutaikomi.
n = O C
n = 1 C C1
n - 2 C20 C^ C22 Du vykiai vadinami nesutaikomais, jei jie negali vykti kartu
n = 3 C30 C31 C32 C33 to paties bandymo metu.
n = 4 C40 C41 C42 C43 C44
Pvz. Bandymas. Vien kart metamas loimo kauliukas.
Pvz. vykiai A1, A2, A3 - atitinkamai ikrito viena, dvi, trys akuts. Tie vykiai
1 yra nesutaikomi.
1 1
1 2 1 Du vykiai A ir A vadinami prieingais, jei to paties bandymo
1 3 3 1 metu jie nesutaikomi ir vienas j btinai vyksta.
1 4 6 4 1
Pvz. Bandymas. Metama moneta. vykis A - atsivert herbas,
vykis A - atsivert skaiius.
Binomo formul
vykis vadinamas btinu, jei atlikus bandym, jis visada
(a + b)n = a" + c V " 1 b + C2nan-2b2 + ... + b". vyksta.
vykis vadinamas negalimu, jei atlikus bandymjis niekada
Pvz, nevyksta.

(a + b)4 = a4 + 4a 3 b + 6a2b2 + 4ab 3 + b4; Pvz. Bandymas. I ds, kurioje visi rutuliai balti,
(a + b)5 = a 5 + 5a 4 b + 10a3b2 + 10a2b3 + 5ab4 + b5. traukiamas rutulys. vykis A - itrauktas baltas rutulys - btinas.
vykis B - itrauktas juodas rutulys - negalimas.

vykis vadinamas atsitiktiniu, jei atliekant bandymjis gali


vykti, gali ir nevykti.
Atsitiktini vyki aib: A = (W1, W2, ... wn}.
-219-
Pvz. vykis - ikrito 6 akuts metant loimo kauliuk - 2. Dje yra 20 vienod rutuli, i kuri 12 balti ir 8 juodi.
atsitiktinis. Jis galjo ir nevykti. Atsitiktinai iimami du rutuliai. Raskite tikimyb vykio, kad vienas
j baltas, kitas juodas.
vyki A ir B suma vadinamas vykis C = A + B, kuris reikia,
kad vyko arba vykis A, arba vykis B. Sprendimas.
vyki A ir B sandauga vadinamas vykis C = A - B , kuris m = 12 -8
reikia, kad vyko ir vykis A, ir vykis B. n = C220

Pvz. Bandymas. Du auliai auna taikin po vien kart. - 12-8 - 12-8-2.1 _ 48 _ n c


U
Cl20 20-19 95 '
vykis A - pataiko taikin pirmasis aulys. vykis B - pataiko taikin
antrasis aulys. vykis A + B reikia, kad pataik taikin pirmasis Ats, ~ 0,5.
arba antrasis aulys.
vykis AB - pataik taikin ir pirmasis, ir antrasis auliai. Tikimybi savybs

Nagrinjamo bandymo vyki aib, kai vienas j bandymo Dviej nesutaikom vyki sjungos tikimyb lygi t vyki
metu btinai vyksta, o bet kurie du i j nesutaikomi, vadinama to tikimybi sumai.
bandymo elementarij vyki aibe. P(A U B ) = P(A) + P(B).
Elementarij vyki aib: u = {u,, u2, ..., un}.
Pvz. Bibliotekoje atsitiktine tvarka lentynoje sustatyta 15
Pvz. Bandymas. Metamas loimo kauliukas. I vyki U1 - vadovli, kuri 5 nauji. Bibliotekinink atsitiktinai pam 3
"atsivert dvi akuts", u 2 - "atsivert keturios akuts", U3 - "atsivert vadovlius. Raskite tikimyb vykio, kad bent vienas paimt
eios akuts" susideda vykis u - "atsivert lyginis akui skaiius". vadovli naujas.

Tikimybs apibrimas Sprendimas.


Sakykime, vykis B - vienas vadovlis naujas. vykis C - du
P(A) = ^ vadovliai nauji. vykis D - trys vadovliai nauji.
Iekomas vykis A = B + C + D.
vykio A tikimybe vadinamas santykis vykiui A palanki P(A) = P(B) + P(C) + P(D)
vyki skaiiaus su vis galim elementarij vyki skaiiumi.
C2-C1 45
P(B) = - * 91
Pvz,
20
1. Raskite tikimyb vykio, kad metant loimo kauliuk, ikris P(C) = 91
lyginis akui skaiius (vykis A).
C3 2
P(D) 91
Sprendimas.
n = 6, m = 3 91 91 91 91
P(A) = - f = 0,5.
Prieingo vykio tikimyb Jei A ir B - priklausomi vykiai, tai slygine tikimybe P A (B)
vadinama vykio tikimyb, apskaiiuota inant, kad vykis A jau
vyks.
Prieing vyki tikimybi suma lygi vienetui:
D v i e j p r i k l a u s o m v y k i s a n d a u g o s t i k i m y b lygi
P(A) + P(A) = 1.
sandaugai vieno i vyki tikimybs ir kito vykio slygins
tikimybs: P(AB) = P(A)P A (B).
Pvz. Dje yra 10 detali, i kuri keturios daytos.
Atsitiktinai itrauk tris detales. Raskite tikimyb vykio, kad itraukta
Pvz. Firmoje dirba septyni vyrai ir trys moterys. Pagal srao
bent viena dayta detal.
numerius atsitiktinai parenkami trys mons. Raskite tikimyb vykio,
kad visi irinkti mons yra vyrai.
Sprendimas.
vykis A - n viena itrauktj detali nra dayta.
Sprendimas.
pr M = _2L = 6
5'4 = 1. Sakykime, vykis A - pirmasis irinktas mogus yra vyras,
vykis - antrasis irinktas vyras, vykis C - treiasis irinktas vyras.
r
VV c,30 10 98 6 Tikimyb vykio, kad antrasis irinktas vyras su slyga, kad
pirmasis irinktas jau buvo vyras, t.y. slygin tikimyb vykio B
Iekoma tikimyb P(A) = 1 - y = f - .
Ats.: f . lygi: P a (B) = I - = I - .
Tikimyb vykio, kad treiasis bus irinktas vyras su slyga,
kad pirmieji du irinktieji yra vyrai, t.y. slygin tikimyb vykio C
Sankirtos tikimyb iygi: P a b (C) = f -

N e p r i k l a u s o m vyki sankirtos tikimyb lygi t vyki Iekomoji tikimyb, kad visi trys irinktieji yra vyrai lygi:
tikimybi sandaugai.
P ( A B C ) = P(A)- PA(B). PAB(C) = 4 4 = , .
Ats - 24
M l s
Pvz. Metama moneta ir loimo kauliukas. Raskite tikimyb
vykio, kad atsivers herbas ir lyginis akui skaiius.

Sprendimas. Pilnos tikimybs formul


Tikimyb vykio B - atsivers herbas - lygi: P(B) = ^ . Tikimyb
Tikimyb vykio A, kuris gali vykti tik kartu su vienu i vyki
vykio C - atsivers lyginis akui skaiius - lygi: P(C) = Y = Y
B 1 , B 2 , ... BN, kurie sudaro piln tarpusavyje nesutaikom vyki
Iekoma tikimyb: P(A) = P(B)-P(C).
grup, lygi sandaug kiekvienos hipotezs tikimybs i atitinkamos
vykio A slygins tikimybs sumai:
P(A) - y ' y 4~
P ( A ) = P ( B 1 ) - P ( A ) + P ( B 2 ) - P B ( A ) + ... + P ( B N ) - P B ( A ) , k u r
2
P ( B 1 ) + P ( B 2 ) + ... + P ( B N ) = 1.
Ats.: f4 .
Pvz. d, kurioje yra du rutuliai, dtas baltas rutulys, po Pvz.:
to atsitiktinai traukiamas vienas rutulys. Raskite tikimyb vykio, kad
atsitiktinai itrauktas rutulys bus baltas. 1. Kokia tikimyb, kad metant loimo kauliuk deimt kart,
3 akuts atsivers lygiai 2 kartus?
Sprendimas.
Sakykime, vykis A - itrauktas baltas rutulys. Galimos ios Sprendimas.
hipotezs apie pirmin rutuli sudt dje: B 1 - balt rutuli dje
n = 10, k = 2, p = - , q = 1 - p = J - .
nebuvo, B 2 - vienas baltas rutulys buvo dje, B3 - buvo du balti
rutuliai dje. 2
P(A) = C 1 ^ M f)8 - f ^ 0,29.
Vis trij hipotezi tikimybs lygios:
P(B1) = P(B2) = P(B3) = - .
Afs.; = 0.29.
Slygin tikimyb vykio, kad itrauktas baltas rutulys, jei i 2. Monet meta penkis kartus. Raskite tikimyb vykio, kad
pradi dje nebuvo balt rutuli, lygi: P b (A) = y . herbas ikris maiau negu du kartus.

Slygin tikimyb vykio, kad itrauktas baltas rutulys, jei i Sprendimas.


pradi dje buvo vienas baltas rutulys: P 82 (A) = y .
Iekomoji tikimyb P = P5(O) = P 5 (I) = C5I y) - (y )5 +

Slygin tikimyb vykio, kad itrauktas baltas rutulys, jei


+ C1 5 ( ! )4 =3 2^
2 * 2 '
+ 3A2 =
32 16
i pradi dje buvo du balti rutuliai, lygi: P^(A) = y = 1.

Iekoma tikimyb:
P(A) = P(B 1 )-P 61 (A) + P(B2)Pes2(A) + P(B 3 )P a5 (A) =
Atsitiktiniai dydiai

Atsitiktinis dydis yra toks dydis, kuris po bandymo gyja


Ats.: f-. konkrei, i anksto neinom skaitin reikm.
Atsitiktinis dydis ymimas X. X1, X2 xn - atsitiktinio dydio
reikm, p v p 2 p n - t reikmi tikimybs.
Bernulio bandymas
Atsitiktinio dydio skirstinys
Nepriklausomi bandymai, kai pirmame bandyme stebime
vyk A1, antrame - A2, ... , n - tame An; P(A1) = P(A2) = ... = P(An) = X X X X
1 2

= p, vadinami Bernulio bandymais. P


P1 P2 Pn
Tikimyb, kad vyks k stebim vyki, lygi:
P(k) = C > q n q = 1 - p . Atsitiktinio dydio matematin viltis (vidurkis)

E X = X 1 P 1 + X 2 P 2 + ... +

- 225 -
Atsitiktinio dydio dispersija
Pvz.: 1. Raskite matematin vilt atsitiktinio dydio X, kurio
skirstinys:
DX = E(x - EX)2 arba DX = EX2 - (EX)2.
X -4 6 10
P 0,2 0,3 0,5
Pvz.
Tarp 10 detali yra 8 standartins detals. Atsitiktinai Sprendimas.
irenkamos dvi detals. Atsitiktinis dydis - standartini detali E(X) = - 4 - 0 , 2 + 6 - 0 , 3 + 10-0,5 = 6.
tarp irinktj skaiius. Paraykite jo skirstin.
Ats.: 6.
Sprendimas.
Atsitiktinis dydis turi galimas reikmes X1 = 0; X2 = 1,
2. Raskite dispersij ir vidutin kvadratin nuokryp atsitiktinio
3 = 2 p o r 0
dydio X, kurio skirstinys:
Tada P(x = 0 ) = ^ = ^ .
X -5 2 3 4
P 0,4 0,3 0,1 0,2
P ( X = I ) = ^ = I -
Sprendimas.
p ( x = 2) = - ^ = f . E(X) = - 5 - 0 , 4 + 2 - 0 , 3 + 3 - 0 , 1 + 4 - 0 , 2 = - 0,3.

Iekomasis atsitiktinio dydio skirstinys: Paraome x2 skirstin:


X2 25 4 9 16
X 0 1 2
P 0,4 0,3 0,1 0,2
P 1 16 28
45 45 45
E(X2) = 25-0,4 + 4 - 0 , 3 + 9 - 0 , 1 + 16-0,2 = 15,3
D(X) = E(X)2 - (E(X))2 = 15,3 - (- 0,3)2 = 15,21.
Patikrinimas: 4-- + 41 + -f- = 1.
() = VD(X)* = V l 5 ^ T = 3,9.

Vidutinis kvadratinis nuokrypis = VDX .


Binominio skirstinio matematin viltis
Binominis skirstinys
E(X) = n -P, kur n - bandym skaiius, P - vykio pasirodymo
Binominiu vadinamas skirstinys atsitiktinio dydio - vykio
tikimyb viename bandyme, kur P = P n = C n p k q n_k
pasirodym n nepriklausomuose bandymuose skaiiaus, kuri
kiekviename vykio pasirodymo tikimyb lygi p; tikimybs galimos
Binominio skirstinio dispersija
reikms = k (vykio pasirodymo skaiiaus k) skaiiuojama pagal
Bernulio formul P n (k) = C n pkqn k, kur q = 1 - p.
D(X) = npq
X 0 1 2 n-1 N
P PN(O) P ( D P ( 2 ) P n (n - 1 ) P N ( H )

kur P n (k) = C n p k q n " k


Pvz.: 1. Prietaisas susideda i trij nepriklausomai veikiani
element. Kiekvieno elemento nustojimo veikti vieno bandymo metu 104. Statistika
tikimyb lygi 0,1. Paraykite element nustojimo veikti vieno
bandymo metu skaiiaus skirstin. imtis

Sprendimas. Tyrimui parinkta objekt dalis vadinama imtimi.


Atvirktinis dydis (nustojusi veikti e l e m e n t vieno Imties stebiniai: X1, x2, ... , xn
bandymo metu skaiius) gali gyti ias reikmes: X1 = 0 (n vienas Imties element skaiius yra imties tris.
elementas nenustojo veikti), X2 = 1 (vienas elementas nustojo veikti), Variacin eilut yra nemajaniai sutvarkyta imtis.
X3 = 2 (nustojo veikti du elementai) ir X4 = 3 (nustojo veikti visi trys
elementai). Pvz. Imtis ireikta skaiiais X1 = 3; X2 = 2,6; X3 = 1,5. Jei
T i k i m y b s n u s t o t i veikti k i e k v i e n a m elementui duomenis suraome didjania tvarka, tai gauname imties variacin
(nepriklausomai vienam nuo kito) lygios. eilut 1,5 2,6 3.
Vadinasi, n = 3, p = 0,1, q = 1 - p = 0,9. ios imties tris yra 3.
P3(O) = q 3 = 0,93 = 0,729;
P 3 (I) = C 3 pq 2 = 3 -0,1 0,9 2 = 0,243; Imties plotis
P 3 (2) = C 2 p 2 q = 3 0,1 2 0,9 = 0,027;
P 3 (3) = p3 = 0,13 = 0,001.

Iekomas binominis skirstinys: Pvz. Imtis 2; 3; 5; 6.


X 0 1 2 3 Plotis = 6 - 2 = 4.
P 0,729 0,243 0,027 0,001
Imties centras
2. Raskite matematin vilt atsitiktinio dydio - skaiiais,
toki penki loimo kauliuk metim, ant kuri dviej atsivert po C
vien akut, jei i viso buvo 20 metim.
Pvz. Imties 2; 3,5; 6 centras C = 4.
Sprendimas.
Randame P - tikimyb vykio, kad ant dviej i penki
kauliuk atsivert po vien akut.
Mediana
Taikome formul E(x) = nP, kur n - bendras b a n d y m
(penki kauliuk metim) skaiius. X - mus dominani vyki (ant Mediana vadinamas skaiius, dalijantis imties tr dvi lygias
dviej kauliuk i penki atsivert po vien akut) skaiius n dalis.
bandymuose; P - tikimyb vykio, kad ant dviej kauliuk atsivert
po Vienat akut metant vien kart 5 kauliukus. Pvz. Imties 2; 6; 8; 9; 10 mediana lygi 8, o imties 2; 3; 4; 5;
Pagal slyg n = 20. Atsiversti vienai akutei ant vieno 6; 8 mediana lygi = 4,5.
kauliuko tikimyb yra p = 4 - . Tada q = - f - .
, ; p - 2 > ./ 6 An 3 - 5-j- 6_ Skaiius, parodantis, kiek kart elementas pasikartoja imtyje,
is cia - g ) ( 6 ) ~ b ?. 6 s - 3.64 54
vadinamas to elemento daniu ir ymimas na.
Iekomoji matematin viltis () = - P = 20 - ^ = 3.
Ats.: = 3. - 229 -
mties 4; 5; 3; 7; 4; 5; 3; 5; 6 dani lentel yra
X 3 4 5 6 7
m. 2 2 3 1 1

Elemento danio m j santykis su imties element skaiiumi


n vadinamas santykiniu daniu ir ymimas P i '.

1 4 5 . 7

Intervalas, kuriame telpa stebjimo duomenys X1, x 2 , . . . , xn,


Pvz. Duota imties dani lentel: skaidomas kelis dalinius h ilgio intervalus.
X 2 5 7 D a n i h i s t o g r a m a v a d i n a m a figra, s u s i d e d a n t i i
nI 1 3 6 staiakampi, kuri pagrindai yra h ilgio daliniai intervalai, o aukiai
Raskite santykinius danius. lygs .

Sprendimas. Pvz. Duota imtis, suskaidyta h = 4 ilgio dalinius intervalus.


X [1; 5) [5; 9) [9; 13) [13; 17) [17; 21)
h
Randame imties tr: n = 1 + 3 + 6 = 10 m
i 10 TO 50 12 8

D

1 I -~ j10
_ . P I JL r p ' - J L
' 2 10 3 10 Nubraiykite histogram.
Paraome santykini dani lentel:
X 2 5 7 Sprendimas.
p; 0,1 0,3 0,6 Randame santykius: . Cia h = 4.

Tada 10 2,5; f =5; f = 12,5; f12 =_ 3; f = 2.


4
Poligonas ir histograma

13 - -
Dani poligonu vadinama laute, kurios atkarpos jungia 12
takus (X1; m,), (X2; m 2 ), ... , (xk; m k ).

Pvz. Imties dani lentels


X 1 4 5 7
5
mI 20 10 14 6
3
poligonas atrodo taip:
2

13 17 21
Imties intervalas Imties vidutinis kvadratinis nuokrypis

Imties vidurkis =
X + X + ... + X1
s = Vs2"

Sugrupuotos imties vidurkis: Pvz. Raskite imties skirstinio


Xi 186 192 194
mI 2 5 3
Pvz. Duota imties dani lentel:
X 2 5 7 10 vidutin kvadratin nuokryp.
mI 16 12 8 14
Sprendimas.
Raskite x. Kadangi Xi - dideli skaiiai, tai galima i Xi reikmi atimti
skaii 191, artim skirstinio vidurkiui. Tada
Sprendimas. U 1 = 186 - 191 = - 5,
16-2 + 12.5 + 8.7 + 14 -10 c U 2 = 192 - 191 = 1,
= so = 5,76
U 3 = 194 - 191 = 3.
Gauname sa lygin skirstin:
Imties dispersija uI -5 1 3
m. 2 5 3
Imties dispersija: S2 = 2-)4 + ... + (x k -x)4
I ia
Q2 252 + 512 + 332 /2.(- 5) + 51 + 33 ? _

10 ^ 10 >-
Pvz. Strypo ilgis buvo matuojamas penkis kartus tuo paiu
prietaisu. Gauti tokie rezultatai (milimetrais): 92; 94; 103; 105; 106. = 8 , 2 ^ 0 , 1 6 = 8,04.
Raskite imties dispersij. S = /8,04" = 2,84.

Sprendimas. Ats.: = 2,84.


92 + 94 + 103 + 105 + 106
Randame iiimties vidurk: = , = 100.

Dispersija: Koreliacijos koeficientas

g 2 _ (92 - 100)2+ (94 - 100)2 + (103 - 100)2 + (105 - 100)2+ (106 - 100)2 _ ^ Sakome, kad tarp dviej atsitiktini dydi ir y yra
tikimybikai tiesin koreliacija, jei pirmajam didjant, antrasis turi
Ats.. 34. t e n d e n c i j didti (arba mati) apytikriai pagal tiesin
priklausomyb y = ax + b (a ir b - pastovs teigiami ar neigiami
skaiiai).
Atsitiktini dydi X ir Y koreliacinis koeficientas I lentels randame:
E(X) = ( 5 - 2 + 8 - 2 + 1 0 - 5 + 12-12 + 1 5 - 6 +
r(X y \ - E(XY)-E(x>E(y)
* ' ' ()() + 17-11 + 20-10 + 25-15 + 28-20 + 30-10 + 32-4 +35-1 +
+ 4 0 - 1 + 4 5 - 1 ) = 21,80.
Tiesins koreliacijos koeficientas turi tokias savybes:
1) | r ( X , Y ) k 1 . Panaiai apskaiiuojame E(Y) = 19,32.
2) Jei r(X, Y) = + 1, tai y = ax + b, t.y. tarp atsitiktini E(X, Y) (5-2- 5 + 5 - 2 - 8 + 8 - 5 - 1 2 +
dydi X ir Y yra tiksli tiesin koreliacija. + 10-10-5 + 10-12-7 + 16-6-17 + 18-6-15 + 20-10-30 +
3) Nepriklausom atsitiktini dydi koreliacijos koeficientas + 21-10-20 + 2 2 - 5 - 1 7 + 22- 20- 28 + 4 2 - 4 3 2 + 31 35 +
lygus nuliui. + 35- 40) = 467,43
4) Jei r(X, Y) = O, tai tarp atsitiktini dydi X ir Y negali bti
tikslios tiesins koreliacijos. D(X) = E(X2) - E(X)2 = 77,11
() = 8,8
Pvz.: 1. Skirtingose alies vietose buvo tirta 100 sklyp. D(Y) = E(Y 2 )-E(Y) 2 = 92,14
Kiekvieno sklypo pus buvo patrta bandomosiomis tromis, kita ( Y ) - 9 , 6
pus - ne. Sklypuose buvo pasti kvieiai. Numus derli, gauti
/ v, _ 467.43-21,8-19,32
tokie rezultatai:
V
. Y J - 8,8-9,6
Sklyp skaiius 2 2 5 7 5 6 6 5 1 0 1 5 20 1 0 4 1 1 1
Derlingumas 5 8 10 12 12 15 17 17 20 25 28 30 32 35 40 45 r(X, Y) = 0,55
patrus (cnt/ha)
Derlingumas 5 5 10 10 8 18 16 22 21 22 22 20 42 35 35 20 r(X, Y) - 0, vadinasi, tarp atsitiktini dydi X ir Y yra tiesin
netrus (cnt/ha) koreliacija y = ax + b, tai yra panaudotos tros turi takos kviei
Atsitiktinis dydis X - derlingumas patrtuose sklypuose, derlingumui. Kadangi E(X) > E(Y) ir r(X, Y) > 0, tai patrtose sklyp
dydis Y - netrtuose. pusse vidutinis derlius didesnis.
Raskite koreliacijos koeficient. Ar panaudotos tros turi
takos kviei derlingumui? 2. Duota atsitiktini dydi ir y skirstinio lentel:
x\y -1 0 1
Sprendimas. 0 0,1 0.15 0,2
Sudarome koreliacin lentel: 1 0,15 0,25 0,15
X5 8 1 0 1 2 1 5 17 2 0 2 5 2 8 3 0 3 2 3 5 4 0 4 5 X
Raskite koreliacijos koeficient r(X, Y).
5 22 4
8 5 5
10 5 7 12
16 6 6
Sprendimas.
18 6 6 _ E(XY)-E(X)E(Y)
20 10 1 11
21 10 10
^y4' t
^ o(x)o(y)
22 5 1520 40
E(X; Y) = 0 - ( - 1)-0,1 + 0 - 0 - 0 , 1 5 + 0 - 1 - 0 , 2 +
35 1 1 2
42 4 4 + 1 - (- 1)-0,15 + 1 - 0 - 0 , 2 5 + 1 -1-0,15 = 0.
nx 2 2 5 1 2 6 11 1 0 1 5 2 0 1 0 4 1 1 1 n = 100
Sudarome dydi X i r Y skirstini lenteles: 4. Firmos sekretor turi isisti 5 ratus 5 adresatams. Keliais
O 1 bdais, sekretorei suklydus, visi laikai bus isisti ne ten, kur reikia?
0,45 0,55 - skirstinio X dsnis
Ats.: 44.
-1 0 1
0,25 0,4 0,35 - dydio Y skirstinio dsnis. 5. Jonas ir Egl susitar su 8 savo draugais kartu sutikti
Naujuosius metus. Kokia tikimyb, kad u apvalaus veni stalo
E(X) = 0 0,45 + 1- 0,55 = 0,55 jie suss greta, jei visi sveiai sodinami atsitiktinai?
E(Y) = (- 1). 0,25 + 0 - 0 - 0,4 + 1. 0,25 = 0,1.
D(X) = E(X2) - E(X)2 = 0,55 - 0,552 = 0,2475 Ats.:-
() = /0,24751 = 0,497
D(Y) = M(Y2) - (M(Y)2 = 0,6 - 0,12 = 0,59 6. Dje yra 90 tinkam naudoti ir 10, kuriems pasibaigs
() = 0,59 vartojimo terminas, aliejaus buteli. Kokia tikimyb, kad tarp 10
() = / = 0,768 atsitiktinai iimt aliejaus buteli nebus n vieno su pasibaigusiu
vartojimo laiku?
- / \ - 0 - 0,550,1
(
'
> * 0,497-0,768
0,144.
Ats.: 0,33.
Ats.: = -0,144.
7. Loterijoje yra 1000 biliet. I j vienas bilietas iloia 500
Lt, 10 biliet - po 100 Lt, 50 biliet - po 20 Lt ir 100 biliet - po 5 Lt.
Udaviniai Likusieji nieko neiloia. Kokia tikimyb, kad mogus, nusipirks 1
biliet, ilo ne maiau 20 Lt?
1. Treniruotes lanko 12 krepinink. Keliais bdais gali bti
sudarytas pagrindinis penketukas? Afs.: 0,061.

Ats.: 792. 8. Metami du loimo kauliukai. Kokia tikimyb, kad bent


ant vieno j atsivers 6 akuts?
2. Keliais bdais galima idstyti achmat lentoje 8
boktus, kad vienas kito jie negalt kirsti? Ats.: 41
36
.

Ats.: 40320. 9. Moneta metama tris kartus. Raskite tikimyb vykio, kad
herbas atsivers lygiai du kartus.
3. Laukini vri dresuotojas turi ivesti cirko aren 5 litus
ir 4 tigrus. Keliais bdais jis gali sugrupuoti vris taip, kad 2 tigrai Ats.: 4-
neit vienas paskui kit?

A ts. :43200.
10. I skaitmen 1; 2; 3; 4; 5 atsitiktinai irenkamas vienas,
Trio vienetai
o i likusi - antras. Raskite tikimyb vykio, kad abu irinktieji
skaitmenys nelyginiai.
mm 3 cm 3 dm 3 m3 km3
1 mm 3 1 - 3 - 6 - 9 1 0 -18
Ats.: 0,3.
1 cm2 103 1 - 3 - 6 - 15
1 dm 3 106 103 1 10-3 10 12
11. Atsitiktinis dydis X duotas skirstinio lentelje:
1 m 3 109 106 103 1 10-3
X 1 2
1 km3 18
1 0
1015 1012 109 1
P 0,5 0,2
Raskite E(x), D(x) iro(x).
1 i = 1 dm 3
Ats.: E(x) = 0,9; D(x) = 0,49; () = 0,7.
Mass vienetai
12. Apskaiiuokite imties 4, 2, 6, 2, 3, 3, 2, 6,4, 7 dispersij.
mg g kg t
3
Ats.: 3,09. 1 mg 1 - - 6 10-9
1 g 103 1 10-3 10-6
1 kg 106 103 1 10-3
1t 109 106 103 1
105. Dydi sryi lentels

Ilgio vienetai
Laiko vienetai
mm cm dm m km
1km r T o 6 105 104 103 1
s min h
1 m 103 102 10 1 -3
1s 1 1,667-10" 2 2,778-10" 4
1 dm 102 10 1 -1 10 4
1 min 60 1 1,667.10 2
1 cm 10 1 - 1
- 2
10"5
1h 3,6.10 3 60 1
1 mm 1 10"1 10" 2
-3 10"6

Ploto vienetai

mm 2 cm 2 dm 2 m2 a ha km 2
2
-6 10"8 1 0 -io 10-12
1 mm 1 10"2 10"4
1 cm 2 10 2 1 -2 10^ 10' 6 - 8 -10-10
1 dm2 10 4 10 2 1 -2 10 -4 10 -6 - 0-8
1 m 2 10 6 10 4 10 2 1 10"2 10 4 10 -6

1 a 108 10 6
10 4
102 1 10-2 104
1 ha 1 0 i o 108 106 104 102 1 10-2
1 km2 1012 10io 108 106 104 102 1
Grebenienkait P., Tumnait E. Matematikos
korepetitorius namuose.

Leidinyje glaustai ir aikiai pateikiama teorin ir


praktin madiaga, apimanti vidurins mokyklos kurs.
Sis korepetitorius abiturientui pads savarankikai pasikar-
toti ir mokytis sprsti udavinius namuose.
Leidinys skirtas moksleiviams ir mokytojams.

Virelio dailinink Lina utautien


Techniniai redaktoriai:
Saulius Stropus, Jolanta Stropuvien

SL 2031. 2002 08 10. 15 sp.l.


Ileido "iaurs Lietuvos" leidykla
(tel. 8 -686 35112)
Spausdino "Maoji poligrafija",
A. S m e t o n o s al. 35-1, Kaunas
Kaina sutartin

También podría gustarte